Vascular Surgery: Cases, Questions and Commentaries, Third Edition - PDF Free Download (2024)

Vascular Surgery Third Edition

George Geroulakos  •  Bauer Sumpio (Editors)

Vascular Surgery Cases, Questions, and Commentaries Third Edition

Editors George Geroulakos, MD, FRCS, FRCSE, DIC, PhD President, Section of Vascular Medicine Royal Society of Medicine London, UK and Consultant Vascular Surgeon Charing Cross Hospital London, UK and Senior Lecturer Imperial College London, UK

Bauer Sumpio, MD, PhD, FACS Professor of Surgery and Radiology Yale University, New Haven, CT, USA and Chief, Vascular Surgery Service Yale-New Haven Hospital New Haven, CT, USA and Director, Heart and Vascular Center of Excellence, Yale Medical Center New Haven, CT, USA and Director, Vascular Surgery Residency and Fellowship Training Program Yale Medical Center, New Haven, CT, USA

ISBN 978-1-84996-355-8 3rd edition e-ISBN 978-1-84996-356-5 3rd edition ISBN 978-1-85233-963-0 2nd edition e-ISBN 978-1-84628-211-9 2nd edition ISBN 978-1-85233-533-5 1st edition DOI  10.1007/978-1-84996-356-5 Springer London Dordrecht Heidelberg New York British Library Cataloguing in Publication Data A catalogue record for this book is available from the British Library Library of Congress Control Number: 2010930072 © Springer-Verlag London Limited 2011 First published 2003 Second edition 2006 Third Edition 2011 Whilst we have made considerable efforts to contact all holders of copyright material contained in this book, we may have failed to locate some of them. Should holders wish to contact the Publisher, we will be happy to come to some arrangement with them. Apart from any fair dealing for the purposes of research or private study, or criticism or review, as permitted under the Copyright, Designs and Patents Act 1988, this publication may only be reproduced, stored or transmitted, in any form or by any means, with the prior permission in writing of the publishers, or in the case of reprographic reproduction in accordance with the terms of licences issued by the Copyright Licensing Agency. Enquiries concerning reproduction outside those terms should be sent to the publishers. The use of registered names, trademarks, etc. in this publication does not imply, even in the absence of a specific statement, that such names are exempt from the relevant laws and regulations and therefore free for general use. Product liability: The publisher can give no guarantee for information about drug dosage and application thereof contained in this book. In every individual case the respective user must check its accuracy by consulting other pharmaceutical literature. Cover design: eStudioCalamar, Figueres/Berlin Printed on acid-free paper Springer is part of Springer Science+Business Media (www.springer.com)

Dedicated to The late Dr. Robert W. Hobson II Co-editor of the 1st and 2nd edition of this book. Humanitarian, charismatic surgeon, distinguished academic and role model for an entire generation of American vascular surgeons. Dr. William Smead with profound gratitude for supporting me to become clinical vascular fellow at the Ohio State University Hospital. Professor John Lumley (St Bartholomew’s Hospital, London) and Professor Brian Hopkinson (University Hospital, Nottingham). Heartfelt gratitude to my mentors, friends and teachers. George Geroulakos

Union Européenne des Médecins Spécialistes VASCULAR

S

U.E.

M.S.

SECTION AND BOARD OF VASCULAR SURGERY

President of the Section of Vascular Surgery: F. Benedetti-Valentini Secretary / Treasurer of the Section and Board of Vascular Surgery: M. Cairols

SURGERY

President of the Board of Vascular Surgery: K. Balzer Vice President of the Board of Vascular Surgery: A. Nevelsteen

Barcelona, March 2006 Vascular Surgery is a discipline that deals with one of the true plagues of the 20th century. Moreover, atherothrombosis will continue to be the main cause of death in the near future. New developments in the investigation, and endoluminal treatment of vascular disease have recently attracted significant publicity from the mass media and patient groups, and have significantly changed the management of the vascular patient. The provision of a high quality vascular service is closely linked with the need to give residents an appropriate training and to further introduce Vascular Surgery as an outstanding specialty. The book, “Vascular Surgery; Cases, Questions and Commentaries,” by Mr. Geroulakos, Prof Hero van Urk and Dr. R W Hobson II, will indeed contribute to a better understanding of Vascular Surgery as a specialty that deals with the pathology of arteries, veins and lymphatics. The experience and the teaching capabilities of the authors are unquestionable. This book, being so comprehensive, enhances the idea of considering Vascular Surgery as an independent entity from other specialties. Before achieving adequate competence to deal with the variety of cases shown in the book, the need for an appropriate training is obvious. Besides, the present text will help candidates to better prepare for the EBSQ-Vasc examination. The book utilises a time proven concept for teaching by questions and answers based on real problems, an essential part of CME. The book proposes learning following the Socratic method, by exercising our mind rather than reading told facts. On the other hand, it may improve our clinical practice and care of our vascular patients, as it incites Continuous Professional Development as a step forward in CME. The European Board of Vascular Surgery congratulates the authors for their initiative and gladly endorses the book. Marc Cairols Secretary General UEMS Section and Board of Vascular Surgery

vii

Foreword to the First Edition

This book is rather unique among textbooks in vascular surgery. Most cover the surgical management of vascular diseases, in whole or in part, in standard textbook fashion, with the text organized to cover the topics methodically in a didactic manner, and supported by tables, illustrations and references. Others have special purposes, such as atlases on technique or algorithm based books on decision-making. All have their place, but if the educational goals are training of the young surgeon, self-assessment and continuing medical education for the practitioner or preparation for oral examination, this book fills a special need, and fills it very well by breaking away from the didactic approach. It has long been recognized by educators that retention of knowledge, i.e. true learning, are much better achieved using the Socratic method of questions and answers, as opposed to simply reading or being told facts. In this book this approach is developed and presented in a very effective manner. In each “chapter,” one is presented with a case report representing a real life scenario. The case reports-scenarios in this book together cover most of vascular surgery experience. Following the case report, one is presented with questions and answers based on various aspects of the case, forcing the reader to commit to an answer. Whether the answer is right or wrong is not critical, in fact getting a wrong answer may be more beneficial in terms of correcting knowledge and retaining information. The commentary and conclusions that follow analyze the choice of answers, correct and incorrect, and discuss them in concise, authoritative detail, many of which are truly “pearls of information.” The conclusion then summarizes the current state of knowledge on the clinical issues under consideration. Numerous references are included. Together, these components constitute one of the most effective vehicles for self-education in vascular surgery today. Importantly, all aspects of management are covered: diagnostic evaluation and appropriate treatment, whether it is non-operative or interventional, endovascular or open surgery. To accomplish their goals the editors have gathered together a large number of experienced contributors, many well-known for their special areas of interest within vascular surgery, reflected in the contributions they make to this book. As such, the book should be useful to future and practicing vascular surgeons all over the world. It is full of statements covering most of the current state of knowledge in vascular surgery, and it does so in an entertaining and effective manner. Robert B. Rutherford

ix

Preface to the First Edition

This book is a unique collection of real life case histories written by experts that highlight the diversity of problems that may be encountered in vascular surgery. Each case scenario is interrupted by several questions that aim to engage the reader in the management of the patient and to give him the opportunity to test his knowledge. The comments reflect to as much as possible the principles of evidence based medicine and provide the answers to the questions. Several chapters are authored by individuals that contributed to the development of innovations in the management and prevention of vascular disease and are of interest for both the vascular trainee and the experienced vascular specialist. The goal of this book is to help vascular trainees review for Board and other examinations as well as to provide vascular surgeons who wish to expand or refresh their knowledge with an update and interactive source of information relevant to case scenarios that could be encountered in their practice. The European Boards in Vascular Surgery is a relatively new examination. Although the American Boards in Vascular Surgery were established many years earlier, there are no “dedicated” guides to cover the needs of these examinations. We hope that our book will provide a helpful hand that does not come from the standard text books, but directly from daily practice and therefore contains a high content of “how to do it” and “why we do it.” The references show the close relation between daily practice and “evidence based” practice, and we hope the two are not too different. We would like to thank all the authors who have contributed generously their knowledge and time to this project. George Geroulakos Hero van Urk Keith D. Calligaro Robert W. Hobson II

xi

Preface to the Second Edition

The author’s principal objective of the first edition was the presentation of the principles of vascular and endovascular surgery through interactive real life clinical scenarios. The success of the first edition has been gratifying. We have received many suggestions for additions and changes from vascular trainees, specialists and teachers at various institutions in Europe, USA and other parts of the world. These comments have been well received and have been important in improving and expanding the second edition. We wish to acknowledge our appreciation and gratitude to our authors and publishers. London Rotterdam New Jersey

George Geroulakos Hero van Urk Robert W. Hobson II

xiii

Preface to the Third Edition

The third edition updated most chapters that were focusing on the endovascular management of arterial and venous disease providing the reader with practical and updated, well referenced information on the full spectrum of options for the management of vascular disease. We are pleased to report the translation of the second edition of our book to Portuguese. We wish to express our thanks to our authors and publishers for their contribution to this project. London Yale, New Haven

George Geroulakos Bauer Sumpio

xv

Contents

Part I  Arterial Aneurysms   1 Preoperative Cardiac Risk Assessment and Management of Elderly Men with an Abdominal Aortic Aneurysm...................................... Don Poldermans and Jeroen J. Bax

3

  2 Abdominal Aortic Aneurysm............................................................................... 15 Daniel Danzer and Jean-Pierre Becquemin   3 Endoluminal Treatment of Infra-renal Abdominal Aortic Aneurysm............................................................................... 25 Frederico M.V. Bastos Gonçalves, Geoffrey H. White, Theodossios Perdikides, and Hence J.M. Verhagen   4  Ruptured Abdominal Aortic Aneurysm............................................................. 43 Jeffrey S.Weiss and Bauer E. Sumpio   5  Thoracoabdominal Aortic Aneurysm................................................................. 53 Hernan A. Bazan, Nicholas J. Morrissey, and Larry H. Hollier   6  Endovascular Management of Thoracic Aneurysm.......................................... 65 Reda Jamjoom, Nasser Alkhamees, and Cherrie Z. Abraham   7  Aortic Dissection................................................................................................... 75 Barbara Theresia Weis-Müller and Wilhelm Sandmann   8  Popliteal Artery Aneurysms................................................................................. 85 Susanna Shin and Michel Makaroun   9  Renal Artery Aneurysm....................................................................................... 91 Lutz Reiher, Tomas Pfeiffer, and Wilhelm Sandmann 10  Anastomotic aneurysms....................................................................................... 97 Jonothan J. Earnshaw

xvii

xviii

Contents

11  False Aneurysm in the Groin Following Coronary Angioplasty......................105 Steven S. Kang 12  Acute Thrombosis.................................................................................................113 Zachary M. Arthurs and Vikram S. Kashyap Part II  Acute Ischemia 13  Arterial Embolism................................................................................................127 Andre Nevelsteen 14  Blast Injury to the Lower Limb...........................................................................135 Paul H.B. Blair, Adrian K. Neil, and Christopher T. Andrews 15 Endovascular Management of Aortic Transection in a Multiinjured Patient......................................................................................145 Shiva Dindyal and Constantinos Kyriakides Part III  Management of Chronic Ischemia of the Lower Extremities 16  Cardiovascular Risk Factors and Peripheral Arterial Disease........................165 Stella S. Daskalopoulou and Dimitri P. Mikhailidis 17 Lower Limb Claudication due to Iliac Artery Occlusive Disease....................173 Marcus Brooks and Fabien Koskas 18 Lower Limb Claudication due to Bilateral Iliac Artery Occlusive Disease: The Case for Iliac Stenting and Femorofemoral Crossover Bypass...............................................................187 Jean-Baptiste Ricco and Olivier Page 19 Endovascular Management of Lower Limb Claudication due to Infra-Inguinal Disease...............................................................................199 Daniel J. Reddy and Mitchell R. Weaver 20  Endovascular Management of Non-Healing Leg Ulceration............................215 Jean Starr and Patrick Vaccaro 21  Bypass to the Popliteal Artery.............................................................................225 Keith D. Calligaro and Matthew J. Dougherty 22 Bypass to the Infrapopliteal Arteries for Chronic Critical Limb Ischemia....................................................................231 Enrico Ascher and Anil P. Hingorani

Contents

xix

23  Popliteal artery entrapment...............................................................................237 Luca di Marzo and Norman M. Rich 24  Adventitial Cystic Disease of the Popliteal Artery.............................................245 Bernard H. Nachbur and Jon Largiadèr 25  The Obturator Foramen Bypass..........................................................................255 Jørgen J. Jørgensen, Andries J. Kroese, and Lars E. Staxrud 26  Diabetic Foot..........................................................................................................265 Mauri J.A. Lepäntalo, Milla Kallio, and Anders Albäck Part IV  Surgery of the Major Branches of the Infradiaphragmatic Aorta 27  Chronic Visceral Ischemia...................................................................................277 George Geroulakos and William Smead 28  Acute Mesenteric Ischemia..................................................................................283 Jonathan S. Refson and John H.N. Wolfe 29  Renovascular Hypertension.................................................................................293 Constantina Chrysochou and Philip A. Kalra 30  Midaortic Syndrome.............................................................................................305 James C. Stanley and Jonathan L. Eliason Part V  Management of Portal Hypertension 31  Management of Portal Hypertension..................................................................319 Yolanda Y.L. Yang and J. Michael Henderson Part VI  Management of Extracranial Cerebrovascular Disease 32  Management of Carotid Bifurication Disease....................................................331 Wesley S. Moore 33  The Carotid Body Tumor.....................................................................................339 Mark-Paul F.M. Vrancken Peeters, Johanna M. Hendriks, Ellen V. Rouwet, Marc R.H.M. van Sambeek, Hero van Urk, and Hence J.M. Verhagen 34  Vertebrobasilar Ischemia: Embolic and Low-Flow Mechanisms.....................347 Ramon Berguer 35  Takayasu’s Arteritis Associated with Cerebrovascular Ischemia....................357 Duk-Kyung Kim and Young-Wook Kim

xx

Contents

Part VII  Neurovascular Conditions of the Upper Extremity 36  Neurogenic Thoracic Outlet Syndrome and Pectoralis Minor Syndrome.......373 Richard J. Sanders 37  Acute Axillary/Subclavian Vein Thrombosis.....................................................381 Torbjørn Dahl, Jarlis Wesche, and Hans O. Myhre 38  Raynaud’s Phenomenon.......................................................................................387 Ariane L. Herrick Part VIII  Prevention and Management of Complications of Arterial Surgery 39  Aortofemoral Graft Infection..............................................................................397 Christopher P. Gibbons 40  Aortoenteric Fistulas............................................................................................409 David Bergqvist Part IX  Vascular Access 41  The Optimal Conduit for Hemodialysis Access.................................................417 Frank T. Padberg and Robert W. Zickler 42 Acute Ischemia of the Upper Extremity Following Graft arteriovenous Fistula.................................................................................431 Miltos K. Lazarides and Vasilios D. Tzilalis Part X  Amputations 43  Amputations in an Ischemic Limb......................................................................441 Kenneth R. Ziegler and Bauer Sumpio Part XI  Vascular Malformations 44  Congenital Vascular Malformation.....................................................................457 Byung-Boong Lee 45  Klippel-Trenaunay Syndrome.............................................................................473 Magdiel Trinidad-Hernandez and Peter Gloviczki Part XII  Management of Venous Disorders 46  Deep Venous Thrombosis.....................................................................................483 Fahad S. Alasfar, Dwayne Badgett, and Anthony J. Comerota

Contents

xxi

47  Endoluminal Ablation of Varicose Veins............................................................491 Cassius Iyad N. Ochoa Chaar and Jeffrey Indes 48 Ultrasound Guided Foam Sclerotherapy for the Management of Recurrent Varicose Veins.................................................................................499 Christopher R. Lattimer and George Geroulakos 49  Venous Ulcers Associated with Deep Venous Insufficiency...............................507 Seshadri Raju 50  Venous Ulcers Associated with Superficial Venous Insufficiency.....................519 Guðmundur Daníelsson and Bo Eklöf 51  Iliofemoral Venous Thrombosis...........................................................................529 William P. Paaske 52  Iliofemoral Deep Venous Thrombosis During Pregnancy.................................535 Anthony J. Comerota Part XIII  Lymphodema 53  Management of Chronic Lyphedema of the Lower Extremity.........................549 Byung-Boong Lee and James Laredo 54 Management of Upper Extremity Lymphoedema with Microsurgical Lympho-Venous Anastomosis (LVA).................................567 Corradino Campisi and Francesco Boccardo Index .............................................................................................................................. 579

Contributors

Cherrie Z. Abraham, MD, FRCS Jewish General Hospital, Montreal QC, Canada Fahad S. Alasfar, MD Department of Surgery, Temple University Hospital, Philadelphia PA, USA Anders Albäck, MD Department of Vascular Surgery, Helsinki University Central Hospital, Helsinki, Finland Nasser Alkhamees, MD Department of Cardiac Surgery, McGill University, Montreal QC, Canada Christopher T. Andrews, MB, ChB, FRCS Department of Orthopaedic Surgery, Royal Victoria Hospital, Belfast, UK Zachary M. Arthurs, MD Department of Vascular Surgery, The Cleveland Clinic Foundation, Cleveland OH, USA Enrico Ascher, MD, FACS The Vascular Institute of New York, Brooklyn NY, USA

Dwayne Badgett, MD Department of Surgery, Temple University Hospital, Philadelphia PA, USA Frederico M.V. Bastos Gonçalves, MD Vascular Surgery Department, Santa Marta Hospital, CHLC, Lisbon, Portugal and Erasmus University Medical Center, Rotterdam, The Netherlands Jeroen J. Bax, MD, PhD Department of Cardiology, Leiden University Medical Center, Leiden, The Netherlands Hernan A. Bazan, MD Ochsner Clinic Foundation, Department of Surgery, Section of Vascular /Endovascular Surgery, New Orleans, LA, USA Jean-Pierre Becquemin, MD Department of Vascular and Endocrine Surgery, Henri-Mondor Hospital, Créteil, France David Bergqvist, MD, PhD, FRCS, FEBVS Department of Surgery, Uppsala University Hospital, Uppsala, Sweden

xxiii

xxiv

Ramon Berguer, MD, PhD Cadiovascular Center, The University of Michigan, Ann Arbor MI, USA Paul H.B. Blair, MD, FRCS Vascular Surgery Unit, Royal Victoria Hospital, Belfast, UK Francesco Boccardo, MD Professorial Unit of Medical Oncology, University and National Cancer Research Institute, Genoa, Italy Marcus Brooks, MA, MD, FRCS Department of Vascular Surgery, University Hospitals Bristol NHS Foundation Trust, Bristol, UK Marc A. Cairols, MD, PhD, FRCS Department of Vascular Surgery, University of Barcelona, Spain Keith D. Calligaro, MD Section of Vascular Surgery and Endovascular Therapy, Vascular Surgery Fellowship, Pennsylvania Hospital, Clinical Professor of Surgery, University of Pennsylvania School of Medicine, 700 Spruce St - Suite 101, Philadelphia, PA 19106 Corradino Campisi, MD, PhD Department of General Surgery, University Hospital, San Martino, Genoa, Italy Joseph M. Caruso, MD Division of Vascular Surgery, Department of Surgery, New Jersey Medical School,

Contributors

University of Medicine and Dentistry of New Jersey, Newark NJ, USA Jeannie K. Chang, MD Section of Vascular Surgery, University of Pennsylvania Health System, Pennsylvania Hospital, Philadelphia PA, USA Constantina Chrysochou, MRCP Department of Renal Medicine, Salford Royal Hospital and University of Manchester, Manchester, UK Anthony J. Comerota, MD, FACS, FACC Department of Surgery, Temple University Hospital, Philadelphia PA, USA Torbjørn Dahl, MD, PhD Department of Surgery, St. Olavs Hospital, University Hospital of Trondheim, Trondheim, Norway Guŏmundur Daníelsson, MD, PhD Department of Vascular Surgery, The National University Hospital of Iceland, Fossvogi, 108 Reykjavík, Iceland Daniel Danzer, MD Department of Vascular and Endocrine Surgery, Henri-Mondor Hospital, Créteil, France Stella S. Daskalopoulou, MD, MSc, DIC, PhD Department of Medicine, McGill University, Montreal QC, Canada

Contributors

Luca di Marzo, MD Department of Surgery P Valdoni, Sapienza University of Rome, Rome, Italy Shiva Dindyal, BSc, MB, BS, MRCS Department of General Surgery, The Royal London Hospital, London, UK Matthew J. Dougherty, MD Section of Vascular Surgery, University of Pennsylvania Health System, Pennsylvania Hospital, Philadelphia PA, USA Jonothan J. Earnshaw, DM, FRCS Department of Surgery, Gloucestershire Royal Hospital, Gloucester, UK Bo Eklöf, MD, PhD John A. Burns School of Medicine, University of Hawaii, Honolulu HI, USA University of Lund, Sweden Jonathan L. Eliason, MD Section of Vascular Surgery, Department of Surgery, University of Michigan Cardiovascular Centre, University of Michigan Medical School, Ann Arbor MI, USA George Geroulakos, MD, FRCS, FRCSE, DIC, PhD Imperial College of Science Technology and Medicine, Charing Cross Hospital, and Ealing Hospital, London, UK

xxv

Christopher P. Gibbons, MA, DPhil, MCh, FRCS Department of Vascular Surgery, Morriston Hospital, Swansea, UK Peter Gloviczki, MD Division of Vascular and Endovascular Surgery, Gonda Vascular Center, Mayo Clinic, Rochester MN, USA J. Michael Henderson, MD Division of Surgery, Cleveland Clinic Foundation, Cleveland OH, USA Joke M. Hendriks, MD Department of Vascular Surgery, Erasmus University Medical Center, Rotterdam, The Netherlands Ariane L. Herrick, MD, FRCP University of Manchester, Manchester Academic Health Science Centre, Salford Royal NHS Foundation Trust, Salford M6, 8HD, UK Anil P. Hingorani, MD The Vascular Institute of New York, Brooklyn NY, USA Larry H. Hollier, MD Louisiana State University Health Sciences Center, School of Medicine, New Orleans LA, USA Jeffrey Indes, MD Department of Vascular Surgery, Yale University School of Medicine, New Haven CT, USA Reda Jamjoom, MD, MEd, FRCSC McGill University, Montreal QC, Canada

xxvi

Contributors

Jørgen J. Jørgensen, MD, PhD Department of Vascular Surgery, Oslo University Hospital, Aker, Oslo, Norway

Andries J. Kroese, MD, PhD Department of Vascular Surgery, Oslo University Hospital, Aker, Oslo, Norway

Milla Kallio, MD Department of Vascular Surgery, Helsinki University Central Hospital, Helsinki, Finland

Constantinos Kyriakides, MB, ChB, MD, FRCS Department of General Surgery, The Royal London Hospital, London, UK

Philip A. Kalra, MD, FRCP Department of Renal Medicine, Salford Royal Hospital and University of Manchester, Manchester, UK Steven S. Kang, MD Department of Surgery, Florida International University School of Medicine, Miami FL, USA Vikram S. Kashyap, MD, FACS Department of Vascular Surgery, The Cleveland Clinic Foundation, Cleveland OH, USA Duk-Kyung Kim, MD, PhD Division of Cardiology, Department of Medicine, Samsung Medical Center, Sungkyunkwan University School of Medicine, Seoul, Korea Young-Wook Kim, MD, PhD Division of Vascular Surgery, Department of Surgery, Samsung Medical Center, Sungkyunkwan University School of Medicine, Seoul, Korea Fabien Koskas, MD, PhD Service of Vascular Surgery, Groupe Hospitalier Pitié-Salpétrière, Paris, France

James Laredo, MD, PhD, FACS Department of Vascular Surgery, Georgetown University School of Medicine, Washington DC, USA Jon Largiadèr, MD University Hospital of Zürich, Zürich, Switzerland Christopher R. Lattimer, MB, BS, FRCS, FdIT, MS Department of Vascular Surgery, Ealing Hospital NHS Trust, Middlesex, UK Miltos K. Lazarides, MD, EBSQvasc Department of Vascular Surgery, Demokritos University Hospital, Alexandroupolis, Greece Byung-Boong Lee, MD, PhD, FACS Department of Vascular Surgery, Georgetown University School of Medicine, Washington DC, USA Mauri J.A. Lepäntalo, MD, PhD Department of Vascular Surgery, Helsinki University Central Hospital, Helsinki, Finland Michel Makaroun, MD Division of Vascular Surgery, University of Pittsburgh Medical Center, Pittsburgh PA, USA

Contributors

xxvii

Dimitri P. Mikhailidis, BSc, MSc, MD, FCP, FFPM, FRCP Department of Clinical Biochemistry (Vascular Disease Prevention Clinics) Royal Free Hospital campus, University College London Medical School, University College London, London, UK Wesley S. Moore, MD Division of Vascular Surgery, UCLA, Los Angeles CA, USA Nicholas J. Morrissey, MD Columbia University, New York NY, USA Hans O. Myhre, MD, PhD Department of Surgery, St. Olavs Hospital, University Hospital of Trondheim, Trondheim, Norway Bernard H. Nachbur, MD University of Berne, Berne, Switzerland Adrian K. Neill, MRCS Department of Vascular Surgery, Royal Victoria Hospital, Belfast, UK Andre Nevelsteen , MD, PhD, FRCS Department of Vascular Surgery, University Hospital Gasthuisberg, Leuven, Belgium †

Aarhus University Hospital, Aarhus, Denmark Frank T. Padberg Jr, MD Division of Vascular Surgery, Department of Surgery, New Jersey Medical School, University of Medicine and Dentistry of New Jersey, Newark NJ, USA Olivier Page, MD Section of Vascular Surgery and Vascular Intervention, University of Poitiers Medical School, Poitiers, France Theodossios Perdikides, MD Vascular and Thoracic Surgery Department, Hellenic Air Force Hospital, Athens, Greece Tomas Pfeiffer, MD Klinik für Gefäßchirurgie und Nierentransplantation, Universitätsklinikum Düsseldorf Heinrich-Heine-Universität, Düsseldorf, Germany Don Poldermans, MD, PhD, FESC Department of Vascular Surgery, Erasmus MC, Rotterdam, The Netherlands

Cassius Iyad N. Ochoa Chaar, MD, MS General Surgery Department, Yale New Haven Hospital, New Haven CT, USA

Seshadri Raju, MBBS, MS Department of Surgery, University of Mississippi Medical Center, Flowood MS, USA

William P. Paaske, MD, FRCS, FRCSEd, FACS Department of Cardiothoracic and Vascular Surgery,

Daniel J. Reddy, MD Department of Surgery, Wayne State University, Detroit MI, USA

xxviii

Jonathan S. Refson, MBBS, MS, FRCS Department of Vascular Surgery, Princess Alexandra Hospital, Harlow, UK Lutz Reiher, MD Klinik für Gefäßchirurgie und Nierentransplantation, Universitätsklinikum Düsseldorf Heinrich-Heine-Universität, Düsseldorf, Germany Jean-Baptiste Ricco, MD, PhD Section of Vascular Surgery and Vascular Intervention, University of Poitiers Medical School, Poitiers, France

Contributors

William L. Smead, MD Department of Surgery, The Ohio State University, Columbus OH, USA James C. Stanley, MD Section of Vascular Surgery, Department of Surgery, University of Michigan Cardiovascular Centre, University of Michigan Medical School, Ann Arbor MI, USA Jean Starr, MD, FACS Division of Vascular Diseases and Surgery, The Ohio State University, Columbus OH, USA

Norman M. Rich Department of Surgery, F Edward Hébert School of Medicine, Uniformed Services University of the Health Sciences, Bethesda, MD, USA

Lars E. Staxrud, MD Department of Vascular Surgery, Oslo University Hospital, Aker, Oslo, Norway

Ellen V. Rouwet, MD Department of Vascular Surgery, Erasmus University Medical Center, Rotterdam, The Netherlands

Bauer E. Sumpio, MD, PhD, FACS Department of Vascular Surgery, Yale University School of Medicine, New Haven, CT, USA

Richard J. Sanders Department of Surgery, University of Colorado Health Science Center, Aurora CO, USA

Magdiel Trinidad-Hernandez, MD Division of Vascular and Surgery, Gonda Vascular Center, Mayo Clinic, Rochester MN, USA

Wilhelm Sandmann, MD Department of Vascular Surgery and Kidney Transplantation, University Clinic of Düsseldorf, Düsseldorf, Germany

Vasilios D. Tzilalis, MD Department of Vascular Surgery, General Military Hospital, Athens, Greece

Susanna Shin, MD Division of Vascular Surgery, University of Pittsburgh Medical Center, Pittsburgh PA, USA

Patrick Vaccaro, MD, FACS Division of Vascular Diseases and Surgery, The Ohio State University, Columbus OH, USA

Contributors

Marc R.H.M.van Sambeek, MD Department of Vascular Surgery, Erasmus University Medical Center, Rotterdam, The Netherlands Hero van Urk, MD Department of Vascular Surgery, Erasmus University Medical Center, Rotterdam, The Netherlands Hence J.M. Verhagen, MD, PhD Department of Vascular Surgery, Erasmus University Medical Center, Rotterdam, The Netherlands Mark-Paul F.M. Vrancken Peeters, MD Department of Vascular Surgery, Erasmus University Medical Center, Rotterdam, The Netherlands Mitchell R. Weaver, MD Henry Ford Medical Group, Henry Ford Hospital, Detroit MI, USA Jeffrey S. Weiss, MD Department of Vascular Surgery, Yale University School of Medicine, New Haven CT, USA Barbara Theresia Weis-Müller, MD Department of Vascular Surgery and Kidney Transplantation, University Clinic of Düsseldorf, Düsseldorf, Germany

xxix

Jarlis Wesche, MD, PhD Department of Surgery, Akershus University Hospital, University of Oslo, Lørenskog, Norway Geoffrey H. White, MD Endovascular Research Unit, Department of Surgery, University of Sydney, Sydney, Australia John H.N. Wolfe, MS, FRCS Regional Vascular Unit, St. Mary’s Hospital, London, UK Yolanda Y.L. Yang, MD, PhD Department of General Surgery, Cleveland Clinic Foundation, Cleveland OH, USA Robert W. Zickler, MD Division of Vascular Surgery, Department of Surgery, New Jersey Medical School, University of Medicine and Dentistry of New Jersey, Newark NJ, USA Kenneth R. Ziegler, MD Department of Vascular Surgery, Yale University School of Medicine, New Haven CT, USA

Part I Arterial Aneurysms

Preoperative Cardiac Risk Assessment andManagement of Elderly Men with anAbdominal Aortic Aneurysm

1

Don Poldermans and Jeroen J. Bax

A 72-year-old male presented with an abdominal aortic aneurysm. He had a history of chest pain complaints and underwent percutaneous transluminal coronary angioplasty (PTCA) 6 years ago. After the PTCA procedure he had no chest pain symptoms until 2 years ago. The chest pain complaints are stable and he was able to perform moderate exercise, such as a round of golf, in 4.5 h. Physical examination showed a friendly man, with blood pressure 160/70 mmHg and pulse 92 bpm. Examination of the chest revealed no abnormalities of the heart. Palpation of the abdomen showed an aortic aneurysm with an estimated diameter of 7 cm. The patient was referred to the vascular surgeon. Blood test showed an elevated fasting glucose of 10.0 mmol/l and low-density lipoprotein (LDL) cholesterol of 4.1 mmol/l. Electrocardiography showed a sinus rhythm and pathological Q-waves in leads V1–V3, suggestive of an old anterior infarction.

Question 1 Which of the following statements regarding postoperative outcome in patients undergoing major vascular surgery is correct? A.  Cardiac complications are the major cause of perioperative morbidity and mortality. B. Perioperative myocardial infarctions are related to fixed coronary artery stenosis in all patients. C. Perioperative cardiac events are related to a sudden, unpredictable progression of a nonsignificant coronary artery stenosis in all patients. D. Perioperative cardiac complications are related to both fixed and unstable coronary artery lesions. This patient experienced angina pectoris in the past. He was successfully treated with a PTCA procedure, but recently angina pectoris reoccurred. Because of the multiple risk factors and the planned high-risk surgery a dobutamine stress echocardiography was performed. Figure1.1 shows the normal stress protocol, with increasing doses of dobutamine and test

D. Poldermans () Department of Vascular Surgery, Erasmus MC, Rotterdam, The Netherlands G. Geroulakos and B. Sumpio (eds.), Vascular Surgery, DOI: 10.1007/978-1-84996-356-5_1, © Springer-Verlag London Limited 2011

3

4

D. Poldermans and J.J. Bax

endpoints. In Fig.1.2 the scoring of the left ventricle for wall motion abnormalities is shown. Figure1.3 is an example of a normal resting echocardiogram, showing respectively, apical views and one short-axis view. In Fig.1.4, the different stages of the stress test are shown for the apical four-chamber view: rest, low-dose dobutamine, peak dose dobutamine, and recovery. As indicated by arrows, the posterior septum shows an outward movement during peak stress, suggesting dyskinesia, and myocardial ischemia of the posterior septum. Dobutamine-atropine Stress Echocardiography 40 Atropine 2 mg 30 Target heart rate Side effects Ischemia

20 10

5

Echocardiography Heart rate/Blood pressure 0

5

10

13

16

19 minutes

Fig.1.1  The normal stress protocol, with increasing doses of dobutamine and test endpoints

6

6

1

7 10

LAD RCA

15

9

16 LAX

17 13 8

10 5

3

4CH

CX

SAX 17

11

8

12

14

7

9

1 = normal 2 = mild hypokinesia 3 = severe hypokinesia 4 = akinesia

2

4

5 = dyskinesia

2CH

Fig.1.2  The scoring of the left ventricle for wall motion abnormalities. LAX, long axis; SAX, short axis; 4CH, four.chambers; 2CH, two chambers; LAD, left anterior descending artery; RCA, right coronary artery; LCX, left circumflex artery

1  Preoperative Cardiac Risk Assessment andManagement of Elderly Men with anAbdominal Aortic Aneurysm

5

Fig.1.3  An example of a normal resting echocardiogram, showing respectively, apical views and one short-axis view

Fig.1.4  The different stages of the stress test of the apical four-chamber view, rest, low-dose dobutamine, peak dose dobutamine, and recovery. As shown and indicated with arrows, the posterior septum shows an outward movement during peak stress, suggestive of dyskinesia, and also myocardial ischemia of the posterior septum

6

D. Poldermans and J.J. Bax

Question 2 Postoperative outcome in patients undergoing major vascular surgery has been improved in those taking beta-blockers and statins. Medical therapy may reduce the need for additional preoperative testing for coronary artery disease as the incidence of perioperative cardiac mortality is reduced to less than 1%, and may even reduce the indications for preoperative coronary revascularization. A. Beta-blockers are associated with a reduced perioperative cardiac event rate in patients undergoing vascular surgery, both in retrospective and prospective studies. B. Statin use is associated with an improved postoperative outcome. C. Statin use is not associated with an increased incidence of perioperative myopathy. D. Beta-blockers and statins are independently associated with an improved postoperative outcome.

Question 3 Preoperative beta-blocker therapy is widely used. However, the dose and duration of preoperative therapy is uncertain. A. Beta-blockers should be started preferably 30 days prior to surgery. B. Beta-blockers should be initiated several hours before surgery. C. Heart rate control should be aimed at a heart rate between 90 and 100 bpm. D. Heart rate control should be aimed at a heart rate between 60 and 70 bpm. In this patient beta-blockers were started 6 weeks before surgery. Starting dose of bisoprolol was 2.5 mg; the dose was increased to 5.0 mg to obtain a resting heart between 60 and 70 bpm.

Question 4 Perioperative statin therapy has recently been introduced to improve postoperative outcome. A. Statins improve postoperative outcome by reducing the cholesterol level. B. Withdrawal of perioperative statin therapy is associated with an increased perioperative cardiac event rate. C. Perioperative statin use is associated with an increased incidence of myopathy. D. Perioperative statin use is associated with a reduced perioperative cardiac event rate in vascular surgery patients only. Statins were prescribed in this patient, Lescol (fluvastatin) XL 80 mg daily, at the same time as beta-blockers were introduced.

1  Preoperative Cardiac Risk Assessment andManagement of Elderly Men with anAbdominal Aortic Aneurysm

7

Question 5 Preoperative coronary revascularization seems to be an attractive option to improve not only direct postoperative outcome in high-risk patients but also long-term survival after surgery. A. Preoperative coronary revascularization improves postoperative outcome in all patients with significant coronary artery disease prior to major vascular surgery. B. Preoperative coronary revascularization in patients with one- or two-vessel disease is not associated with an improved postoperative outcome compared to patients receiving medical therapy. C. Preoperative coronary revascularization is associated with an improved 2-year outcome compared to medical therapy. D. Patients with proven coronary artery disease who are treated medically are at increased risk of late coronary revascularization after surgery. After late revascularization, longterm outcome is similar to that with revascularization prior to surgery. This 72-year-old male had multiple cardiac risk factors: elderly age, angina pectoris, diabetes mellitus, and a previous MI. He underwent a noninvasive stress test, dobutamine stress echocardiography, which showed myocardial ischemia, suggesting left anterior descending artery (LAD) disease. Beta-blockers and statins were prescribed and continued during surgery. Surgery was uneventful; after 2 years angina pectoris complaints increased and a PTCA procedure was successfully performed on the LAD.

1.1  Commentary Cardiac complications are the major cause of perioperative morbidity and mortality, which may occur in 1–5% of unselected patients undergoing major vascular surgery.1 [Q1: A] This high frequency of cardiac complications is related to the high prevalence of coronary artery disease; 54% of patients undergoing major vascular surgery have advanced or severe coronary artery disease and only 8% of patients have normal coronary arteries.2 Perioperative cardiac complications are equally caused by prolonged myocardial ischemia or by coronary artery plaque rupture with subsequent thrombus formation and coronary artery occlusion.1,3 [Q1: B, C, D] Prolonged perioperative myocardial ischemia usually occurs from either increased myocardial oxygen demand or reduced supply, or from a combination of the two. There are several perioperative factors that can increase myocardial oxygen demand including tachycardia and hypertension resulting from surgical stress, postoperative pain, interruption of beta-blocker use, or the use sympathomimetic drugs. Decreased oxygen supply, on the other hand, can occur as a result of hypotension, vasospasm, and anemia, hypoxia or coronary artery plaque rupture. Beta-blockers primarily reduce myocardial oxygen demand, while statins may prevent coronary artery plaque rupture. [Q2: A, B]

8

D. Poldermans and J.J. Bax

1.2  Beta-Adrenergic Antagonists Several retrospective and prospective clinical trials have shown that perioperative use of beta-blockers is associated with reduction in the incidence of postoperative myocardial ischemia, nonfatal myocardial infarction and cardiac death.4–6 [Q2: A] The majority of these studies were small in sample size, and the studies were designed to explore the protective effect of beta-blockers for the reduction of perioperative myocardial ischemia. To overcome the limitations of these studies two randomized clinical trials addressed the issue of perioperative use of beta-blockers for the prevention of cardiac death and myocardial infarction. Mangano etal.7 studied the effect of atenolol on mortality and cardiovascular morbidity after noncardiac surgery including vascular surgery. The investigators enrolled and randomized 200 patients to atenolol (given intravenously before and immediately after surgery and orally thereafter for the duration of hospitalization) or placebo. No difference was observed in 30-day mortality but mortality was significantly lower at 6 months following discharge (0% vs. 8%, p  60 mm B.  Association with an hypogastric aneurysm C.  Diabetic patient D.  Lower limb occlusive disease E.  Smoking F.  COPD

2  Abdominal Aortic Aneurysm

17

Question 3 With the imaging you have been provided with, is (are) there any reason(s) for performing an arteriogram A.  No need, CT-scan is sufficient B.  An angiogram is mandatory to facilitate the planning of the surgical procedure in case of difficult anatomy C.  Angiogram would be needed in case of endovascular treatment D. Angiography is necessary to rule out any asymptomatic associated visceral arterial stenosis

Question 4 To assess the operative cardiac risk would you need any further test in our patient. A.  None, ECG is sufficient. B.  Cardiac scintigraphy. C.  Cardiac echography. D.  Cardiac echography with Dobutamine test. E.  Coronary angiography.

Question 5 If an operation were being considered, which of the following factors are associated with an increased post-operative mortality? A.  Diameter > 60 mm B.  Association with an hypogastric aneurysm C.  Diabetic patient D.  Renal insufficiency E.  Smoking

Question 6 With the current information you got from the case report, what would you recommend to the patient (a) and which in case of a higher operative risk (b) A.  Duplex scan surveillance every 3 months B.  Aorto bifemoral through a midline incision C.  Aorto bifemoral graft through a left retroperitoneal incision D.  Aorto bi iliac graft through a left retroperitoneal incision E.  Stent-graft

18

D. Danzer and J.-P. Becquemin

The patient underwent, via a left retroperitoneal approach, an aorto-right and left common iliac bypass with end-to-end anastomosis. The aortic anastomosis was performed just at the level of the renal artery with a supra renal clamping of 10 min. This was justified by the necessity of suturing the prosthesis on the healthiest segment of aorta as possible. Therefore the retroperitoneal route gave a better access to the supra renal aorta. A cell saver was used and no heterogeneous blood had to be transfused. The patient’s postoperative course was uneventful, and he was discharged on the ninth post operative day.

Question 7 During open operation for AAA cell-saver autotransfusion (CSA) can be used. Which of the following is/are correct? A.  It should be used systematically. B.  It should be reserved for when the expected blood loss is significant. C. It should be substituted in all cases with preoperatively deposited autologous blood transfusion. D.  It presents fewer complications than unwashed cell autotransfusion. E.  It should not be used in case of ruptured aneurysm.

Question 8 Does a genetic predisposition to AAA exist? Describe the pathogenesis of AAA.

Question 9 A duplex scan has been performed to the patient’s brother which found a 40 mm abdominal aneurysm. What recommendation(s) would you give this patient’s brother? A. Serial duplex studies at 3-monthly intervals, and intervention when the diameter reaches 5.5 cm. B. Serial duplex studies at 6-monthly intervals, and intervention if the diameter reaches or exceeds 5 cm. C. Serial duplex studies at 12-monthly intervals until the diameter reaches 4.5 cm, then every 6 months until the diameter reaches 5 cm, then every 3 months, and then intervention when the aneurysm reaches 5.5 cm. D. Schedule the patient for surgery as he is a smoker and therefore his aneurysm will most likely require intervention.

2  Abdominal Aortic Aneurysm

19

2.1  Commentary The question of the optimal format for population screening and its cost effectiveness for AAA is still under debate. Many studies have attempted to identify high-risk populations in order to reduce healthcare costs and maximize the yield. Simon etal.1 have demonstrated a prevalence of AAA of 11% in male patients aged 60–75 years with a systolic blood pressure greater than 175 mmHg. No patient with uncomplicated hypertension had AAA. Claudication was the only cardiovascular complication associated independently with AAA (relative risk 5.8). Baxter etal. found a prevalence of 9% in patients older than 65 years old regardless of cardiovascular risk factors.2 Furthermore, preliminary results from the Aneurysm Detection and Management (ADAM) study revealed that smoking was the most important risk factor associated with AAA (odds ratio [OR] 5.57), followed by a positive family history (OR 1.95), age, height, coronary artery disease, atherosclerosis, high cholesterol level and hypertension.3 Similar results were found in the later Multicentre Aneurysm Screening Study (MASS) demonstrating that screening in male patients older than 65 years old would be cost effective.4 Therefore, most vascular surgeons agree that all men over the age of 65 years and women who did smoke5 should systematically be offered an abdominal ultrasound, the screening should be done at 55years if indicated family history.6 [Q1: B, C, D] Natural history of aneurysms and risk of rupture are better understood with the results of the UK small aneurysms trial7 and the ADAM trial. As in former cohort studies of patients who refused early operation8 or who were considered to be inoperable, risk of rupture increased with size, and intervention seems justified over 5.5 cm, in patients with sufficient life expectancy. Growth is recognized as related to tobacco use but diabetes mellitus and female gender are protective. Controversial opinion regarding other risk factors persist as recent data suggests no influence of hypertension, statin use and ACE on aneurysm growth as published in former studies.9 Rupture is strongly correlated with persistent tobacco use, female gender, aneurysm size, diminution of FEV1, HTA and presence of transplant. [Q2: A, E, F] Pre-operative planning is of outmost importance in order to avoid intra-operative unexpected findings, shortening of the surgery and/or evaluate the possibility of endovascular treatment. Nowadays, the CT scanner with 3D reconstruction, the gold standard, and invasive conventional angiography, is only needed for treatment of subsequent visceral significant and symptomatic stenosis. Albeit relatively frequently in patients requiring AAA surgery, visceral arterial stenosis10–12 should be treated separately if needed and via endovascular means when possible. One stage surgery with visceral reconstruction increases the operative difficulty and consequently the operative risk.13 Actual data shows better assessment of vessel morphology with CT reconstruction than angiography for EVAR14 but is also useful in open surgery to evaluate the vessels morphology and planning of surgery in case of any anatomical anomaly (e.g, horseshoe kidney). [Q3: A]

20

D. Danzer and J.-P. Becquemin

Concerning a pre-operative work-out; routine coronary angiography in vascular patients has shown that 60% of them have severe coronary artery disease.15 However a large randomized study in patients with stable angina have clearly demonstrated that pre-operative coronary bypass or angioplasty do not improve the post-operative and 5 year survival rate.16 Beta blockers, statins and antiplatelets have all contributed to the reduction of cardiac events following major vascular surgery. Thus pre-operative investigation can be restricted to patients with poor functional capacity and at least three identified predictive factors of severe coronary artery disease.17 In the current case diabetes, hypertension and mild renal insufficiency are three of these markers and pre-operative cardiac screening would have been indicated if the patient hadn’t shown a good functional capacity.18 [Q4: A] When mandatory cardiac echography with dobutamine probably is the most reliable test.19 And pre-operative coronary revascularization is only indicated for those patients with acute ST elevation MI, unstable angina, or stable angina with left main coronary artery or threevessel disease, as well as those patients with two-vessel disease that includes the proximal left anterior descending artery, and either ischemia on non-invasive testing or an ejection fraction of less than 0.50. Analysis of predictive factor of mortality in patients submitted to open repair of AAA have shown that age, cardiac status, renal insufficiency and pulmonary status were strongly predictive of post operative complications and deaths. Difficult operations are also associated with an increased operative risk mostly related to the increase of blood loss. Unilateral or bilateral hypogastric aneurysm increased the operative risk.20 [Q5: B, D] In this case surveillance was not recommended due to the aneurysm size and the relatively young age of the patient. Open surgery via a trans abdominal or retroperitoneal approach is a wise option in case of low operative risk and difficult anatomy as in our case where the infrarenal neck was not suitable for a regular endovascular graft implantation. We choose a retroperitonal approach because of the better exposure of the aorta at the level of the visceral arteries and his obesity. A retroperitoneal approach is an appealing way especially in case of obese patient or the need for preparation of the aorta at the level or upper the renal arteries. Nevertheless the distal right iliac axis remains the Achilles heel’s of this approach which would have required a second contra lateral incision for reconstruction of the right external iliac axis if needed. In our case the aneurysm involved only the proximal right common iliac artery and the right iliac anastomosis could be achieved with a slight enlargement of the retroperitoneal route toward the midline. Femoral anastomosis is not recommended because of the increased infection rate after a groin incision. [Q6a: D] Although a retroperitoneal approach provides a better access to the suprarenal aorta, the former advocated superiority of the retroperitonal route in terms of pain. Bowel and respiratory function was never supported by randomized trials especially in the era of peri­ operative peridural analgesia. No actual data support the systematic use of trans versus retro peritoneal approach in terms of post operative outcome, therefore the choice should be based on the anatomical features and surgeon preference. Less invasive with a lower operative mortality (1.5% vs. 4.6% for Open Repair21), a shorter in-hospital stay and recovery time, EVAR could have been considered if the

21

2  Abdominal Aortic Aneurysm

patient had a suitable aortic neck, major comorbidities or hostile abdomen. Although the two major early randomized trials (EVAR 1 and DREAM) failed to show sustained benefit of the post-operative mortality at 2 years, no death in the EVAR group was aneurysm related,22,23 and a former survey showed an incidence of ongoing aneurysm related mortality after EVAR of 1% per year.24 A large retrospective case match cohort study including more than 40,000 participants did not show inferiority of the long term results of EVAR compared to Open Repair and the rate of secondary procedure in the EVAR group was largely overwhelmed by the rate of wound hernia after OR. Subsequently secondary procedure frequency seems to decrease after the first year following EVAR.25 Therefore EVAR is considered by many teams as the first option in case of adequate anatomy. Usual recommendation for endovascular aneurysm treatment requires a proximal neck length under the renal artery of 15 mm, a limited angulation of the aorta (30 mm. B.  Infrarenal neck length 47 min, lack of autotransfusion devices, bifurcated grafts, and technical complications (i.e., left renal vein injury).11–13 Postoperative risk factors include renal failure, coagulopathy, and cardiac complications. Hardman etal.10 found that possession of three or more preoperative risk factors correlated with 100% mortality. Currently, no recommendation exists to withhold surgery for patients with any or all of these risk factors; this decision is made on a case-by-case basis, making risk factor analysis useful mostly from the standpoint of guiding patient decisions on surgery and family discussions on prognosis. Patients who present with symptoms of a rAAA can be divided into two groups based on whether or not they have a known AAA (Fig.4.3).14 Unstable patients with known AAAs present the least diagnostic challenge as they belong in the operating room. In contrast, the unstable patient without known AAA can be the hardest to evaluate. If an rAAA is suspected, this patient needs to be assessed expeditiously with an ECG as myocardial infarction can often mimic these symptoms. If cardiogenic shock is clinically apparent, resuscitation should override emergent surgery; however, cardiac ischemia secondary to hypovolemic shock from a rupture needs both rapid resuscitation and emergent surgery as the underlying cause of shock is the rupture and not the heart. Patients without hemodynamic instability allow the examiner the time to proceed with radiological confirmation.15 [Q3: A] Ultrasound is fast and convenient as it allows an examination while resuscitation is taking place at the bedside. The sensitivity is as high as 100% for detecting an AAA, but it is inaccurate on diagnosing rupture (49%).12,16 This study is ideal on hemodynamically stable patients without known AAA, minimal operative risk factors, and symptoms or signs suggestive of rupture. [Q4: B, C] In this case, the mere presence of an AAA would warrant surgery without delay. CT scans are more difficult to obtain and place the patient at some increased risk because of time delay and interruption of resuscitation. They are clearly only indicated for patients who are stable and offer the advantage of being able to diagnosis rupture. The groups of patients most likely to benefit from CT scan are those with significant comorbidities where delay could allow preoperative optimization.17 The sensitivity and specificity of CT scan for diagnosing rupture is quoted to be as high as 94% and 95%, respectively.15 Once the decision to operate has been made, several preoperative measures should be undertaken. A natural instinct is to bolus intravenous (IV) fluid in an attempt to normalize the blood pressure; this should be avoided. Instead, adopting a permissive hypotensive strategy will allow the patient´s own physiologic response to minimize blood loss.18 Although there are times when fluids are necessary, this strategy can be effective in preventing accelerated blood loss until the aorta is clamped or occluded. Every effort should be made to keep the patient warm with blankets, raising the operating room temperature, and utilizing warmed IV fluids and blood products.8 The patient should be prepared and draped before induction as the loss of sympathetic tone with anesthesia may cause a marginally compensated patient to collapse.

48

J.S. Weiss and B.E. Sumpio

Fig.4.3  Algorithm for suspected ruptured abdominal aortic aneurysm (rAAA)

A midline laparotomy provides the quickest route of entry and best exposure in most cases. A low threshold to obtain supraceliac control will prevent inadvertent venous injury, especially in cases with large retroperitoneal hematomas. This control is obtained by incising the gastrohepatic ligament and diaphragmatic crura, and then bluntly dissecting the periaortic tissue; a preoperative nasogastric tube can aid in identification of the laterally positioned esophagus. A clamp or manual pressure is applied to the supraceliac aorta. The transverse colon is reflected cephalad and the small bowel eviscerated. The supraceliac control can then be moved to the infrarenal neck after it is carefully dissected out. Systemic heparinization is avoided and heparinized saline (10 units/mL) is used locally down both iliacs before balloon occlusion. The use of intraoperative blood recuperation and autotransfusion devices is crucial in minimizing postoperative mortality by limiting hom*ologous blood transfusions.13 The use of a tube graft, typically knitted Dacron or PTFE

4  Ruptured Abdominal Aortic Aneurysm

49

polytetrafluoroethylene, will shorten operative times and restore flow sooner than a bifurcated graft; this may necessitate leaving aneurysmal iliac arteries alone.14 [Q5: B, C, D] After completion of grafting, bowel and lower extremity perfusion are assessed, usually by inspection and Doppler probe. The aneurysm sac is closed around the graft in an attempt to prevent later aortoenteric fistulas. Depending on the size of retroperitoneal hematoma and degree of resuscitation, the abdomen may not close easily. In these cases, it is best to perform a temporary closure with plans to return to the operating room for washout and definitive closure at a later, more stable time. The dismal mortality following open repair of rAAA and the expansion of endovascular techniques has prompted recent exploration into application of stent grafts for primary therapy. Patient candidacy for an endovascular repair of AAA (EVAR) is the first hurdle when considering this approach. Measurements to determine this are typically done by CT angiography, although the Montefiore group have been successful utilizing digital subtraction angiography in two views.19 The concern of sending a potentially unstable patient with known or suspected ruptured AAA to the CT scanner was recently addressed by Lloyd etal.20 from Leicester; they found that 87.5% of patients survived longer than 2 h after admission, with 92% of these patients having systolic blood pressures greater than 80 mm Hg. Ruptured or symptomatic AAAs are found to have larger infrarenal neck diameters and smaller neck lengths.21 Despite these morphological differences, several reports have found amazingly high feasibility rates for EVAR, ranging from 46% to 80%.22,23 Dimensional requirements for endografts are constantly shifting as new devices improve the field, but currently an infrarenal neck = 10 mm and a diameter = 30 mm are needed.24 [Q6: A, B, D] The next hurdle is availability of an endograft team and the graft itself. The importance of a knowledgeable and experienced team cannot be overstated as any program without this is destined for failure. A variety of grafts are being utilized, with favor towards a modular aorto-uniiliac device; this set-up decreases the need for large inventories.23,24 The Montefiore group have developed an aorto-unifemoral graft which they use in conjunction with a crossover femoral-femoral graft.19 Surprisingly few patients are rejected for EVAR secondary to unfavorable hemodynamics. Supraceliac balloon occlusion via a brachial or femoral route under fluoroscopic guidance can allow proximal aortic control under local anesthesia; a technique being utilized by some for control prior to laparotomy in open cases.25 Prospective randomized studies are underway to examine the morbidity and mortality rates of EVAR with respect to open repair, but preliminary nonrandomized results are already favoring this approach.19,24,26 The most common complication of rAAA repair is renal failure, followed by ileus, sepsis, myocardial infarction, respiratory failure, bleeding, and bowel ischemia.1,11 [Q7: E] Postoperative renal failure has been found by several authors to correlate with mortality.1,11 Minimizing suprarenal clamp time and use of mannitol before cross-clamping the aorta to initiate brisk diuresis may limit renal damage. The inflammatory mediators and cytokines released from the shock state, visceral hypoperfusion, and massive transfusions associated with open repair can lead to multi-organ system failure; the avoidance of supraceliac clamping and lower blood loss are some of the potential advantages of the EVAR approach. But EVAR has its own unique complications which include endoleaks, graft malfunction, and groin wound issues.

50

J.S. Weiss and B.E. Sumpio

References   1. Noel AA, Gloviczki P, Cherry KJ, etal. Ruptured abdominal aortic aneurysms: the excessive mortality rate of conventional repair. J Vasc Surg. 2001;34:41-46.   2. Dardik A, Burleyson GP, Bowman H, Gordon TA, Webb TH, Perler BA. Surgical repair of ruptured abdominal aortic aneurysms in the state of Maryland: factors influencing outcome among 527 recent cases. J Vasc Surg. 1998;28:413-421.   3. Heller JA, Weinberg A, Arons R, etal. Two decades of abdominal aortic repair: have we made any progress? J Vasc Surg. 2000;32:1091-1100.   4. Bengtsson H, Bergqvist D. Ruptured abdominal aortic aneurysm: a population-based study. J Vasc Surg. 1993;18:74-80.   5. Rose J, Civil I, Koelmeyer T, Haydock D, Adams D. Ruptured abdominal aortic aneurysms: clinical presentation in Auckland 1993–1997. Aust NZ J Surg. 2001;71:341-344.   6. Wakefield TW, Whitehouse WM, Wu SC, etal. Abdominal aortic aneurysm rupture: statistical analysis of factors affecting outcome of surgical treatment. Surgery. 1982;91:586-596.   7. Sasaki S, Yasuda K, Yamauchi H, Shiya N, Sakuma M. Determinants of the postoperative and long-term survival of patients with ruptured abdominal aortic aneurysms. Surg Today. 1998;28:30-35.   8. Piper G, Patel N, Chandela S, etal. Short-term predictors and long-term outcome after ruptured abdominal aortic aneurysm repair. Am Surg. 2003;63:703-710.   9. Shackelton CR, Schechter MT, Bianco R, Hildebrand HD. Preoperative predictors of mortality risk in ruptured abdominal aortic aneurysm. J Vasc Surg. 1987;6:583-589. 10. Hardman DT, Fisher CM, Patel MI, etal. Ruptured abdominal aortic aneurysms: who should be offered surgery? J Vasc Surg. 1996;23:123-129. 11. Donaldson MC, Rosenberg JM, Bucknam CA. Factors affecting survival after ruptured abdominal aortic aneurysm. J Vasc Surg. 1985;2:564-570. 12. Markovic M, Davidovic L, Maksimovic Z, etal. Ruptured abdominal aortic aneurysm predictors of survival in 229 consecutive surgical patients. Herz. 2004;29:123-129. 13. Marty-Ane CH, Alric P, Picot MC, Picard E, Colson P, Mary H. Ruptured abdominal aortic aneurysm: influence of intraoperative management on surgical outcome. J Vasc Surg. 1995;22:780-786. 14. Hallett JW, Rasmussen TE. Ruptured abdominal aortic aneurysm. In: Cronenwett JL, Rutherford RB, eds. Decision Making in Vascular Surgery. Philadelphia: Saunders; 2001: 104-107. 15. Kvilekval KH, Best IM, Mason RA, Newton GB, Giron F. The value of computed tomography in the management of symptomatic abdominal aortic aneurysms. J Vasc Surg. 1990;12: 28-33. 16. Tayal VS, Graf CD, Gibbs MA. Prospective study of accuracy and outcome of emergency ultrasound for abdominal aortic aneurysm over two years. Acad Emerg Med. 2003;10: 867-871. 17. Sullivan CA, Rohrer MJ, Cutler BS. Clinical management of the symptomatic but unruptured abdominal aortic aneurysm. J Vasc Surg. 1990;11:799-803. 18. Owens TM, Watson WC, Prough DS, Uchida T, Kramer GC. Limiting initial resuscitation of uncontrolled hemorrhage reduces internal bleeding and subsequent volume requirements. J Trauma. 1995;39:200-209. 19. Veith FJ, Ohki T, Lipsitz EC, Suggs WD, Cynamon J. Treatment of ruptured abdominal aneurysms with stent grafts: a new gold standard? Semin Vasc Surg. 2003;16:171-175. 20. Lloyd GM, Bown MJ, Norwood MG, etal. Feasibility of preoperative computer tomography in patients with ruptured abdominal aortic aneurysm: a time-to-death study in patients without operation. J Vasc Surg. 2004;39:788-791.

4  Ruptured Abdominal Aortic Aneurysm

51

21. Lee WA, Huber TS, Hirneise CM, Berceli SA, Seeger JM. Eligibility rates of ruptured and symptomatic AAA for endovascular repair. J Endovasc Ther. 2002;9:436-442. 22. Reichart M, Geelkerken RH, Huisman AB, van Det RJ, de Smit P, Volker EP. Ruptured abdominal aortic aneurysm: endovascular repair is feasible in 40% of patients. Eur J Vasc Endovasc Surg. 2003;26:479-486. 23. Hinchliffe RJ, Braithwaite BD, Hopkinson BR. The endovascular management of ruptured abdominal aortic aneurysms. Eur J Vasc Endovasc Surg. 2003;25:191-201. 24. Peppelenbosch N, Yilmaz N, van Marrewijk C, etal. Emergency treatment of acute symptomatic or ruptured abdominal aortic aneurysm. Outcome of a prospective intent-to-treat by EVAR protocol. Eur J Vasc Endovasc Surg. 2003;26:303-310. 25. Matsuda H, Tanaka Y, Hino Y, etal. Transbrachial arterial insertion of aortic occlusion balloon catheter in patients with shock from ruptured abdominal aortic aneurysm. J Vasc Surg. 2003;38:1293-1296. 26. Lee WA, Huber TS, Hirneise CM, Berceli SA, Seeger JM. Impact of endovascular repair on early outcomes of ruptured abdominal aortic aneurysms. J Vasc Surg. 2004;40:211-215.

Thoracoabdominal Aortic Aneurysm

5

Hernan A. Bazan, Nicholas J. Morrissey, and Larry H. Hollier

A 72-year-old white male presented to his primary-care physician with a history of left chest pain for the past month. The pain was dull and constant and radiated to the back, medial to the scapula. He denied a new cough or worsening shortness of breath. He had no recent weight loss, and his appetite was good. He had a history of hypertension, which was currently controlled medically, and a significant 60 pack-a-year smoking history. In addition, he suffered a myocardial infarction (MI) 5years ago. The patient denied any history of claudication, transient ischaemic attacks or stroke. He had undergone surgery in the past for bilateral inguinal hernias, and underwent cardiac catheterization after his MI. On physical examination, the patient was thin but did not appear malnourished. Vital signs were heart rate 72 beats/min, blood pressure 140/80mmHg, respiratory rate 18/min, and temperature 36.8°C. His head and neck examination was remarkable for bilateral carotid bruits. Cardiac examination revealed a regular rate and rhythm without murmurs. Abdominal examination revealed no bruits and a palpable aortic mass. His femoral and popliteal pulses were normal (2+); Posterior tibial pulses were 1+ bilaterally, and dorsalis pedis signals were detectable only by Doppler. No prominent popliteal pulses were appreciated. Routine blood work was unremarkable, and an electrocardiogram (ECG) revealed changes consistent with an old inferior wall MI and left ventricular (LV) hypertrophy. Chest X-ray (Fig. 5.1) was remarkable for a tortuous aorta, which had calcification within the wall and appeared dilated. There were no pleural effusions, but both hemidiaphragms did demonstrate some flattening, and bony structures were normal. Lung fields were clear of masses or consolidation.

H.A. Bazan () Ochsner Clinic Foundation, Department of Surgery, Section of Vascular/Endovascular Surgery, New Orleans, LA 70121, USA e-mail: [emailprotected] G. Geroulakos and B. Sumpio (eds.), Vascular Surgery, DOI: 10.1007/978-1-84996-356-5_5, © Springer-Verlag London Limited 2011

53

54

H.A. Bazan et al.

Fig.5.1  Chest X-ray demonstrating a tortuous and dilated descending thoracic aorta suggestive of a thoracoabdominal aortic aneurysm

Question 1 Which of the following is the single most likely diagnosis causing this man’s pain? A.  Acute MI B.  Acute aortic dissection C.  Thoracic aortic aneurysm D.  Lung cancer E.  Pneumonia

Question 2 Which of the following studies should be performed in this patient in order to plan therapy? A.  Aortography B.  Computed tomography (CT) scan of chest C.  Carotid duplex studies D.  Cardiac stress test E.  Arterial blood gas (ABG) analysis Although aortography was routinely done before, CT scan of the chest and abdomen was obtained (Fig.5.2) and deemed sufficient for operative planning. Findings were consistent with a thoracoabdominal aneurysm without concomitant dissection of the aorta. There was

5  Thoracoabdominal Aortic Aneurysm

55

Fig.5.2  CTA scan demonstrating aneurysmal dilatation of the descending thoracic aorta

no ­evidence for acute leak or rupture, and the maximal diameter of the thoracic aorta was7.3cm.

Question 3 In the Crawford classification system for thoracoabdominal aortic aneurysms (TAAAs), which represents the most extensive TAAA? A.  Type I B.  Type II C.  Type III D.  Type IV The patient underwent a cardiac stress test, which was normal. Carotid duplex studies revealed minimal atherosclerotic disease with bilateral stenoses of less than 50%. ABG analysis showed pH 7.38, pCO2 42 and pO276 on room air.

Question 4 Which of the following management schemes seems most reasonable for this patient? A.  Observation with annual follow-up chest CT B.  Repair of thoracoabdominal aneurysm after bilateral carotid endarterectomies C.  Cardiac catheterization followed by repair of TAAA D.  Elective repair of TAAA The patient is scheduled for elective repair of his TAAA. He expresses concern about the possibility of complications from the surgery. You explain to him the most likely complications related to this surgery.

56

H.A. Bazan et al.

Question 5 Of the following, which is not a common complication following TAAA repair? A.  Pulmonary B.  Cardiac C.  Renal D.  Gastrointestinal The patient seems most concerned about the risk of postoperative paralysis. You explain to him that there are things you can do to decrease his risk of suffering these complications, although nothing can eliminate the risk.

Question 6 Which of the following technical modifications is not believed to be beneficial in the prevention of spinal cord dysfunction following TAAA repair? A.  Tumor necrosis factor-a monoclonal antibody B.  Cerebrospinal fluid drainage C.  Reimplantation of key intercostal arteries D.  Epidural cooling The patient undergoes repair of TAAA and tolerates the procedure well. Postoperatively, the chest tubes are draining 100–150cm3 blood/h for the first 3h. In addition, urine output is steady at 500cm3/h. The patient has transient drops in blood pressure to a systolic blood pressure in the 70s, with central venous pressure dropping to 5mmHg.

Question 7 (a) Outline the initial work-up and potential correction of the bleeding problem described above in order to prevent a return to the operating room. (b) What fluid resuscitation approach should be taken to stabilize this patient’s hemodynamic status? The patient’s temperature is 34.6°C, international normalized ration (INR) is 1.7 and partial thromboplastin time (PTT) is 50s (control, 34s). Platelet count is 33,000. After infusion of warm fluids, the use of a warming blanket, and platelet and fresh frozen plasma (FFP) transfusions, the parameters return to normal and the drainage from the chest tubes decreases to about 10–20cm3/h. On the second postoperative day, the patient is noted to have loss of motor function in his lower extremities.

Question 8 What therapeutic intervention, if carried out in a timely fashion, may restore this patient’s neurological function partially or fully?

5  Thoracoabdominal Aortic Aneurysm

57

Following appropriate intervention, the patient’s neurological function returns to normal. The patient’s recovery is otherwise uneventful, and he is discharged on postoperative day 8 with clean incisions, intact neurological status and adequate analgesia.

Question 9 Following a successful recovery from his surgery, this gentleman’s approximate predicted 5-year survival is: A.  20% B.  50% C.  70% D.  90%

Question 10 Is there a role for endovascular or hybrid repair of this TAAA?

5.1  Commentary TAAAs are less common than infrarenal abdominal aortic aneurysms. One populationbased study suggested an incidence of 5.9 TAAAs per 100,000 person-years.1 Although TAAAs are more common in males, the male:female ratio of 1.1–2.1:1 is not as weighted as the ratio of abdominal aortic aneurysm (AAA). The aetiology of TAAAs is related to atherosclerotic medial degenerative disease (82%) and aortic dissection (17%) in most cases.2 About 45% of TAAAs are asymptomatic and detected during work-up of other systems, usually on chest X-ray or cardiac echocardiography examinations. Patients with TAAAs tend to be older than AAA patients and, therefore, may have more severe comorbidities. When present, symptoms are usually chest or back pain related to compression of adjacent structures by the aneurysm or cough from compression/erosion of airways. Fistulization is rare but erosion into the bronchial tree presents with massive haemoptysis, while erosion into the esophagus presents with upper-gastrointestinal bleeding. Presentation with acute, severe pain may reflect leak, acute expansion or dissection of the aneurysm and require urgent evaluation and treatment. The risk factors associated with TAAA are smoking, hypertension, coronary artery disease, chronic obstructive pulmonary disease (COPD), and disease in other vascular beds. Syphilitic aneurysms are a rare cause of TAAA in this era but, when present, usually involve the ascending aorta. Other causes of vague chest and back pain in a patient such as this include myocardial ischemia, pulmonary neoplasm, acute dissection, pneumonia, and bony metastases. [Q1: C]

58

H.A. Bazan et al.

The clinical and X-ray findings in this particular case argue against these other possibilities. The work-up of patients with TAAA requires assessment of the aneurysm extent, size, and condition of the remaining aorta. Before any studies are carried out, a thorough history and physical examination, including vascular assessment, are needed. [Q2: B] With marked improvements in non-invasive imaging in the past decade, computed tomography angiography (CTA) has replaced invasive aortography for defining the extent of TAAA, status of aortic branches, delineation of any associated dissection, and presence of leak. Magnetic resonance imaging (MRI) and magnetic resonance angiography (MRA) have also improved dramatically and offer unique benefits over CT, such as lack of radiation and non-nephrotoxic contrast agents. However, MRA has yet to achieve the resolution of CTA and its use is contraindicated in unstable patients. Transesophageal echocardiography can assess the status of the aortic valve as well as cardiac function. Significant aortic insufficiency is a contraindication to thoracic aortic cross-clamping, unless a shunt or pump is used to bypass the left heart. The Crawford classification [Q3: B] is used to characterise TAAAs.3 According to this system, aneurysms beginning just distal to the left subclavian artery and involving the aorta up to, but not below, the renals are termed type I. Type II TAAAs are the most extensive – they begin just beyond the left subclavian and continue into the infrarenal aorta. Type III aneurysms involve the distal half of the thoracic aorta, usually originating at the level of T6, and varying extents of the abdominal aorta. Type IV TAAAs refer to those aneurysms involving the entire abdominal aorta, up to the diaphragm and including the visceral segments. This classification scheme has been useful for predicting morbidity and mortality following repair of TAAAs. In addition to assessment of the aneurysm, the high incidence of comorbidities in this patient population mandates thorough evaluation of cardiac, as well as pulmonary reserve. Preoperative studies should include electrocardiography and cardiac stress testing. Further work-up is dictated by the presence of positive findings. Screening chest X-ray and preoperative ABG provides information regarding the patient’s pulmonary status. Formal pulmonary function tests should be reserved for those patients with evidence of significant pulmonary compromise. Since the risk factors for TAAA are the same as those for atherosclerotic disease, a careful history and physical will dictate whether there is a need to work up disease in other vascular beds (carotid, mesenteric, renal, lower extremity). Carotid duplex studies may be done routinely preoperatively and significant carotid stenoses are treated before TAAA repair. The status of the patient’s clotting system must be determined and optimized, if necessary. In the absence of indications to carry out other operations first, this patient with a TAAA of >6cm should undergo elective repair of his aneurysm. [Q4: D] Observation with follow-up imaging studies is dangerous and puts the patient at risk of death due to aneurysm rupture. The natural history of TAAAs is related to size and growth rate. Understanding the behaviour of these lesions is of crucial importance when determining treatment. Crawford’s series of 94 TAAAs followed for 25years demonstrated 2-year survival of 24%, with about half of deaths due to rupture.4 This series included dissected as well as non-dissected aneurysms. A more recent series of non-dissected TAAAs revealed rupture rates of 12% at 2years and 32% at 4years; for aneurysms greater than 5cm in diameter, rupture rates increased to 18% at 2years.5 Rupture is uncommon in aneurysms measuring less than 5cm in diameter. Another risk factor for rupture

5  Thoracoabdominal Aortic Aneurysm

59

seems to be an increased expansion rate, with aneurysms growing more than 5 mm in 6months at higher risk than those growing more slowly. Survival in non-operated patients was 52% at 2years and 17% at 5years. Patients who underwent repair of TAAA had a 5-year survival of 50%. Another series revealed 61% 5-year survival following TAAA repair. Survival decreased to 50% for patients with dissecting TAAA.6 Operative repair is usually through a left thoracotomy with a paramedian abdominal extension, depending on the distal extent of the aneurysm. A retroperitoneal approach to the abdominal segment is used. The distal extent of the aneurysm determines which intercostal space will be used for a thoracotomy. The incision is made in the fourth or fifth intercostal space for type I or high type II TAAAs, while an incision in the seventh, eighth or ninth intercostal spaces is appropriate for types III or IV.7 Careful identification and reimplantation of visceral vessels is important, as is re-attachment of intercostal arteries when feasible. Successful repair of TAAA results from careful yet quick technique, as well as maintenance of optimal physiology by the anaesthesia and surgical teams. Distal aortic perfusion is accomplished either with left heart bypass and selective visceral perfusion or an axillary-femoral artery bypass before thoracotomy. Distal aortic perfusion manoeuvres are important for the prevention of major systemic morbidity following TAAA repair. Patients undergoing TAAA repair frequently are older and have significant cardiac, pulmonary and other vascular comorbidities. These factors, combined with the magnitude of the operation and extent of aortic replacement, can lead to significant rates of mortality and serious morbidity. [Q5: D] Pulmonary complications remain the most common and result from a combination of preoperative tobacco use, chronic obstructive pulmonary disease (COPD), and the effect of the thoracoabdominal incision on postoperative pulmonary mechanics. Reperfusion injury may also lead to pulmonary microvascular injury and subsequent pulmonary dysfunction.8 Cardiac complications remain the next most common, in spite of preoperative cardiac optimisation. Avoidance of hypotension, close monitoring perioperatively with pulmonary artery catheters, and minimisation of strain on the left ventricle can help decrease postoperative cardiac dysfunction. Using the bypass circuit to control ventricular afterload can reduce the risk of cardiac complications.9 Renal insufficiency preoperatively increases the risk of postoperative renal failure and mortality. Minimising ischaemic time, selective renal perfusion during cross-clamping, distal aortic perfusion techniques, and avoidance of hypovolaemia are important in preventing renal failure.10 Perhaps the most devastating complication following TAAA repair is paraplegia. Despite years of research and development of protective strategies, paraplegia rates following TAAA repair remain between 5% and 30%, with an average of 13%.6 Risk factors for postoperative paraplegia include extent of aneurysm (and therefore most common in Type II TAAAs), cross-clamp time, postoperative hypotension, previous abdominal aortic reconstruction, and oversewing of intercostal arteries. Cross-clamp times of less than 30min are generally safe, while those in the range of 30–60min are associated with increasing risk; cross-clamp times of more than 60 min carry the highest risk for neurological complications. Minimizing cross-clamp time and avoiding hypotension will decrease the risk of paraplegia. Sequential reperfusion of intercostal vessels by moving the cross clamp caudally as segments are reimplanted is useful to re-establish flow to these vessels quickly. In addition, avoiding prolonged mesenteric ischemia, which may worsen reperfusion injury to the lungs, heart and possibly spinal cord through release of cytotoxic cytokines, is beneficial.

60

H.A. Bazan et al.

Numerous adjuncts have been studied for their ability to prevent paraplegia. [Q6: A] The use of cerebrospinal fluid (CSF) drainage to keep CSF pressure at less than 10mmHg has been shown to decrease the incidence of postoperative paraplegia, when combined with distal aortic perfusion and/or moderate hypothermia.11 Reimplantation of intercostal vessels, particularly in the important segment of T8–T12, is most likely beneficial in preventing postoperative paraplegia, provided this manoeuvre does not excessively prolong clamp time.12 Epidural cooling by continuous infusion of cool saline via catheter has been reported to decrease the incidence of paraplegia following TAAA repair in a high-volume centre.13 Preoperative angiographic localisation of the artery of Adamkiewicz followed by successful reimplantation of this vessel during surgery has resulted in no neurological sequelae in another Centre’s series.14 Patients who did not have preoperative localisation, or in whom reimplantation was unsuccessful, had a 50% paraplegia rate. These results have not been reproduced, and angiographic localisation has not gained widespread acceptance. General anesthetic agents can also help to prevent paraplegia, with propofol being the most protective. When left heart bypass is performed using pump techniques, moderate hypothermia can be used to protect the spinal cord. Other pharmacological adjuncts that may be beneficial include steroids and mannitol. Free-radical scavengers and inhibitors of excitatory neurotransmitter pathways have shown benefit experimentally but have not been proven clinically.15 At present, the best strategy for preventing spinal cord complications appears to involve a combination of physiological optimization of the patient perioperatively, avoidance of intra-operative hypotension, intraoperative use of spinal drainage and some form of distal aortic perfusion, reimplantation of patent intercostal vessels, and minimisation of cross-clamp time. Other protective adjuncts are used based on surgeon preference and experience. Repair of a TAAA represents a major physiological insult. Excellent anesthesia care and post-operative critical care monitoring are essential components of a successful operation. Postoperatively, large volumes of urine output must be replaced on a 1:1 basis in order to avoid hypovolaemia. Use of warmed, balanced electrolyte solutions is preferred. [Q7] Coagulopathy in the postoperative period is usually related to incomplete replacement of clotting factors and hypothermia. In addition, supracoeliac aortic clamping has been shown to result in a state of fibrinolysis that may exacerbate bleeding.17 The aneurysm itself can be responsible for chronic coagulation factor consumption and a subsequent increased tendency to perioperative coagulopathy.18 Ongoing bleeding after TAAA repair may require reoperation, and results in an increase in major morbidity and mortality. It is important to ensure that any increased prothrombin and partial thromboplastin times are corrected with plasma transfusions. Platelets should be replaced if thrombocytopenia occurs in the face of ongoing bleeding. Since hypothermia is often used intraoperatively as a spinal cord protective measure, it may persist as a problem postoperatively. Aggressive correction with warm fluids, blood products and warming blankets is needed to restore normothermia and proper function of coagulation as well as other enzymatic systems. Reoperation is reserved for ongoing significant bleeding following correction of coagulopathy and hypothermia. Reoperation for bleeding results in mortality rates of 25% or greater in these patients.19 Some patients, as in the case we present here, will awake neurologically intact only to develop paraplegia hours to days later. [Q8] This phenomenon of delayed-onset paraplegia

5  Thoracoabdominal Aortic Aneurysm

61

may represent reperfusion injury to areas of the spinal cord at risk from intraoperative hypoperfusion. Avoidance of postoperative hypoperfusion may decrease the incidence of this complication. The epidural catheter is left in place for 3days postoperatively. In cases of delayed-onset paraplegia, maintenance of CSF pressure below 10mmHg may permit restoration of function. There are anecdotal reports of reversal of delayed-onset paraplegia by placement of an epidural catheter after onset of paralysis and removal of CSF to decrease pressure to below 10mmHg.16 Lowering the CSF pressure may increase cord perfusion pressure enough to rescue the threatened regions of neuronal tissue. Lowering the CSF pressure to below 5mmHg may cause intracerebral haemorrhage, therefore the pressure must be monitored closely and maintained in the safe range. [Q9: B] Patients undergoing successful TAAA repair have a 5-year survival of 50–61%5, 6 [also please refer to discussion following Q4]. [Q10] Since the patient presented did not have any significant contraindications to an open thoracoabdominal repair, endovascular repair of his TAAA would not have been appropriate at this time. However, various institutional studies have demonstrated the feasibility and safety of endovascular repair of TAAAs for patients at significant risk for open repair.20 Pre-operative planning with high-resolution, thin-cut CTA is mandatory. Fenestrated endografts may be used for treatment of juxta-renal aortic aneurysms or more extensive type IV and other TAAAs. Fenestrations are circular openings in the aortic graft fabric that are circumferentially reinforced with a nitinol ring, which is ultimately matted with a balloon-expandable stent graft into the target visceral vessel. Branched endografts are aortic endografts with side branches pre-sewn to the graft fabric; these in combination with fenestrations, help treat even the most complex TAAAs. A recent French series of 33 patients undergoing treatment of TAAAs with fenestrated and branched endografts for a variety of TAAAs types (type I [3%], II [21%], III [37%], IV [13%]) demonstrated an inhospital mortality of 9%.21 Type II and III endoleaks were present in 15% of patients and transient spinal cord ischemia occurred in 12% of patients, though permanent paraplegia remained in only 3%. A review of six single-institution series encompassing 496 patients with TAAAs, demonstrated a 30-day mortality of fewer than 9%, spinal cord ischemia of 2.7–20%, and remarkably high branch patency rates (96–100%). As potential loss of visceral branch vessels is a feared complication of fenestrated and branched endografts repair, mid- and long-term results with larger patient populations will be important to determine whether material fatigue and fracture, migration, and/or component separation occur. Aside from fenestrated or branched endograft repair, early follow-up demonstrated no renal of visceral branch vessel occlusion; all 109 vessels were patent in this high-volume centre single centre study. Recently, Lachat etal. have introduced a novel hybrid open and endovascular approach that may be particularly useful for the treatment of Type IV TAAAs.22 This technique involves placement of a self-expanding stent graft (Viabahn grafts, Gore and Associates, Flagstaff, Az) thru a retrograde Seldinger technique into the origin of the renal or visceral vessel. Using this Viabahn Open Rebranching TEChnique (VORTEC) technique, the distal end of the self-expanding stent graft is deployed in the visceral or renal vessel and partially projects outside the vessel. The proximal end of the graft is then anastomosed to the debranching graft, which may originate from a common iliac artery; the proximal stump of the visceral vessel is ligated to avoid retrograde perfusion of the aneurysm and

62

H.A. Bazan et al.

endoleak after subsequent endovascular aneurysm repair. VORTEC may be particularly useful in re-do operations, where entire dissection of the visceral vessel is not necessary. This novel hybrid technique remains a single institution experience and more broad experience is necessary to establish reproducibility and safety.

References   1. Bickerstaff LK, Pairolero PC, Hollier LH, etal. Thoracic aortic aneurysms: a population based study. Surgery. 1982;92:1103-1108.   2. Panneton JM, Hollier LH. Nondissecting thoracoabdominal aortic aneurysms: part I. Ann Vasc Surg. 1995;9:503.   3. Crawford ES, Crawford JL, Safi HJ, etal. Thoracoabdominal aortic aneurysms: preoperative and intraoperative factors determining immediate and long term results of operations in 605 patients. J Vasc Surg. 1986;3:389-404.   4. Crawford ES, DeNatale RW. Thoracoabdominal aortic aneurysm: observations regarding the natural course of the disease. J Vasc Surg. 1986;3:578-582.   5. Cambria RA, Gloviczki P, Stanson AW, etal. Outcome and expansion rate of 57 thoracoabdominal aortic aneurysms managed nonoperatively. Am J Surg. 1995;170:213-217.   6. Panneton JM, Hollier LH. Dissecting descending thoracic and thoracoabdominal aortic aneurysms: Part II. Ann Vasc Surg. 1995;9:596-605.   7. Hollier LH. Technical modifications in the repair of thoracoabdominal aortic aneurysms. In: Greenlagh RM, ed. Vascular Surgical Techniques. London: W.B. Saunders; 1989:144-151.   8. Paterson IS, Klausner JM, Goldman G, et al. Pulmonary edema after aneurysm surgery is modified by mannitol. Ann Surg. 1989;210:796-801.   9. Hug HR, Taber RE. Bypass flow requirements during thoracic aneurysmectomy with parti­ cular attention to the prevention of left heart failure. J Thorac Cardiovasc Surg. 1969;57: 203-213. 10. Kazui T, Komatsu S, Yokoyama H. Surgical treatment of aneurysms of the thoracic aorta with the aid of partial cardiopulmonary bypass: an analysis of 95 patients. Ann Thorac Surg. 1987;43:622-627. 11. Safi HJ, Miller CC 3rd, Huynh TT, etal. Distal aortic perfusion and cerebrospinal fluid drainage for thoracoabdominal and descending thoracic aortic repair: ten years of organ protection. Ann Surg. 2003;238:372-380. 12. Safi HJ, Estrera AL, Azizzadeh A, Coogan S, Miller CC 3rd. Progress and future challenges in thoracoabdominal aortic aneurysm management. World J Surg. 2008;32:355-360. 13. Black JH, Davison JK, Cambria RP. Regional hypothermia with epidural cooling for prevention of spinal cord ischemic complications after thoracoabdominal aortic surgery. Semin Thorac Cardiovasc Surg. 2003;15:345-352. 14. Webb TH, Williams GM. Thoracoabdominal aneurysm repair. Cardiovasc Surg. 1999;7:573-585. 15. Wisselink W, Money SR, Crockett DE, etal. Ischemia-reperfusion of the spinal cord: protective effect of the hydroxyl radical scavenger dimethylthiourea. J Vasc Surg. 1994;20:444-450. 16. Hollier LH, Money SR, Naslund TC, etal. Risk of spinal cord dysfunction in patients undergoing thoracoabdominal aortic replacement. Am J Surg. 1992;164:210-214. 17. Gertler JP, Cambria RP, Brewster DC, etal. Coagulation changes during thoracoabdominal aneurysm repair. J Vasc Surg. 1996;24:936-945. 18. Fisher DF, Yawn DH, Crawford ES. Preoperative disseminated intravascular coagulation caused by abdominal aortic aneurysm. J Vasc Surg. 1986;4:184-186.

5  Thoracoabdominal Aortic Aneurysm

63

19. Svensson LG, Crawford ES, Hess KR, Coselli JS, Safi HJ. Experience with 1509 patients undergoing thoracoabdominal aortic operations. J Vasc Surg. 1993;17:357-370. 20. D’Elia P, Tyrrell M, Sobocinski J, Azzaoui R, Koussa M, Haulon S. Endovascular thoracoabdominal aortic aneurysm repair: a literature review of early and mid-term results. J Cardiovasc Surg. 2009;50:439-445. 21. Haulon S, D’Elia P, O’Brien N, etal. Endovascular repair of thoracoabdominal aortic aneurysms. Eur J Vasc Endovasc Surg. 2009;50(4):475-481. 22. Donas KP, Lachat M, Rancic Z, etal. Early and midterm outcome of a novel technique to simplify the hybrid procedures in the treatment of thoracoabdominal and pararenal aortic aneurysms. J Vasc Surg. 2009;50:1280-1284.

Endovascular Management of Thoracic Aneurysm

6

Reda Jamjoom, Nasser Alkhamees, and Cherrie Z. Abraham

A 75-year-old male has been referred to your service after a contrast–enhanced spiral computed tomography (CT) performed for investigation of chronic cough revealed an incidental finding of a 7.3cm thoracic aortic aneurysm (TAA). Past medical history includes moderate chronic obstructive pulmonary disease (COPD), hypertension, insulin-dependent diabetes and a history of coronary artery catheterization and stenting 5 years ago. The patient denies current angina symptoms. On examination, vital signs are stable, cardio-respiratory examination is within normal limits, and arterial examination reveals no carotid bruits, normal heart sounds without murmurs, no palpable abdominal masses and all upper and lower limb distal pulses are palpable. His routine blood work is within normal range.

Question 1 What is your next investigation? A.  Ankle brachial index (ABI) B. Contrast-enhanced computed tomography angiography (CTA) of chest, abdomen and pelvis with 3D reconstruction C.  Duplex ultrasound of the abdomen D.  Cardiac stress test CTA was obtained (Fig.6.1). It demonstrates a 7.3cm saccular thoracic aortic aneurysm, beginning 3cm distal to the subclavian artery. External iliac artery diameters are 8mm on the right and 9 mm on the left. Due to the patient’s age and medical comorbidities, ­endovascular repair was the sole treatment option offered to the patient, who subsequently consented to the procedure.

R. Jamjoom () McGill University, Montreal, QC, Canada G. Geroulakos and B. Sumpio (eds.), Vascular Surgery, DOI: 10.1007/978-1-84996-356-5_6, © Springer-Verlag London Limited 2011

65

66

R. Jamjoom et al.

Fig.6.1  CTA of chest demonstrating 7.3 cm TAA in size

Question 2 What are the contraindications for standard TEVAR? A.  Diseased (3mm/year.3,4 However, whereas OSR is appropriate only for relatively physiologically fit patients, TEVAR has the advantage of being able to treat less fit patients who might otherwise be turned down for open repair. Most surgeons treating this pathology favour TEVAR as their first option given the fact that the chest cavity does not need to be opened thus avoiding the common pulmonary complications that are associated with OSR. Perhaps the most important advantage of TEVAR is that the thoracic aorta does not need to be cross-clamped. This can obviously lead to deleterious consequences in any patient with cardiac insufficiency or valvular abnormalities. Relative contraindications for standard TEVAR include inadequate proximal and distal landing zone ( 80 mmHg), avoiding high blood pressure (SBP > 160) to minimize the chance of stent migration as well as hypertensive medical complications such as stroke. Urine output should be recorded, and regular neurological assessment should be carried out to assess for stroke and spinal cord ischemia. Patients with cerebrospinal fluid (CSF) drains should have continuous CSF pressure monitoring and CSF should be drained according to a standardized protocol.22,23 [Q7] [Q8: A, B, C, D] Case number 2 describes a situation in which there is inadequate length of healthy aorta distal to the left carotid artery for an adequate seal. A hybrid procedure consisting of extra-anatomic bypass (right to left carotid-carotid bypass with or without a left carotidsubclavian bypass) and TEVAR was chosen as the treatment option.24 Recently, technological innovation has demonstrated the possibility of circumventing debranching Extent A

Extent B

Extent C

(L. subclavian a. to T6)

(T6 to diaphragm)

(L. subclavian a. to diaphragm)

Fig.6.5  Extent of aortic coverage during TEVAR

6  Endovascular Management of Thoracic Aneurysm

73

procedures with fenestrated/scalloped and branched arch grafts. However, further cases and published case series are necessary before these procedures become ready for prime time. [Q10: A, B] If extra-anatomic bypass is being considered for aortic arch debranching, carotid duplex ultrasound is mandatory to assess for occlusive disease as well as vertebral artery flow dynamics. Carotid endarterectomy may need to be performed in conjunction with the bypass. [Q11: C] In some instances, it is acceptable to cover the origin of the subclavian artery with the thoracic aortic stent graft without subclavian revascularization, however carotid-subclavian bypass should be considered in patients who have radiological evidence of a dominant left vertebral artery and in patients who demonstrated the following: aberrant origin of the left vertebral artery, history of CABG using the left internal mammary artery, history of AAA repair, occluded or diseased hypogastric arteries, patent left axillary-femoral bypass graft, functional left arm arteriovenous fistula, and in any patients who require long segment coverage with TEVAR (B, C extent).16,25 [Q12: A. B, C] The authors prefer staging the extra-anatomic debranching procedures when possible. Advantages to this method include minimizing operative time, and identifying the etiology of potential neurological complications after each procedure. [Q13]

References   1. Parodi JC, Palmaz JC, Barone HD. Transfemoral intraluminal graft implantation for abdominal aortic aneurysms. Ann Vasc Surg. (Nov) 1991;5(6):491-499.   2. Dake MD, Miller DC, Semba CP, Mitchell RS, Walker PJ, Liddell RP. Transluminal placement of endovascular stent-grafts for the treatment of descending thoracic aortic aneurysms. N Engl J Med. (Dec 29) 1994;331(26):1729-1734.   3. Katzen BT, Dake MD, MacLean AA, Wang DS. Endovascular repair of abdominal and thoracic aortic aneurysms. Circulation. (Sept 13) 2005;112(11):1663-1675.   4. Cambria RP, Crawford RS, Cho JS, etal. A multicenter clinical trial of endovascular stent graft repair of acute catastrophes of the descending thoracic aorta. J Vasc Surg. (Dec) 2009;50(6):1255-1264. e1251-1254.   5. Jones LE. Endovascular stent grafting of thoracic aortic aneurysms: technological advancements provide an alternative to traditional surgical repair. J Cardiovasc Nurs. (Nov–Dec) 2005;20(6):376-384.   6. Patel HJ, Williams DM, Upchurch GR Jr, et al. A comparison of open and endovascular descending thoracic aortic repair in patients older than 75 years of age. Ann Thorac Surg. (May) 2008;85((5):1597-1603. discussion 1603-1594.   7. Ueda T, Fleischmann D, Rubin GD, Dake MD, Sze DY. Imaging of the thoracic aorta before and after stent-graft repair of aneurysms and dissections. Semin Thorac Cardiovasc Surg. (Winter) 2008;20(4):348-357.   8. Bernard EO, Schmid ER, Lachat ML, Germann RC. Nitroglycerin to control blood pressure during endovascular stent-grafting of descending thoracic aortic aneurysms. J Vasc Surg. (Apr) 2000;31(4):790-793.   9. Dorros G, Cohn JM. Adenosine-induced transient cardiac asystole enhances precise deployment of stent-grafts in the thoracic or abdominal aorta. J Endovasc Surg. (Aug) 1996;3(3):270-272.

74

R. Jamjoom et al.

10. Kahn RA, Moskowitz DM, Marin ML, et al. Safety and efficacy of high-dose adenosineinduced asystole during endovascular AAA repair. J Endovasc Ther. (Aug) 2000;7(4):292-296. 11. p*rnratanarangsi S, Webster MW, Alison P, Nand P. Rapid ventricular pacing to lower blood pressure during endograft deployment in the thoracic aorta. Ann Thorac Surg. (May) 2006;81(5):e21-23. 12. David F, Sanchez A, Yanez L, et al. Cardiac pacing in balloon aortic valvuloplasty. Int J Cardiol. (Apr 4) 2007;116(3):327-330. 13. Webb JG, Pasupati S, Achtem L, Thompson CR. Rapid pacing to facilitate transcatheter prosthetic heart valve implantation. Catheter Cardiovasc Interv. (Aug) 2006;68(2):199-204. 14. Criado FJ. Iliac arterial conduits for endovascular access: technical considerations. J Endovasc Ther. (Jun) 2007;14(3):347-351. 15. Criado FJ, Barnatan MF, Rizk Y, Clark NS, Wang CF. Technical strategies to expand stentgraft applicability in the aortic arch and proximal descending thoracic aorta. J Endovasc Ther. 2002;9(Suppl 2):II32-38. 16. Gutsche JT, Szeto W, Cheung AT. Endovascular stenting of thoracic aortic aneurysm. Anesthesiol Clin. (Sept) 2008;26(3):481-499. 17. Buth J, Harris PL, Hobo R, et al. Neurologic complications associated with endovascular repair of thoracic aortic pathology: incidence and risk factors. A study from the European Collaborators on Stent/Graft Techniques for Aortic Aneurysm Repair (EUROSTAR) registry. J Vasc Surg. (Dec) 2007;46(6):1103-1110. discussion 1110-1101. 18. Chiesa R, Melissano G, Marrocco-Trischitta MM, Civilini E, Setacci F. Spinal cord ischemia after elective stent-graft repair of the thoracic aorta. J Vasc Surg. (Jul) 2005;42(1):11-17. 19. Kawaharada N, Morish*ta K, Kurimoto Y, etal. Spinal cord ischemia after elective endovascular stent-graft repair of the thoracic aorta. Eur J Cardiothorac Surg. (Jun) 2007;31(6): 998-1003. discussion 1003. 20. Gutsche JT, Cheung AT, McGarvey ML, etal. Risk factors for perioperative stroke after thoracic endovascular aortic repair. Ann Thorac Surg. (Oct) 2007;84(4)):1195-1200. discussion 1200. 21. Feezor RJ, Martin TD, Hess PJ Jr, etal. Extent of aortic coverage and incidence of spinal cord ischemia after thoracic endovascular aneurysm repair. Ann Thorac Surg. (Dec) 2008;86(6): 1809-1814. discussion 1814. 22. Estrera AL, Miller CC 3rd, Chen EP, et al. Descending thoracic aortic aneurysm repair: 12-year experience using distal aortic perfusion and cerebrospinal fluid drainage. Ann Thorac Surg. (Oct) 2005;80(4):1290-1296. discussion 1296. 23. Hnath JC, Mehta M, Taggert JB, etal. Strategies to improve spinal cord ischemia in endovascular thoracic aortic repair: outcomes of a prospective cerebrospinal fluid drainage protocol. JVasc Surg. (Oct) 2008;48(4):836-840. 24. Cina CS, Safar HA, Lagana A, Arena G, Clase CM. Subclavian carotid transposition and bypass grafting: consecutive cohort study and systematic review. J Vasc Surg. (Mar) 2002;35(3):422-429. 25. Feezor RJ, Lee WA. Management of the left subclavian artery during TEVAR. Semin Vasc Surg. (Sept) 2009;22(3):159-164.

Aortic Dissection

7

Barbara Theresia Weis-Müller and Wilhelm Sandmann

7.1  Dissection: Stanford A A 68-year-old woman spontaneously and suddenly developed severe retrosternal pain during her holiday in Turkey. Without knowing the diagnosis, she flew home 2 days later. Computed tomography (CT) scans taken immediately after arrival revealed a dissection of the ascending aorta, the aortic bow and the descending aorta.

Question 1 How would you classify the aortic dissection? A.  Stanford A dissection. B.  Stanford B dissection. C.  de Bakey I dissection. D.  de Bakey II dissection. E.  de Bakey III dissection On the same day, she underwent an emergency operation. The dissected ascending aorta with the entry of dissection was incised in a cardiopulmonary bypass and replaced by a graft using the in-graft technique. The aortic valve was patent and remained in situ. For reconstruction of the aortic root, the sandwich technique was used. Two Teflon strips were placed externally and into the true lumen to reattach the dissected membrane to the aortic wall. The aortic graft was then sutured into the reconstructed aortic root.

B.T. Weis-Müller () Department of Vascular Surgery and Kidney Transplantation, University Clinic of Düsseldorf, Düsseldorf, Germany G. Geroulakos and B. Sumpio (eds.), Vascular Surgery, DOI: 10.1007/978-1-84996-356-5_7, © Springer-Verlag London Limited 2011

75

76

B.T. Weis-Müller and W. Sandmann

Question 2 Which of the following statements are wrong? A.  Stanford A dissections should be treated medically. B.  Stanford A dissections should undergo operation immediately. C.  Stanford B dissections without ischemic complications should be treated medically. D.  Stanford B dissections require operative intervention immediately. E.  Stanford A dissections require an aortic stent graft immediately. The postoperative course was uneventful at the beginning. However, 3 days later, renal function deteriorated and the patient required haemofiltration. Moreover, the patient developed severe hypertension and had to be treated with three different antihypertensive drugs. Contrast CT scans revealed that the right kidney was without function due to an old hydronephrosis, while the left renal artery was probably dissected. Furthermore, the patient developed left leg ischemia and was transferred to our centre. We explored the abdomen via the transperitoneal approach. The pulsation of the left iliac artery was weak due to aortic and left iliac dissection. Infrarenal aorto-iliac membrane resection was performed to restore the blood flow to the extremities. Then the left renal artery was explored; the renal artery dissection was found to extend towards the hilus of the kidney. Revascularisation was achieved with a saphenous vein interposition graft placed between the left iliac artery and the distal left renal artery (Fig.7.1).

Question 3 Which of the following statements are correct? A.  Complications of Stanford A dissection are aortic valve insufficiency and perforation into the pericardium. B.  Stroke is a typical complication of Stanford B dissection. C.  Paraplegia is a typical complication of aortic dissection. D.  Most patients with Stanford B dissections die of aortic perforation. E.  Typical complications of aortic dissection are organ and lower-extremity ischaemia. The postoperative course was uneventful. The patient recovered promptly from the operative intervention, while renal function and blood pressure improved substantially. Urine production and laboratory findings became normal, and only one antihypertensive drug (a beta-blocker) was necessary to maintain normal blood pressure. The postope­ rative angiography showed a patent iliac-renal interposition graft and normal perfusion of the left kidney (Fig. 7.1). CT scans taken 2 years later displayed a hypertrophic, ­well-functioning left kidney, while the right kidney was small and hydronephrotic (Fig.7.2).

7  Aortic Dissection

Fig.7.1  (a) Left common iliac artery. (b) Left renal artery saphenous vein bypass

Fig.7.2  Computed tomography (CT) scans taken 18 months after operative intervention show a well-functioning, hypertrophic left kidney and a small, hydronephrotic right kidney. Note the dissected but non-dilated aorta

77

78

B.T. Weis-Müller and W. Sandmann

7.2  Dissection: Stanford B A 54-year-old woman was admitted to another hospital with the provisional diagnosis of a myocardial infarction (MI). She experienced a sudden chest pain. Some hours later, she developed paraesthesia in both legs, which improved spontaneously. Subsequently, she felt abdominal discomfort and developed diarrhoea and vomiting. The patient had been normotensive throughout her life, but now she required five different antihypertensive drugs to stabilise blood pressure. Some laboratory data were abnormal, including leucocytes, transaminases, lactic dehydrogenase and lactate. Duplex sonography and transoesophageal echocardiography revealed an aortic dissection of the thoracic and abdominal aorta beginning distal to the left subclavian artery; blood flow into the visceral arteries and the right renal artery was reduced. Contrast CT scans confirmed Stanford B aortic dissection.

Question 4 What diagnostic methods are involved in acute aortic dissection? A.  Computed tomography. B.  Magnet resonance imaging. C.  Angiography. D.  Transoesophageal echocardiography. The patient was first treated medically with parenteral therapy and antihypertensive drugs (including beta-blockers). Under this management, clinical outcome and laboratory findings improved, but 3 weeks later the patient deteriorated again and developed severe right upper abdominal pain. She was referred to our hospital for operation. CT scans displayed the aortic dissection and a dissected superior mesenteric artery. The true aortic lumen was very small and partially thrombosed (Fig. 7.3). Abdominal exploration via the transperitoneal approach revealed borderline ischaemia of all intra-abdominal organs due to aortic dissection. The dissection had affected the coeliac trunk, the superior mesenteric artery and the right renal artery. The right upper abdominal pain was caused by an ischaemic cholecystitis. The gallbladder had to be removed. The para-aortic tissue displayed severe inflammation; therefore no fenestration and membrane resection could be carried out. Instead, intestinal and renal blood flow was restored by a 12-mm Dacron graft, which was placed end to side into the left iliac artery and end to end to the coeliac trunk. The superior mesenteric artery was implanted directly into the Dacron graft, while the right renal artery was attached by means of a saphenous vein interposition graft (Fig.7.4).

7  Aortic Dissection

79

Fig.7.3  Aortic dissection, with a small, partially thrombosed “true” aortic lumen and dissected superior mesenteric artery

Fig.7.4  Extra-anatomical reconstruction with a Dacron graft, which was placed end to side between the left common iliac artery and end to end to the coeliac trunk. The superior mesenteric artery was implanted directly into the graft, while the right renal artery was implanted via the interposition of a saphenous vein. The left renal artery originates from the aorta

Question 5 What techniques are used to restore blood flow to the visceral organs and extremities following ischaemia from aortic dissection? Which of the following statements are wrong? A.  Aortic stent graft. B.  Percutaneous transluminal angioplasty (PTA) of organ and limb arteries and stenting. C.  Aortic fenestration and membrane resection. D.  Cardiopulmonary bypass. E.  Extra-anatomic revascularisation, e.g. axillo-femoral bypass.

80

B.T. Weis-Müller and W. Sandmann

The complication of postoperative retroperitoneal bleeding from the reconstructed right renal artery had to be managed by relaparotomy and single vascular stitches, and clinical recovery was delayed. The patient required 4 months of rehabilitation until she had regained her previous health status. At this point, digestion and renal function had recovered, laboratory findings became normal, and hypertension had to be treated with only one drug (beta-blocker). Postoperative angiographies showed good perfusion of all visceral and renal arteries via the Dacron graft (Fig.7.4).

7.3  Commentary The life-threatening aortic dissection starts with an intimal tear (entry) in the ascending aorta (Stanford A, de Bakey I or II) or distally to the left subclavian artery (Stanford B, de Bakey III). De Bakey II dissection affects the ascending aorta only, while de Bakey I and III dissections also involve the descending aorta.1, 2 [Q1: A, C] Most patients with acute aortic dissection present with severe chest pain, which might be misinterpreted as acute MI.3,4 Echocardiography, particularly by the transoesophageal approach, is a reliable and rapid method for diagnosis of aortic dissection and differentiation into Stanford A or B type.5 Nevertheless, the evaluation of organ arteries and their blood flow by ultrasound may be difficult in acute dissection. In our opinion, contrast thoracic and abdominal CT scans, especially using the spiral technique, are appropriate diagnostic methods for determining the extension of dissection and the relation of its dissecting membrane to major branches of the aorta. The perfusion of abdominal organs, and often of their arteries, can be seen easily. In the case of organ malperfusion, angiography may be helpful to determine whether the ischaemia is caused by the dissecting membrane of the aorta or whether the dissection extends into the organ arteries.6 Magnetic resonance imaging (MRI) or magnetic resonance angiography (MRA) are effective alternatives in the diagnosis of patients with dissection and renal failure.7 [Q4: A–D] Without treatment, the prognosis of acute aortic dissection is very poor. In 1958, Hirst etal. reviewed 505 cases of aortic dissection and found that 21% of patients died within 24 h of onset and only 20% survived the first month.3 Causes of death in patients with Stanford A dissection include intrapericardial and free intrapleural rupture, acute aortic valve insufficiency, and, to a minor extent, cerebral and coronary malperfusion. In patients with type B dissections, free rupture of the aorta is less frequent. Dissection of the descending aorta may lead, in about 30% of cases, to obstruction of visceral, renal and extremity arteries, resulting in visceral ischaemia, renal insufficiency and acute limb ischaemia, which may be lethal without prompt and adequate therapy.8–10 [Q3: A, C, E] To improve the natural course of the disease, in 1955 de Bakey etal. started to treat acute aortic dissections surgically. Within only a few years, they had developed the current principles of operative intervention in acute Stanford A dissection with replacement of the ascending aorta by a graft in cardiopulmonary arrest. Their results were outstanding, with an overall mortality of 21%.1,11

7  Aortic Dissection

81

However, the surgical experiences of other workgroups were not so successful. Therefore, Wheat et al. developed a new medical treatment with ganglionic blockers, sodium nitroprusside or beta-blockers to influence the hydrodynamic forces of the bloodstream based on the theory that blood pressure and the steepness of the pulse wave are propagating the dissecting haematoma.12 In 1979, a meta-analysis of 219 patients with acute aortic dissection from six centres revealed that Stanford A patients treated medically had a mortality of 74%, whereas 70% of patients survived after surgical therapy. On the other hand, in patients with acute type B dissection, drug therapy alone had a survival rate of 80%, whereas 50% died after operative intervention.13 Therefore in most centres, current therapy for acute dissection type Stanford A is surgical,14–17 and for uncomplicated Stanford B dissection it is medical.18–22 [Q2: A, D, E] An acute dissection, involving the ascending aorta, should be considered a surgical emergency. The aim of operative intervention is to prevent or treat dilation or rupture of the aortic root, and to maintain aortic valve function. The following reconstructive approach is recommended: in patients in whom the root is not involved by dissection, a tubular graft is anastomosed to the sinotubular ridge. In the presence of commissural detachment, the valve is resuspended before supra-commissural graft insertion. If the aortic valve is affected by congenital or acquired abnormalities, then it is generally replaced.15 Patients with acute uncomplicated Stanford B dissection should be treated medically. Careful monitoring is obligatory, while antihypertensive drugs, such as beta-blockers,23 and analgesics are administered. The aim of treatment is to stabilise the dissected aortic wall within 2 weeks and to prevent further extension of dissection or perforation. Careful clinical and laboratory examinations are necessary to detect symptoms of organ or extremity malperfusion in time. Limb, renal and visceral ischaemia can be observed frequently, but paraplegia due to malperfusion of intercostal arteries is rare.6, 8–10 If peripheral vascular complications occur, several therapeutic strategies are possible. Newer publications describe endovascular procedures, for example emergency aortic stenting to close the “entry” and the false aortic lumen.24–27 Ultrasound-guided endovascular catheter aortic membrane fenestration was performed to restore the blood flow to the aortic branches. Dilation and stenting of dissected organ or iliac arteries were performed to resolve stenosis and restore blood flow.28–30 These new therapeutic methods need to be evaluated in long-term follow-up. Aortic surgery in the acute stage of aortic dissection is a dangerous procedure. The dissected aortic wall is extremely friable and does not hold sutures well. Therefore we, and many other centres, try to leave the aorta itself untouched and to restore organ or extremity blood flow by extra-anatomical bypass procedures. Extra-anatomical revascularisation also becomes necessary if the aortic branches themselves are dissected.6, 8, 15 Normally, we use one common iliac artery as the donor vessel for extra-anatomical bypass grafting, but the distal lumbar aorta might also be suitable. If only one aortic branch requires revascularisation, then the iliac-visceral bypass is performed with the saphenous vein (Fig.7.1). If two or more branches are affected, then a Dacron graft is used and the visceral arteries can be implanted into the graft directly or via interposition of the saphenous vein (Fig.7.4). Blood flow to the legs can be restored with a femoral-femoral crossover bypass or with an axillo-(bi)-femoral graft. If several organ arteries are occluded by the aortic dissecting membrane, and the visceral arteries are undissected, then abdominal aortic fenestration

82

B.T. Weis-Müller and W. Sandmann

and membrane resection combined with thrombectomy of the organ arteries can also be performed.31–34 We prefer the latter to treat paraplegia caused by acute aortic dissection. [Q5: D] Our only indication for total aortic replacement in the acute stage of dissection is aortic penetration or perforation.

References   1. De Bakey ME, Henly WS, Cooley DA, etal. Surgical management of dissecting aneurysm of the aorta. J Thorac Cardiovasc Surg. 1965;49:130.   2. Daily PO, Trueblood HW, Stinson EB, Wuerflein RD, Shumway NE. Management of acute aortic dissections. Ann Thorac Surg. 1970;10:237-47.   3. Hirst AE, Johns VJ, Kime SW. Dissecting aneurysm of the aorta: a review of 505 cases. Medicine. 1958;37:217.   4. De Bakey ME, McCollum CH, Crawford ES, etal. Dissection and dissecting aneurysms of the aorta: twenty year follow up of five hundred twenty seven patients treated surgically. Surgery. 1982;92:1118-34.   5. Nienaber CA, von Kodolitsch Y, Nicolas V, etal. The diagnosis of thoracic aortic dissection by non-invasive imaging procedures. N Engl J Med. 1993;328:1-9.   6. Müller BT, Grabitz K, Fürst G, Sandmann W. Die akute Aortendissektion: Diagnostik und Therapie von ischämischen Komplikationen. Chirurg. 2000;71:209.   7. Nienaber CA, von Kodolitsch Y. Bildgebende Diagnostik der Aortenerkrankungen. Radiologie. 1997;37:402.   8. Cambria RP, Brewster DC, Gertler J, etal. Vascular complications associated with spontaneous aortic dissection. J Vasc Surg. 1988;7:199-209.   9. Da Gama AD. The surgical management of aortic dissection: from university to diversity, a continuous challenge. J Cardiovasc Surg. 1991;32:141. 10. Fann JI, Sarris GE, Mitchell RS, etal. Treatment of patients with aortic dissection presenting with peripheral vascular complications. Ann Surg. 1990;212:705-713. 11. De Bakey ME, Cooley DA, Creech O. Surgical considerations of dissecting aneurysm of the aorta. Ann Surg. 1955;142:586. 12. Wheat MW, Palmer RF, Bartley TD, Seelmann RC. Treatment of dissecting aneurysm of the aorta without surgery. J Thorac Cardiovasc Surg. 1965;49:364. 13. Wheat MW, Wheat MD Jr. Acute dissecting aneurysms of the aorta: diagnosis and treatment. Am Heart J. 1979;99:373. 14. Borst HG, Laas J, Frank G, Haverich A. Surgical decision making in acute aortic dissection type A. Thorac Cardiovasc Surg. 1987;35:134. 15. Borst HG, Heinemann MK, Stone CD. Indications for surgery. In: Surgical Treatment of Aortic Dissection. New York: Churchill Livingstone; 1996:103. 16. Heinemann M, Borst HG. Kardiovaskuläre Erkrankungen des Marfan Syndroms. Dt ärztebl. 1996;93B:934. 17. Vecht RJ, Bestermann EMM, Bromley LL, Eastcott HHG. Acute dissection of the aorta: long term review and management. Lancet. 1980;i:109. 18. Bavaria JE, Brinster DR, Gorman RC, etal. Advances in the treatment of acute type A dissection: an integrated approach. Ann Thorac Surg. 2002;74:S1848. 19. Vecht RJ, Bestermann EMM, Bromley LL, Eastcott HHG. Acute aortic dissection: historical perspective and current management. Am Heart J. 1981;102:1087.

7  Aortic Dissection

83

20. Fradet G, Jamieson WR, Janusz MT, etal. Aortic dissection: a six year experience with 17 patients. Am J Surg. 1988;155:697-700. 21. Glower DD, Speier RH, White WD. Management and long-term outcome of aortic dissection. Ann J Surg. 1990;214:31. 22. Hashimoto A, Kimata S, Hosada S. Acute aortic dissection: a comparison between the result of medical and surgical treatments. Jpn Circ J. 1991;55:821. 23. Shores J, Berger KR, Murphy EA, Pyeritz R. Progression of aortic dilatation and the benefit of long-term beta-adrenergic blockade in Marfan´s syndrome. N Engl J Med. 1994;330:1335. 24. Nienaber CA, Fattori R, Lund G, etal. Nonsurgical reconstruction of thoracic aortic dissection by stent-graft replacement. N Engl J Med. 1999;340:1539-1545. 25. Dake MD, Kato N, Mitchell RS, etal. Endovascular stent-graft replacement for treatment of acute aortic dissection. N Engl J Med. 1999;340:1546. 26. Leurs LJ, Bell R, Drieck Y, etal. Endovascular treatment of thoracic aortic diseases: combined experience from EUROSTAR and United Kingdom thoracic Endograft registries. J Vasc Surg. 2004;40:670-680. 27. Hansen CJ, Bui H, Donayre CE. Complications of endovascular repair of high-risk and emergent descending thoracic aortic aneurysms and dissections. J Vasc Surg. 2004;40:228-234. 28. Chavan A, Hausmann D, Dresler C, etal. Intravasal ultrasound guided percutaneous fenestration of the intimal flap in the dissected aorta. Circulation. 1997;96:2124-2127. 29. Farber A, Gmelin E, Heinemann M. Transfemorale Fensterung und Stentimplantation bei aorto-ili-akaler Dissektion. Vasa. 1995;24:389. 30. Slonim SM, Nyman U, Semba CP, Miller DC, Mitchell RS, Dake MD. Aortic dissection: percutaneous management of ischemic complications with endovascular stents and balloon fenestration. J Vasc Surg. 1996;23:241-251. 31. Gurin D, Bulmer JW, Derby R. Dissecting aneurysm of the aorta: diagnosis of operative relief of acute arterial obstruction due to this cause. NY State J Med. 1935;35:1200. 32. Elefteriades JA, Hammond GL, Gusberg RJ, Kopf GS, Baldwin JC. Fenestration revisited. A safe and effective procedure of descending aortic dissection. Arch Surg. 1990;125:786-790. 33. Harms J, Hess U, Cavallaro A, Naundorf M, Maurer PC. The abdominal aortic fenestration procedure in acute thoraco-abdominal aortic dissection with aortic branch artery ischemia. J Cardiovasc Surg (Torino). 1998;39:273-280. 34. Webb TH, Williams GM. Abdominal aortic tailoring for renal, visceral and lower extremity mal-perfusion resulting from acute aortic dissection. J Vasc Surg. 1997;26:474.

Popliteal Artery Aneurysms

8

Susanna Shin and Michel Makaroun

A 62-year-old male patient presented to the emergency department with a cool right foot. On examination, his femoral pulse is intact and a pulsatile mass is appreciated in the popliteal fossa. His right foot is cool but motor and sensory function are intact. No pedal pulses are palpable and faint Doppler signals are audible.

Question 1 The presence of a popliteal artery aneurysm increases a patient’s risk for: A.  Contralateral popliteal artery aneurysm B.  Infra-renal abdominal aortic aneurysm C.  Other peripheral artery aneurysms D.  All of the above

Question 2 Which of the following is the initial diagnostic test of choice for popliteal artery aneurysm? A.  Magnetic resonance imaging B.  Contrast arteriography C.  Duplex ultrasonography D.  Computed tomography angiography Duplex ultrasonography demonstrates giant bilateral popliteal artery aneurysms and a 4.5cm infra-renal abdominal aortic aneurysm. A computed tomography (CT) angiogram is obtained to further evaluate the aortic aneurysm and both lower extremities are included (Fig.8.1). The right (symptomatic) popliteal artery aneurysm is resected through a posterior approach with the ipsilateral greater saphenous vein used as an interposition graft. After his recovery from this repair, the same approach is used to repair the contralateral aneurysm. S. Shin () Division of Vascular Surgery, University of Pittsburgh Medical Center, Pittsburgh, PA, USA G. Geroulakos and B. Sumpio (eds.), Vascular Surgery, DOI: 10.1007/978-1-84996-356-5_8, © Springer-Verlag London Limited 2011

85

86

S. Shin and M. Makaroun

Fig.8.1  Computed tomography angiogram demonstrating bilateral giant popliteal artery aneurysms

Question 3 Popliteal aneurysms can present with A.  Distal embolization B.  Acute thrombosis C.  Swelling from venous compression D.  Asymptomatic

Question 4 Emergent repair of popliteal artery aneurysms results in similar graft patency and limb preservation when compared to elective repair. A.  True B.  False

An 82-year-old female patient is referred for evaluation of a right blue second toe. She complains of a painful toe that has been blue for quite some time. On examination, her femoral pulse is intact with a prominent popliteal pulse on the right with thready pedal pulses bilaterally. A duplex ultrasound demonstrates a 2.0cm popliteal artery aneurysm with 3–4cm of normal artery proximal and distal to the aneurysm. The left popliteal artery is normal in size without thrombus. A CT angiogram is obtained and confirms a partially thrombosed 2.0 cm popliteal artery aneurysm (Fig.8.2). The patient has a history of CAD, CHF with a left ventricular ejection fraction of 25%. The right lower extremity angiogram shows the runoff (Fig.8.3).

8  Popliteal Artery Aneurysms Fig.8.2  Computed tomography angiogram demonstrating a 2.0cm right popliteal artery aneurysm that is partially thrombosed

Fig.8.3  Diagnostic angiogram demonstrating three-vessel distal runoff

87

88

S. Shin and M. Makaroun

Question 5 Which of the following are acceptable options in the treatment of a popliteal artery aneurysm? A.  Thrombolytics followed by ligation and bypass of an acutely thrombosed aneurysm B. Resection and interposition vein graft of an aneurysm causing local compressive symptoms C. Endovascular stent graft of an aneurysm in a 78-year-old COPD patient with severe CAD D.  Thrombectomy alone of an acutely thrombosed aneurysm This patient is at high risk for operative repair of the popliteal artery aneurysm and endovascular exclusion would offer her better peri-operative morbidity and mortality. The anatomy of her aneurysm is acceptable for endovascular repair with adequate landing zones proximal and distal to the aneurysm with three-vessel runoff. A 6mm (diameter) by 10cm (long) stent graft is placed in the popliteal artery to exclude the aneurysm. Completion angiogram demonstrates preserved runoff and no kinking of the stent graft with the knee bent (Fig.8.4).

Fig.8.4  Completion angiogram demonstrating no kinking with the knee bent and preserved runoff

8  Popliteal Artery Aneurysms

89

8.1  Popliteal Artery Aneurysm Popliteal artery aneurysms are the most common peripheral artery aneurysm. The popliteal artery is considered aneurysmal at a diameter of 1.5cm and complications usually occur once the aneurysm grows to 2cm or greater. Atherosclerosis is the primary underlying pathology in the formation of most popliteal artery aneurysms and they affect a predictable population, occurring most often in men in their 60s and 70s.1–4 The presence of a popliteal artery aneurysm increases the risk for other aneurysms; 36–54% are bilateral and 25–54% occur synchronously with infrarenal abdominal aortic aneuryms.1–6 [Q1: D] Diagnosis of a popliteal artery aneurysm is suspected with the detection of a prominent pulse or ­pulsatile mass in the popliteal fossa on physical exam. This is confirmed with duplex ultrasonography which can differentiate the aneurysmal segment from other masses in the popliteal fossa and demonstrate mural thrombus. [Q2: C] Angiography can be an important adjunctive exam to determine distal run-off in preparation for surgical repair. Although a significant percentage of these aneurysms are diagnosed incidentally, the majority (58–71%) of popliteal artery aneurysms are symptomatic at the time of diagnosis.1,3,5,7 Most common presentations are distal embolization or acute thrombosis, followed by compressive symptoms.6 Compression of adjacent structures of the popliteal fossa can cause venous obstruction (deep venous thrombosis) and pain (compression of adjacent nerves). Rupture can occur rarely, in less than 5% of the presentations.1,6 [Q3: A, B, C, D] Distal embolization can cause minor or major tissue loss but more importantly, it destroys distal run-off, decreasing patency of operative repair. Indications for repair include size of 2cm and greater, the presence of significant mural thrombus, compression of adjacent structures causing pain and/or venous obstruction and symptoms of embolization. Elective repair in asymptomatic patients results in excellent graft patency and limb preservation. Conversely, repair in symptomatic patients has decreased graft patency and limb salvage rates, particularly in emergent repair for acute thrombosis and rarely rupture.1,2,7,8 [Q4: B] Therefore, popliteal aneurysms are better repaired in the asymptomatic state once they reach 2cm, or when associated with significant thrombus. Options for repair include open bypass with ligation using a medial approach, open aneurysmorrhaphy via a posterior approach or endovascular stent grafting. Open repair is the gold standard in the treatment of popliteal artery aneurysm. The medial approach is most often utilized as it offers the best exposure of the distal superficial femoral artery, the trifurcation and the greater saphenous vein. The posterior approach is sometimes preferred in cases with limited extent of the disease especially when ligation of all branches of the aneurysm is necessary to relieve compressive symptoms. Endovascular repair of a popliteal artery aneurysm is a minimally invasive approach that has gained acceptance recently with the addition of kink resistant stent grafts. It is a good alternative to open repair in patients with suitable anatomy especially poor operative candidates. Agood runoff and suitable landing zones are important determinants of success. Small studies have shown excellent results with endovascular repair with similar patency at

90

S. Shin and M. Makaroun

intermediate follow-up and faster/shorter recovery.8 Contraindications to endovascular stent grafting for popliteal artery aneurysms include compressive symptoms and singlevessel run-off. An acutely thrombosed popliteal artery aneurysm often presents as an acutely ischemic limb and requires emergent therapy. Systemic anti-coagulation should be initiated immediately and directed thrombolytics improve distal run-off in preparation for surgical repair. [Q5: A, B, C]

References 1. Lichtenfels E, Frankini AD, Bonamigo TP, etal. Popliteal artery aneurysm surgery: the role of emergency setting. Vasc Endovasc Surg. 2008;42(2):159-164. 2. Ravn H, Wanhainen A, Bjorck M. Surgical technique and long-term results after popliteal artery aneurysm repair: results from 717 legs. J Vasc Surg. 2007;46(2):236-243. 3. Martelli E, Ippoliti A, Ventoruzzo G, etal. Popliteal artery aneurysms. Factors associated with thromboembolism and graft failure. Int Angiol. 2004;23(1):54-65. 4. Huang Y, Gloviczki P, Noel AA, etal. Early complications and long-term outcome after open surgical treatment of popliteal artery aneurysms: is exclusion with saphenous vein bypass still the gold standard? J Vasc Surg. 2007;45(4):706-713. discussion 713-115. 5. Ascher E, Markevich N, Schutzer RW, etal. Small popliteal artery aneurysms: are they clinically significant? J Vasc Surg. 2003;37(4):755-760. 6. Ravn H, Bergqvist D, Bjorck M. Nationwide study of the outcome of popliteal artery aneurysms treated surgically. Br J Surg. 2007;94(8):970-977. 7. Pulli R, Dorigo W, Troisi N, etal. Surgical management of popliteal artery aneurysms: which factors affect outcomes? J Vasc Surg. 2006;43(3):481-487. 8. Antonello M, Frigatti P, Battocchio P, et al. Open repair versus endovascular treatment for asymptomatic popliteal artery aneurysm: results of a prospective randomized study. J Vasc Surg. 2005;42(2):185-193.

Renal Artery Aneurysm

9

Lutz Reiher, Tomas Pfeiffer, and Wilhelm Sandmann

A 45-year-old woman presented with a 10-year history of arterial hypertension. After initially successful conservative therapy with two antihypertensive drugs, arterial blood pressure was not controlled well during the last months. To exclude a ­renovascular origin of hypertension, an angiography was performed, which showed fibrodysplastic disease of the right renal artery with several stenotic segments and aneurysms (Fig.9.1).

Fig.9.1  Selective intraarterial renal artery angiography revealed renal artery aneurysm (RAA) combined with renal artery stenosis (RAS) due to fibromuscular dysplasia.

L. Reiher () Klinik für Gefäßchirurgie und Nierentransplantation, Universitätsklinikum Düsseldorf, Heinrich-Heine-Universität, Düsseldorf, Germany G. Geroulakos and B. Sumpio (eds.), Vascular Surgery, DOI: 10.1007/978-1-84996-356-5_9, © Springer-Verlag London Limited 2011

91

92

L. Reiher et al.

Question 1 Which of the following statements regarding renal artery aneurysm (RAA) is correct? A.  It has a marked female preponderance. B.  It is usually diagnosed during examination for flank pain. C.  It may cause arterial hypertension. D.  It typically leads to proteinuria by compression of the renal vein. E.  It can cause haematuria in rare cases.

Question 2 Which statements about the aetiology of the RAA are true? A. The most frequent underlying diseases of RAA are aortic coarctation with con-comitant disease of the renal artery and renal artery dissection. B. Fibromuscular dysplasia of the renal artery may present with renal artery stenosis (RAS), RAA or both. C.  Arteriosclerosis is a frequent cause of RAA. D.  Some RAA present with inflammation of the arterial wall. E. The incidence of RAAs is increased in Ehlers–Danlos syndrome and Marfan´s syndrome.

Question 3 Which risks of the spontaneous course of the RAA should you explain to your patient? A.  The RAA may rupture and lead to a life-threatening bleeding. B.  The risk of rupture decreases during pregnancy and childbirth. C.  Hypertension in RAA may be caused by concomitant stenosis of the renal artery or its branches. D.  In cases of RAA and hypertension the angiography of the renal artery always shows an additional RAS. E.  The RAA may be a source of embolisation leading to a loss of renal function.

Question 4 Which of the following statements regarding the indication of renal artery repair (RAR) for RAA is correct? A.  There is an indication for RAR only in cases of symptoms other than hypertension. B. There is no reason to perform RAR in women of childbearing age if there is no arterial hypertension. C.  There is a good indication for RAR if a concomitant RAS is found. D.  There is a good indication for RAR only if the RAA is larger than 5.5 cm. E.  There is an indication for RAR in patients presenting with RAA and hypertension even if an additional RAS is not detectable.

9  Renal Artery Aneurysm

93

Fig.9.2  Postoperative angiography demonstrates a patent aortorenal venous graft.

For RAR, a midline abdominal incision was performed for direct access to the infrarenal aorta, where an end-to-side anastomosis was performed with a segment of the patient´s greater saphenous vein. After Kocher´s manoeuvre, the distal renal artery was transected and anastomosed to the saphenous vein, which had been placed on the renal hilus dorsal to the inferior vena cava. Good results were shown by postoperative angiography (Fig.9.2). At re-examination 3 years after the operation, the patient had a normal blood pressure without antihypertensive medication.

Question 5 Which of the following statements regarding the management of RAA is correct? A.  Replacement of the diseased renal artery by prosthetic graft is the RAR of first choice. B.  Protection of the kidney against ischaemic injury is performed only during ex situ reconstruction of the renal artery. C.  RAA exclusion and aortorenal vein graft interposition, or RAA resection and end-toend anastomosis or aneurysmorrhaphy, are valuable methods for RAR. D.  Ex situ repair of the renal artery may be needed in cases presenting with lesions of the distal branch arteries. E.  Tailoring of RAA often leads to recurrent aneurysmatic dilation of the renal artery.

9.1  Commentary RAAs do not usually cause symptoms, and generally they are diagnosed accidentally during work-up for hypertension, as in our patient. In rare cases, flank pain has been described as the initial symptom, which may be due either to the size of the RAA or to a renal artery

94

L. Reiher et al.

dissection. Rupture of the aneurysm into the urinary tract will lead to haematuria. [Q1: A, C, E] The underlying disease is most frequently dysplasia of the arterial wall followed by arteriosclerosis. In our case, fibromuscular dysplasia was found to be the aetiology of the RAA. Rare causes of RAA may be atypical aortic coarctation with concomitant disease of the renal arteries, inflammation of the arterial wall, dissection or trauma, or disorders of the elastic and collagen fibres (i.e. Ehlers–Danlos syndrome or Marfan´s syndrome). [Q2: B, C, D, E] RAA is found about twice as often in the right renal artery as in the left. Selective angiography often reveals concomitant RAS of mainstem and segmental arteries, and segmental arteries may also be aneurysmal. Concomitant renal artery dissection is rare. Rupture of RAA, development or deterioration of arterial hypertension, and loss of renal function by thrombosis or embolisation, are impending spontaneous consequences of RAA. As with all arterial aneurysms, rupture is a possible complication of RAA. While Tham etal.1 experienced no rupture of RAA in 69 patients who had been treated conservatively during a mean observation time of 4.3 years, Henriksson etal.2 observed RAA rupture in four cases (10.2%), and at the time of rupture only a nephrectomy could be performed. There are several case reports about RAA rupture in pregnancy and childbirth,3–5 and one author found the probability of RAA rupture during pregnancy to be as high as 80%.6 As high arterial blood pressure is in itself a risk factor for rupture of arterial aneurysms of any localisation, one can argue that hypertension per se is an indication to remove an RAA. Hypertension was found in 90% of all patients with ruptured RAA.7 The larger the diameter of the RAA, the more likely the danger of rupture seems to be, which can be explained by Laplace’s law. However, RAAs of any diameter can rupture. In one patient cohort,8 the smallest (1 cm) and the largest (16.5 cm) RAAs ruptured. About 80% of patients with RAA have arterial hypertension.9, 10 If RAA is accompanied by RAS on the same or the contralateral side, as in our patient, then it is reasonable to remove both, with the intention to improve hypertension and eliminate the risk of rupture. However, an ipsilateral stenosis may be missed by angiography due to overprojection of the aneurysm. Furthermore, aneurysmal disease includes not only dilation of vessels but also elongation, which might cause kinking with a relevant stenosis.11 [Q3: A, C, E] There is an absolute indication to remove RAAs in all patients with arterial hypertension with and without concomitant RAS and in women of childbearing age. [Q2: C] RAAs with a diameter greater than 2 cm should be removed, even if there is no hypertension. There are good long-term results for autologous RAR; therefore, there is a relative indication for operation in younger patients without hypertension and concomitant RAS with RAA of diameter of 1 cm or more. [Q4: C, E] The most promising method of RAR is by autogenous reconstruction. Methods of RAR are replacement of the renal artery by the greater saphenous vein, resection of diseased sections and reanastomosis. The autoplastic reconstruction by tailoring (synonym: aneurysmorrhaphy) is another appropriate technique. Although the aneurysmatic wall is only resected partially, recurrent RAAs have not been observed. The in situ reconstruction is less traumatic, but ex situ repair of the renal artery may be necessary in cases in which not only the distal mainstem artery but also the segmental arteries are involved. [Q5: C, D]

9  Renal Artery Aneurysm

95

If arterial repair is restricted to renal arteries only, and if concomitant repair of the aorta is not necessary, then a postoperative mortality less than 1% can be expected. Postoperative morbidity is due to temporary kidney insufficiency, graft thrombosis, bleeding, thrombosis and pancreatitis. Affected kidneys can be preserved in more than 85% of cases. The number of patients who benefit from surgical therapy in terms of improvement of arterial hypertension varies considerably between authors, ranging from 5% to 50% and from 25% to 62%, respectively.12

References   1. Tham G, Ekelund L, Herrlin K, Lindstedt EL, Olin T, Bergentz SE. Renal artery aneurysms. Natural history and prognosis. Ann Surg. 1983;197:348-352.   2. Henriksson C, Lukes P, Nilson AE, Pettersson S. Angiographically discovered, non-operated renal artery aneurysms. Scand J Urol Nephrol. 1984;18:59-62.   3. Rijbroek A, Dijk HA, Roex AJM. Rupture of renal artery aneurysm during pregnancy. Eur J Vasc Surg. 1994;8:375-376.   4. Smith JA, Macleish DG. Postpartum rupture of a renal artery aneurysm to a solitary kidney. Aust N Z J Surg. 1985;55:299-300.   5. Whiteley MS, Katoch R, Kennedy RH, Bidgood KA, Baird RN. Ruptured renal artery aneurysm in the first trimester of pregnancy. Eur J Vasc Surg. 1994;8:238-239.   6. Love WK, Robinette MA, Vernon CP. Renal artery aneurysm rupture in pregnancy. J Urol. 1981;126:809-811.   7. Abud O, Chelile GE, Sole-Balcells F. Aneurysm and arteriovenous malformation. In: Novick AC, Scoble J, Hamilton G, eds. Renal Vascular Disease. London: Saunders; 1996:35-46.   8. Hupp T, Allenberg JR, Post K, Roeren T, Meier M, Clorius JH. Renal artery aneurysm: surgical indications and results. Eur J Vasc Surg. 1992;6:477-486.   9. Martin RSD, Meacham PW, Ditesheim JA, Mulherin JL Jr, Edwards WH. Renal artery aneurysm: selective treatment for hypertension and prevention of rupture. J Vasc Surg. 1989;9:26-34. 10. Brekke IB, Sodal G, Jakobsen A, etal. Fibro-muscular renal artery disease treated by extracorporeal vascular reconstruction and renal autotransplantation: short- and long-term results. Eur J Vasc Surg. 1992;6:471-476. 11. Poutasse EF. Renal artery aneurysms. J Urol. 1975;113:443-449. 12. Pfeiffer T, Reiher L, Grabitz K, et al. Reconstruction for renal artery aneurysm: operative techniques and long-term results. J Vasc Surg. 2003;37:293-300.

Anastomotic Aneurysms

10

Jonothan J. Earnshaw

A 70-year-old woman presented with bilateral pulsatile groin masses (Fig.10.1). Six years ago, she had an elective aorto-bifemoral graft for a 6-cm abdominal aortic aneurysm involving both iliac arteries, from which she made a full recovery. She first found the larger, right-sided mass 4months ago, and she had noted gradual enlargement since then. She had no symptoms of claudication or leg ischemia. Her past medical history included a myocardial infarction (MI) 18months ago, but without limitation to her exercise tolerance. On examination, she appeared well. There was a well-healed midline laparotomy scar from the previous operation. Abdominal examination was unremarkable, and there were no bruits on auscultation. Two well-defined expansile masses were palpable in the middle third of the femoral scars, measuring approximately 2cm on the left and 4cm on the right. The masses were not tender. There was no evidence of compromise to the distal circulation, and all pulses were palpable. Duplex imaging identified anastomotic false aneurysms in both groins, measuring 1.8cm on the left and 3.5cm on the right.

Fig.10.1  Female patient with bilateral anastomotic aneurysms from an aortobifemoral graft

J.J. Earnshaw Department of Surgery, Gloucestershire Royal Hospital, Gloucester, UK G. Geroulakos and B. Sumpio (eds.), Vascular Surgery, DOI: 10.1007/978-1-84996-356-5_10, © Springer-Verlag London Limited 2011

97

98

J.J. Earnshaw

Question 1 Which of the following statements regarding the etiology of anastomotic false aneurysms are correct? A. Anastomotic false aneurysms occur in 3–5% of anastomoses to the femoral artery in thegroin B.  Forty per cent are found in the groin C.  Primary degeneration of the arterial wall is an etiological factor D.  Continued smoking is an etiological factor E. At reoperation, approximately one-third will be found to be infected with pathogenic bacteria

Question 2 The patient wished to know the risks of leaving the aneurysm alone. Rank the potential complications of anastomotic aneurysms in order of frequency. A.  Rupture B.  Embolization C.  Pressure symptoms D.  Pain E.  Secondary hemorrhage

Question 3 Which of the following non-operative treatments are also available? A.  Embolization B.  Ultrasound-guided compression C.  Thrombin injection D.  Intravascular stent graft The larger of the two aneurysms was repaired surgically. The previous surgical incision was reopened and extended. A large false aneurysm was confirmed; the graft appeared to have become detached from the artery. There were no signs of infection. The aneurysm was replaced by straight 8-mm gelatin-coated woven Dacron interposition graft (soaked in rifampicin solution 10 mg/mL) taken end to end from the old graft and sutured end to side over the common femoral bifurcation. The thrombus and old graft were sent for microbiology. The patient made a good postoperative recovery. All bacterial cultures were negative, so perioperative antibiotic prophylaxis was stopped after 48h.

10  Anastomotic Aneurysms

99

Question 4 Rank the following surgical procedures in order of value for the management of anastomotic aneurysm in the groin (least useful first): A.  Resuture or local repair B.  Ligation and bypass C.  Prosthetic patch D.  Vein patch E.  Interposition graft This patient at 2-year follow-up had no evidence of recurrence of the anastomotic aneurysm in her right groin. A follow-up ultrasound scan of her left groin revealed that the left anastomotic aneurysm remained 2cm in maximum diameter.

Question 5 Which of the following statements are false. A.  Surgery cures 50% of all anastomotic aneurysms. B.  Surgery cures 90% of all anastomotic aneurysms. C.  Surgery cures 50% of all recurrent anastomotic aneurysms. D.  Surgery cures 90% of all recurrent anastomotic aneurysms. E.  Long-term follow-up of retroperitoneal anastomotic aneurysms is not necessary.

10.1  Commentary The incidence of anastomotic aneurysms is increasing, due primarily to the increased frequency of prosthetic vascular reconstructions involving groin anastomosis. The overall incidence following vascular anastomoses is about 2%, but this increases to 3–8% when the anastomosis involves the femoral artery.1–4 Although they are most common after prosthetic bypass, anastomotic aneurysms occasionally occur after vein bypass, semi-closed endarterectomy, and open endarterectomy with a vein patch. Anastomotic aneurysms can occur anywhere, but they frequently develop near to a joint. About 80% occur at the groin,1 presumably due to movement-related strains. [Q1: A, C, D, E] The etiology is summarized in Fig.10.2; there are three primary factors and a number of secondary factors. One of the first documented causes was suture failure, when braided silk was employed for vascular anastomoses.5 Since monofilament sutures have been used, suture failure has become a less common factor, although occasionally reported disasters highlight the importance of careful suture handling to avoid cracking of the polypropylene.6

100

J.J. Earnshaw

Primary factors

Arterial degeneration 65%

Infection 30%

Suture failure 5%

Secondary factors Arterial weakness

Increased forces across the anastomosis

Endarterectomy Poor suture technique Reoperative surgery Hyperlipidaemia Smoking Distal disease progression Poststenotic dilation Steroid therapy Radiotherapy

Hypertension Anastomotic tension Compliance mismatch Dacron dilation High outflow resistance Hip joint motion Trauma

Fig.10.2  Etiology of anastomotic aneurysms

Arterial degeneration is the most common primary factor. The disease process that mandated the bypass continues after its insertion.1,7,8 Histologically, a chronic inflammatory response can be identified at an anastomosis.9 Secondary factors are numerous and compound the process of arterial degeneration.10 Poor technique, failing to suture all layers of the artery, use of Dacron, and the need for endarterectomy all weaken the arterial graft complex.1 Hypertension and high outflow resistance may theoretically increase strains at the anastomosis, together with physical disruption from both hip motion and poststenotic dilation as the graft passes under the inguinal ligament.9 These and other factors can cause compliance mismatch, which may also be a factor.8 Anastomotic aneurysms can be caused by local infection. Infection with high-virulence bacteria, such as Staphylococcus aureus, usually presents early with clinical graft infection. Late anastomotic rupture is often caused by low-virulence organisms, such as Staphylococcus epidermidis. Up to 30% of anastomotic aneurysms can be shown to harbor pathogenic bacteria at reoperation.7 This has implications for surgical repair (see below). [Q2: D, C, B, A, E]

10.2  Indications for Intervention Treatment of anastomotic aneurysms is aimed at controlling symptoms or preventing the onset of complications. Symptoms of pain are associated with the enlarging mass or

10  Anastomotic Aneurysms

101

pressure on adjacent structures, such as the femoral nerve. Complications may be local or distal. The enlarging aneurysm may occlude the underlying vessel, causing distal ischaemia. Emboli associated with flow disruption may be propagated distally. Aneurysm rupture represents the greatest worry but is relatively rare. Complications are related to aneurysm size. Therefore, conservative management may be undertaken if the aneurysm is small and easily accessible, and demonstrates no evidence of progressive enlargement or symptoms. Aneurysms less than 2cm in diameter can be observed safely.1 Above this size, the incidence of complications rises and intervention should be considered. However, the medical state of the patient may necessitate selected aneurysms larger than 2cm being managed conservatively by watchful waiting. False aneurysms caused iatrogenically following direct arterial puncture must be differentiated from anastomotic aneurysms because their treatment differs substantially. False aneurysms following sterile arterial puncture may be treated by arterial compression under duplex imaging.11 More recently, injection of thrombin into these false aneurysms has been shown to be safe and effective, even in anticoagulated patients.12 This technique is not suitable for anastomotic aneurysms. Other radiological techniques may be used selectively for false aneurysms in inaccessible positions, such as the renal or subclavian arteries, where coil embolization may be used to occlude the feeding vessel.13 Again, this is rarely suitable for anastomotic aneurysms. Occasionally, endovascular treatment with a covered stent can be employed across an anastomotic aneurysm to produce aneurysm sac thrombosis14, 15 and to maintain normal distal flow. This technique is particularly valuable for intra-abdominal aortoiliac anastomotic aneurysms, where reoperation carries substantial risk. It is important that endovascular techniques are not used in situations where there is any risk that the false aneurysm is due to infection. The most common site for anastomotic aneurysm is the groin, where non-operative techniques have not been found to be effective. The groin is also easily accessible for surgery, so direct operation is the usual intervention in this situation. [Q3: A, B, C, D] [Q4: A, C, D, E, B]

10.3  Treatment for Anastomotic Aneurysms Surgical repair should be undertaken in fit patients with large or symptomatic anastomotic aneurysms. Local repair is usually possible in non-infected aneurysms, although graft replacement may be necessary. If infection is the cause of the aneurysm, then more extensive repairs with ligation and remote bypass or replacement of the entire initial graft may be needed.16 Anastomotic aneurysms usually occur in arteriopathic patients. Careful preoperative planning is needed to make the patient as fit as possible. General anesthesia is needed to allow adequate exposure, and the surgery is carried out under antibiotic and heparin cover. Once vascular control above and below the aneurysm has been obtained with minimal dissection, the aneurysm should be opened, along with the entire abnormal artery. Occlusion balloon catheters are often helpful in obtaining vascular control in this situation. The false aneurysm

102

J.J. Earnshaw

is usually resected and the ends of the graft and artery freshened for reanastomosis. Interposition grafting is likely to be needed to ensure that the new anastomosis is created without tension. Autologous saphenous vein is the graft of choice, although often polytetrafluoroethylene (PTFE) or Dacron may be better for size matching. [Q5: A, C, E] Retroperitoneal anastomotic aneurysms present more of a challenge. Proximal aortic anastomotic aneurysms may require supracoeliac clamping or balloon occlusion catheters.17 Aneurysms associated with the distal portion of an aortoiliac graft may present late and catastrophically, illustrating the potential importance of monitoring these grafts for a prolonged period.18 As previously stated, and endovascular approach is used increasingly in this situation.

a

Fig.10.3  (a) This man presented with sudden pain in the right groin. A false aneurysm of a previous axillobifemoral graft was diagnosed on ultrasound imaging. Note the inflammatory nature of the lump suggesting infection. (b) At operation the hood of the graft had separated completely from the artery. There was no sign of sepsis, and all bacterial cultures were negative

b

10  Anastomotic Aneurysms

103

10.4  Infection in Anastomotic Aneurysms Some 80% of anastomotic aneurysms occur in the groin, and they have the highest incidence of infection as their primary cause; approximately 30% contain pathogenic bacteria. A high level of clinical suspicion of infection must be maintained, and Gram staining of all clots and removed graft should be carried out as a matter of routine. Perioperative antibiotics should be continued until results are available (Fig.10.3). The diagnosis of infection is usually obvious if the graft is surrounded by pus. If the graft is frankly infected, it should be excised completely with an extra-anastomotic bypass to restore the distal circulation with prolonged, high-dose antibiotic cover. An obturator bypass may be used for an infected femoral false aneurysm, or a femoral crossover with saphenous vein. Aortic stump oversewing and axillobifemoral grafting can treat the (fortunately rare) infected aortic anastomotic aneurysm. Morbidity and mortality rates are high. Grafts with a more indolent level of infection that becomes apparent only after microbiological investigation may be treated less radically. It is safest to assume that all femoral anastomotic aneurysms are contaminated. If prosthetic material is needed for repair, then measures used to reduce the chance of reinfection include the use of a rifampicin-soaked, gelatin-coated Dacron graft and gentamicin beads laid in close proximity. The reinfection rate after such procedures is 10%.19

10.5  Outcome Outcome depends on the initial site of the aneurysm and any confounding factors.20 As the most common site for anastomotic aneurysms, the femoral artery has one of the highest rates of successful outcome. About 90% of surgical procedures are successful, and those that recur still have a 90% success rate from a second or subsequent operation. In comparison, anastomotic aneurysms that are intra-abdominal have a high complication rate when repaired surgically. A small anastomotic aneurysm in a superficial position can be monitored by ultrasound or by repeated examination by a clinician or the motivated patient. The success rate of operation at these sites is good.21 Retroperitoneal aneurysms require longterm ultrasound follow-up. [Q6: F, T, F, T, F] If possible, minimally invasive techniques should be used for repair to avoid the high morbidity and mortality associated with surgery (providing infection is not present). In patients fit for surgery, excision and graft interposition has excellent long-term results.

104

J.J. Earnshaw

References   1. Szilagyi DE, Smith RF, Elliott JP, Hageman JH, Dall’Olmo CA. Anastomotic aneurysms after vascular reconstruction: problems of incidence, etiology and treatment. Surgery. 1975;78: 800-816.   2. Waibel P. False aneurysm after reconstruction for peripheral arterial occlusive disease. Observations over 15–25years. Vasa. 1994;23:43-51.   3. Stone PA, AbuRhama AF, Flaherty SK, Bates MC. Femoral pseudoaneurysms. Vasc Endovasc Surg. 2006;40:109-117.   4. Corriere MA, Guzman RJ. True and false aneurysms of the femoral artery. Semin Vasc Surg. 2005;18:216-223.   5. Moore WS, Hall AD. Late suture failure in the pathogenesis of anastomotic false aneurysms. Ann Surg. 1970;172:1064-1068.   6. Berridge DC, Earnshaw JJ, Makin GS, Hopkinson BR. A ten-year review of false aneurysms in Nottingham. Ann R Coll Surg Engl. 1988;70:253-256.   7. Wandschneider W, Bull O, Deneck H. Anastomotic aneurysms: an unsolvable problem. Eur J Vasc Endovasc Surg. 1988;2:115-119.   8. Gayliss H. Pathogenesis of anastomotic aneurysms. Surgery. 1981;90:509-515.   9. Sladen JG, Gerein AN, Miyagishima RT. Late rupture of prosthetic aortic grafts. Am J Surg. 1987;15:453-458. 10. De Monti M, Ghilardi G, Sgroi G, Longhi F, Scorza R. Anastomotic pseudoaneurysm, true para-anastomotic aneurysm and recurrent aneurysm following surgery for abdominal aortic aneurysm. Is a unifying theory possible? Minerva Cardioangiol. 1995;43:367-373. 11. Hajarizadeh H, LaRosa CR, Cardullo P, Rohrer MJ, Cutler BS. Ultrasound guided compression of iatrogenic femoral psuedoaneurysm: failure, recurrence and long term results. J Vasc Surg. 1995;22:425-430. 12. Kang SS, Labropoulos N, Mansour MA, etal. Expanded indications for ultrasound-guided thrombin injection of pseudoaneurysms. J Vasc Surg. 2000;31:289-298. 13. Uflacker R. Transcatheter embolisation of arterial aneurysms. Br J Radiol. 1986;59:317-324. 14. Manns RA, Duffield RG. Intravascular stenting across a false aneurysm of the popliteal artery. Clin Radiol. 1997;52:151-153. 15. Brittenden J, Gillespie I, McBride K, McInnes G, Bradbury AW. Endovascular repair of aortic pseudoaneurysms. Eur J Vasc Endovasc Surg. 2000;19:82-84. 16. Clarke AM, Poskitt KR, Baird RN, Horrocks M. Anastomotic aneurysms of the femoral artery: aetiology and treatment. Br J Surg. 1989;76:1014-1016. 17. Ernst CB. The surgical correction of arteriosclerotic femoral aneurysm and anastomotic aneurysm. In: Greenhalgh RM, Mannick JA, eds. The Cause and Management of Aneurysms. London: W.B. Saunders; 1990:245-256. 18. Treiman GS, Weaver FA, Cossman DV, etal. Anastomotic false aneurysms of the abdominal aorta and the iliac arteries. J Vasc Surg. 1988;8:268-273. 19. Earnshaw JJ. Anastomotic/false aneurysms. In: Horrocks M, ed. Arterial Aneurysms: Diagnosis and Management. Bath: Butterworth Heinemann; 1995:209-221. 20. Ylonen K, Biancari F, Leo E, etal. Predictors of development of anastomotic femoral pseudoaneurysms after aortobifemoral reconstruction for abdominal aortic aneurysm. Am J Surg. 2004;187:83-87. 21. Woodburn K. False aneurysms. In: Earnshaw JJ, Parvin S, eds. Rare Vascular Disorders. Tfm Publishing, Ltd.; 2005:283–292.

False Aneurysm in the Groin Following Coronary Angioplasty

11

Steven S. Kang

A 70-year-old female with a history of hypertension developed chest pain and came to the Emergency Room. Her electrocardiogram showed ST segment elevation. The patient was administered aspirin, clopidogrel, and intravenous heparin. Within 60min, she underwent coronary angiography, which showed a critical stenosis of the left anterior descending artery. The lesion was treated with angioplasty and stent placement. The right femoral artery sheath was left in place overnight, and heparin was continued. The following morning after stopping heparin, the sheath was removed and a FemoStop device was placed over the groin for 4h. Heparin was then restarted. The next day, the patient was without any chest pain, but she did have mild discomfort in the right groin. There was a large hematoma in the right groin. The overlying skin had ecchymosis. The femoral pulse was prominent, and popliteal and pedal pulses were normal. A systolic bruit was heard over the femoral artery.

Question 1 What test should be obtained at this time? A.  Computed tomography scan with intravenous contrast B.  Duplex ultrasound C.  Magnetic resonance angiogram D.  Contrast arteriogram A false aneurysm was suspected and confirmed by duplex ultrasound examination. It was arising from the common femoral artery (CFA). The flow cavity measured 3cm in diameter (Fig.11.1).

S.S. Kang Department of Surgery, Florida International University School of Medicine, Miami, FL, USA G. Geroulakos and B. Sumpio (eds.), Vascular Surgery, DOI: 10.1007/978-1-84996-356-5_11, © Springer-Verlag London Limited 2011

105

106

S.S. Kang

Fig.11.1  Duplex ultrasound demonstrates a false aneurysm arising from the common femoral artery

Question 2 The incidence of postcatheterization false aneurysms in the groin is higher under which of the following situations? A.  Puncture of the CFA instead of the superficial femoral artery (SFA) B.  Use of larger sheaths C.  Use of postprocedural anticoagulation D.  Patients with hypertension E.  Manual compression versus mechanical compression with a FemoStop after catheter removal

Question 3 Which of the following statements about postcatheterization false aneurysms is/are true? A.  Urgent surgical repair is indicated B.  This aneurysm is likely to undergo spontaneous thrombosis if observed C.  Spontaneous thrombosis is less common in patients who are anticoagulated D.  They may cause deep venous thrombosis

11  False Aneurysm in the Groin Following Coronary Angioplasty

107

Heparin was discontinued and ultrasound-guided compression repair (UGCR) was attempted.

Question 4 Which are disadvantages of UGCR? A.  Thrombosis of the underlying artery is a frequent complication B.  Most patients find it painful C.  It is less successful in patients who are anticoagulated D.  Approximately 30% of successfully thrombosed false aneurysms recur Due to patient discomfort, intravenous morphine and midazolam were administered. After 60min of compression, the false aneurysm still had flow. Vascular surgery was consulted for ultrasound-guided thrombin injection.

Question 5 Which of the following statements regarding ultrasound-guided thrombin injection is/are true? A.  It requires direct injection of thrombin into the neck of the false aneurysm B.  It involves simultaneous compression of the false aneurysm C.  It is less painful but less effective than UGCR D.  It works well in anticoagulated patients E.  It is appropriate only for femoral false aneurysms Bovine thrombin solution (1,000 units/mL) was loaded into a small syringe and a 22-gauge spinal needle was attached. Under ultrasound guidance, the needle was placed into the center of the false aneurysm (Fig. 11.2) and 0.3 mL thrombin was injected slowly. Within 15 s, the false aneurysm was thrombosed completely (Fig. 11.3). The procedure was tolerated well. Flow in the underlying artery was preserved and pedal pulses were intact. As the patient was otherwise stable, she was discharged soon afterwards.

Question 6 What are the reported complications of thrombin injection? A.  Anaphylaxis B.  Intra-arterial thrombosis C.  Prolonged urticaria D.  Mad cow disease

108

Fig.11.2  The tip of the needle is visible within the false aneurysm cavity

Fig.11.3  The aneurysm is completely thrombosed 15s after thrombin injection

S.S. Kang

11  False Aneurysm in the Groin Following Coronary Angioplasty

109

11.1  Commentary A false aneurysm after catheterization is suspected when there is a hematoma, especially an enlarging one, at the puncture site hours or days after the procedure. There is often significant ecchymosis of the overlying skin. There may be a bruit, but a continuous bruit is usually associated with an arteriovenous fistula. There may be pain or neuralgia, and the site is often tender. A pulsatile mass is usually palpable, but a simple hematoma overlying the artery may give the same impression. Only a minority of false aneurysms are ­diagnosed unequivocally by physical examination. The diagnosis of a femoral false aneurysm has become very easy with duplex ultrasound. [Q1: B] The incidence of postcatheterization femoral false aneurysms varies from less than 0.5% to more than 5%.1 Some of the factors that increase the likelihood of false aneurysm formation include larger sheaths, longer procedure times, multiple catheter exchanges, and peri- and postprocedure anticoagulation. Puncture of the superficial femoral or deep femoral artery instead of the CFA is found to be associated with higher rates of false aneurysm formation. Direct manual compression after catheter removal is better than compression devices, such as the FemoStop or C-clamp. Patient characteristics that may increase the likelihood of false aneurysm formation include atherosclerosis of the punctured artery, obesity and hypertension. [Q2: B, C, D] The potential complications of untreated false aneurysms are well known. Rupture is the most dramatic and life-threatening complication. Compression of surrounding tissues can cause pain, neuropathy, venous thrombosis, and necrosis of the overlying skin. Thrombosis of, or embolisation into, the femoral artery may occur. Infection of these false aneurysms is less common. Because of these potential outcomes, early surgical repair had been advocated in the past. However, in the 1990s, several series showed that the majority of small false aneurysms will develop spontaneous thrombosis.2–4 It is less likely to occur for larger false aneurysms or in patients who are on anticoagulants. [Q3: C, D] Thrombosis may occur within days, or it may take weeks. Once thrombosis occurs, the false aneurysm is then a simple hematoma that gets resorbed slowly over time. The defect in the artery heals uneventfully in most cases. In 1991, Fellmeth etal.5 described the method of UGCR of postcatheterization femoral false aneurysms and arteriovenous fistulas. The ultrasound transducer is used to apply downward pressure on the neck of the false aneurysm to arrest flow. Pressure is maintained until the blood in the aneurysm becomes thrombosed. After the introduction of UGCR, numerous reports were published verifying the efficacy and overall safety of this procedure.6–9 The typical success rate was between 60% and 90%. There were only a few published complications, including thrombosis of the underlying artery or the femoral vein from the compression, rupture during compression, rupture after successful compression, skin necrosis caused by prolonged pressure on the skin, and vasovagal reactions. Therefore, UGCR was shown to be a good alternative to surgical repair or observation, and most centers made it the initial treatment method. There are several disadvantages to the procedure. It is time-consuming, requiring an average of 30–60min of compression. In most hands, the results are significantly poorer

110

S.S. Kang

for patients on anticoagulants.10 The recurrence rate is about 4–11%, but it is as high as 20% for anticoagulated patients.6 About 10% of patients cannot be treated with UGCR because they have false aneurysms that are not compressible or cannot be compressed without also collapsing the underlying artery, which would increase the chance of arterial thrombosis. For most patients, the compression is painful, and intravenous sedation or analgesia is often necessary. Some patients have required epidural or general anesthesia to allow compression. Applying compression is also very uncomfortable for the operator. [Q4: B, C] Various endovascular treatments have been described for false aneurysms that have failed compression. They usually require catheterization of the feeding artery or false aneurysm from a remote access site. Embolisation coils can be used to occlude the neck or to fill the cavity of the false aneurysm.11,12 Stent grafts can be placed in the femoral artery to exclude the false aneurysm, but late occlusion of the grafts is not uncommon.13 They certainly should not be the initial method of treatment. However, for false aneurysms arising from other, less easily accessible arteries, these techniques may have a role. Because of the shortcomings of UGCR, we developed a new method of treating false aneurysms with ultrasound-guided thrombin injection.14,15 Thrombin causes the cleavage of fibrinogen into fibrin, which then polymerises into a solid. It is the final product of the coagulation cascade, and this reaction occurs naturally whenever blood clots. Thrombin has been used topically for many years to control surface bleeding in the operating room. Our technique is as follows: The ultrasound transducer is centered over the false aneurysm. Thrombin at a concentration of 1,000U/mL is placed into a small syringe, and a 22G spinal needle is attached. The needle is inserted at an angle into the false aneurysm along the same plane as the transducer, and the tip is positioned near the center of the false aneurysm. About 0.5mL thrombin solution is injected slowly into the false aneurysm. Within seconds, thrombosis of the false aneurysm is seen. The procedure is not painful, and patients do not require any analgesia or sedation. We allow patients to get out of bed immediately after treatment, and outpatients are sent home soon after the procedure. So far, we have had great success with this procedure. We have treated 165 false aneurysms. Most (149) developed after groin puncture. There were also false aneurysms in six brachial, three subclavian, two radial, two tibial, one distal SFA, and one superficial temporal arteries, and in one arm arteriovenous fistula. Forty-seven patients were anticoagulated at the time of thrombin injection. It was initially successful in 161 of 165 patients. The other four (all femoral) had partial thrombosis. One of these had complete thrombosis 3days later when brought back for repeat injection. Three had surgical repair. There were early recurrences in twelve patients who had initial successful thrombin injection. Seven were reinjected successfully at the time the recurrence was diagnosed. One had spontaneous thrombosis several days after recurrence was identified. Four had surgical repair. Overall, only 7 of 165 required surgical repair. There were three complications. A brachial artery false aneurysm had injection of thrombin directly into its neck, which caused thrombosis of the brachial artery. A femoral false aneurysm had a relatively large volume of thrombin injected and developed a thombus in the posterior tibial artery. Both of these thromboses resolved after intravenous heparin. A femoral false aneurysm with a short neck that was about 10mm wide had partial thrombosis of the aneurysm. Further injection was not able to thrombose the remaining cavity but instead caused a tail of thrombus to

111

11  False Aneurysm in the Groin Following Coronary Angioplasty

form in the SFA. The patient underwent surgical thrombectomy and repair of the aneurysm. [Q5: D] Our results show that intra-arterial thrombosis after thrombin injection is uncommon. The high concentration of thrombin results in almost immediate conversion of the solution into a solid (thrombus) when it mixes with relatively stagnant blood. Since the neck of the false aneurysm is usually much narrower than the aneurysm cavity, the thrombus cannot enter the artery. As long as the volume of the thrombin injected does not approach or exceed the volume of the false aneurysm, which may result in forcing some of the solution out of the cavity, then the risk of native artery thrombosis should be small. It is likely to be higher when the neck is very wide. Other complications that have been reported include single cases of anaphylaxis16 and prolonged urticaria.17 [Q6: A, B, C] Repeated exposure to bovine thrombin can also lead to development of antibodies to bovine factor V, which may cross-react with autogenous factor V, causing hemorrhagic complications.18 Recently available recombinant human thrombin should be similarly effective in treating false aneurysms with fewer immunologic complications.19 Many others have also had good results with this procedure. In the largest series, the success rate is around 96% and the complication rate less than 2% (Table11.1). Given its simplicity, efficacy, and safety, ultrasound-guided thrombin injection should be considered the initial treatment of choice for postcatheterization false aneurysms. Table11.1  Results of ultrasound-guided thrombin injection Current Khoury20 Paulson21 Maleux22 Mohler23 La Perna24 Total

Cases

Successes (%)

Complications

165 131 114 101 91 70 672

158 (96) 126 (96) 110 (96) 99 (98) 89 (98) 66 (94) 648 (96)

3 3 4 0 1 0 11 (1.6)

References   1. Skillman JJ, Kim D, Baim DS. Vascular complications of percutaneous femoral cardiac interventions. Incidence and operative repair. Arch Surg. 1988;123:1207-1212.   2. Kent KC, McArdle CR, Kennedy B, Baim DS, Anninos E, Skillman JJ. A prospective study of the clinical outcome of femoral pseudoaneurysms and arteriovenous fistulas induced by arterial puncture. J Vasc Surg. 1993;17:125-131.   3. Kresowik TF, Khoury MD, Miller BV, etal. A prospective study of the incidence and natural history of femoral vascular complications after percutaneous transluminal coronary angioplasty. J Vasc Surg. 1991;13:328-333.

112

S.S. Kang

  4. Toursarkissian B, Allen BT, Petrinec D, et al. Spontaneous closure of selected iatrogenic pseudoaneurysms and arteriovenous fistulae. J Vasc Surg. 1997;25:803-808.   5. Fellmeth BD, Roberts AC, Bookstein JJ, etal. Postangiographic femoral artery injuries: nonsurgical repair with US-guided compression. Radiology. 1991;178:671-675.   6. Cox GS, Young JR, Gray BR, Grubb MW, Hertzer NR. Ultrasound-guided compression repair of postcatheterization pseudoaneurysms: results of treatment in one hundred cases. J Vasc Surg. 1994;19:683-686.   7. Hajarizadeh H, LaRosa CR, Cardullo P, Rohrer MJ, Cutler BS. Ultrasound-guided compression of iatrogenic femoral pseudoaneurysm failure, recurrence, and long-term results. J Vasc Surg. 1995;22:425-430.   8. Hertz SM, Brener BJ. Ultrasound-guided pseudoaneurysm compression: efficacy after coronary stenting and angioplasty. J Vasc Surg. 1997;26:913-916.   9. Hood DB, Mattos MA, Douglas MG, et al. Determinants of success of color-flow duplexguided compression repair of femoral pseudoaneurysms. Surgery. 1996;120:585-588. 10. Hodgett DA, Kang SS, Baker WH. Ultrasound-guided compression repair of catheter-related femoral artery pseudoaneurysms is impaired by anticoagulation. Vasc Surg. 1997;31:639-644. 11. Jain SP, Roubin GS, Iyer SS, Saddekni S, Yadav JS. Closure of an iatrogenic femoral artery pseudoaneurysm by transcutaneous coil embolization. Catheter Cardiovasc Diagn. 1996;39:317-319. 12. Pan M, Medina A, Suarez DL, etal. Obliteration of femoral pseudoaneurysm complicating coronary intervention by direct puncture and permanent or removable coil insertion. Am J Cardiol. 1997;80:786-788. 13. Thalhammer C, Kirchherr AS, Uhlich F, Walgand J, Gross CM. Postcatheterization pseudoaneurysms and arteriovenous fistulas: repair with percutaneous implantation of endovascular covered stents. Radiology. 2000;214:127-131. 14. Kang SS, Labropoulos N, Mansour MA, Baker WH. Percutaneous ultrasound guided thrombin injection: a new method for treating postcatheterization femoral pseudoaneurysms. J Vasc Surg. 1998;27:1032-1038. 15. Kang SS, Labropoulos N, Mansour MA, etal. Expanded indications for ultrasound-guided thrombin injection of pseudoaneurysms. J Vasc Surg. 2000;31:289-298. 16. Pope M, Johnston KW. Anaphylaxis after thrombin injection of a femoral pseudoaneurysm: recommendations for prevention. J Vasc Surg. 2000;32:190-191. 17. Sheldon PJ, Oglevie SB, Kaplan LA. Prolonged generalized urticarial reaction after percutaneous thrombin injection for treatment of a femoral artery pseudoaneurysm. J Vasc Interv Radiol. 2000;11:759-761. 18. Ofusu FA, Crean S, Reynolds MW. A safety review of topical bovine thrombin-induced generation of antibodies to bovine proteins. Clin Ther. 2009;31:679-691. 19. Chapman WC, Singla N, Genyk Y, etal. A phase 3, randomized, double-blind comparative study of the efficacy and safety of topical recombinant human thrombin and bovine thrombin in surgical hemostasis. J Am Coll Surg. 2007;205:256-265. 20. Khoury M, Rebecca A, Greene K, etal. Duplex scanning-guided thrombin injection for the treatment of iatrogenic pseudoaneurysms. J Vasc Surg. 2002;35:517-521. 21. Paulson EK, Nelson RC, Mayes CE, Sheafor DH, Sketch MH Jr, Kliewer MA. Sonographically guided thrombin injection of iatrogenic femoral pseudoaneurysms: further experience of a single institution. AJR Am J Roentgenol. 2001;177:309-316. 22. Maleux G, Hendrickx S, Vaninbroukx J, etal. Percutaneous injection of human thrombin to treat iatrogenic femoral pseudoaneurysms: short- and midterm ultrasound follow-up. Eur Radiol. 2003;13:209-212. 23. Mohler ER 3rd, Mitchell ME, Carpenter JP, etal. Therapeutic thrombin injection of pseudoaneurysms: a multicenter experience. Vasc Med. 2001;6:241-244. 24. La Perna L, Olin JW, Goines D, Childs MB, Ouriel K. Ultrasound-guided thrombin injection for the treatment of postcatheterization pseudoaneurysms. Circulation. 2000;102:2391-2395.

Acute Thrombosis

12

Zachary M. Arthurs and Vikram S. Kashyap

A 72-year-old female presents with a 2-week history of abdominal/back pain and lower extremity fatigue. She was evaluated by her physician and diagnosed with lumbosacral neuritis. Initial treatment involved lumbar corticosteroid injections. Secondary to sudden onset lower extremity weakness she presented to the emergency department. Her past history included diabetes, hyperlipidemia, and obesity. In the past month, she had undergone heart catheterization which was significant for multivessel coronary artery disease. She denied any prior surgeries. On examination, her pulse is 75 bpm, and blood pressure is 175/60. Heart sounds reveal a regular rhythm. The abdomen is soft and nontender. She has absent pulses and diminished strength in both lower extremities. Both feet are insensate. There are venous Doppler signals in the feet, but no arterial signals. Creatinine on arrival was 0.9 mg/dL, and white blood cell count was 23,000. Pre-operative CTA demonstrates infrarenal aortic occlusion with bilateral renal infarcts.

Question 1 Native arterial or graft thrombosis can be differentiated from embolic occlusion by the following: A.  The presence of palpable pulses in the contralateral extremity B.  A history of cardiac arrhythmias C.  The location of the occlusion D.  The degree of profound ischemia in the affected extremity E.  All of the above

Z.M. Arthurs () Department of Vascular Surgery, The Cleveland Clinic Foundation, Cleveland, OH, USA G. Geroulakos and B. Sumpio (eds.), Vascular Surgery, DOI: 10.1007/978-1-84996-356-5_12, © Springer-Verlag London Limited 2011

113

114

Z.M. Arthurs and V.S. Kashyap

Question 2 What is the SVS/ISCVS category of limb ischemia in this patient? A.  Category I B.  Category II a C.  Category II b D.  Category III

Question 3 What sign differentiates SVS/ISCVS Category IIa from IIb ischemia? A.  Pulselessness B.  Sensory loss C.  Motor loss D.  Loss of venous doppler signals

Question 4 In acute embolism, the sequence of events is: A.  Pulselessness, pain, pallor, paresthesia, paralysis B.  Paralysis, pain, paresthesia, pulselessness, pallor C.  Pulselessness, pain, pallor, paralysis, paresthesia The patient is taken to the endovascular suite, and based on the preoperative CTA, the left groin is accessed utilizing ultrasound guidance. An angiogram is performed from the sheath that reveals an occluded left iliac system with an isolated common femoral artery. A glide wire is traversed through the iliac system into the aorta. After confirmation of position, an aortogram is performed (Fig.12.1).

Question 5 Treatment options for this patient include which of the following: A.  Aortobifemoral bypass B.  Operative thrombo-embolectomy C.  Extra-anatomic bypass D.  Mechanical thrombectomy, thrombolysis and endovascular intervention E.  Intravenous thrombolysis F.  Anticoagulation with heparin and coumadin

12  Acute Thrombosis

115

Fig.12.1  Aortography via a left femoral approach documents infrarenal aortic and bilateral iliac occlusions

Question 6 After thrombolysis, long-term outcome is predicated on: A.  Unmasking a “culprit lesion” that is treated via either endovascular or surgical means B.  The dose of thrombolytic agent used C.  The duration of thrombolysis D.  The arterial outflow E.  Assuring all acute thrombus is lysed This patient underwent lysis from the left groin with a 20 cm infusion catheter. Thrombolysis (TPA, tissue plasminogen activator, dose = 1mg/h) was performed through a multi-side hole infusion catheter, and the following day, there was significant resolution of thrombus in the aorta/left common iliac system (Fig.12.2). A combination of a hydrophilic wire and catheter was used to cross occlusion in the right common iliac system and gain access to the native femoral system (Fig.12.3). A second infusion catheter was placed through this occlusion, and thrombolytic therapy was continued. After another 24h of therapy, the patient was returned to the endovascular suite. While there was significant improvement, there was still residual thrombus at the origin of the right hypogastric artery and right external iliac artery (Fig.12.4). Thrombolytic therapy was continued another 24h at 0.5mg/h TPA.

116 Fig.12.2  After 24h of thrombolysis, there was significant clot resolution throughout the aorta and common iliac segment. The left hypogastric artery is occluded

Fig.12.3  From the left groin, the right common iliac thrombus has been crossed, and angiography confirms a patent external iliac and femoral system. A second 10-cm infusion catheter was positioned across this region

Z.M. Arthurs and V.S. Kashyap

12  Acute Thrombosis

117

Fig.12.4  After 48h of thrombolysis, the right common iliac system was cleared of thrombus; however, there was still residual thrombus in the external iliac and hypogastric arteries. Thrombolysis was continued in attempt to clear the residual thrombus

Question 7 During thrombolytic therapy for peripheral arterial occlusion, the most frequent compli­ cation is: A.  Pulmonary failure B.  Myocardial infarction C.  Intracranial hemorrhage D.  Vascular access bleeding After 72h of thrombolysis, there was still residual thrombus at the right hypogastric artery and external iliac artery origins (Fig.12.5). Because of concerns over pelvic ischemia and residual thrombus in the left hypogastric artery, efforts were made to preserve the right hypogastric artery. The right hypogastric artery occlusion lesion was traversed from the left groin; the right groin was accessed and a second wire was positioned across the right external iliac artery (Fig.12.6). From this position, opposing self-expanding stents were placed at the origins of both the external and internal iliac arteries restoring perfusion to the right lower extremity without embolization (Figs.12.7 and 12.8). The patient had palpable pedal pulses at completion of the procedure. In the postoperative period, a transesophageal echocardiogram documented cardiac thrombus as the source of aortoiliac embolization. She was discharged on anticoagulation.

12.1  Commentary The etiology of acute limb ischemia can be classified into two groups. Thrombotic events occur in the setting of native arterial disease or bypass graft stenoses. In contrast, embolic phenomena usually occur in normal vessels and tend to lodge at arterial bifurcations.1

118

Z.M. Arthurs and V.S. Kashyap

Fig.12.5  After 72h of thrombolysis, the residual clot remained at the origins of the external iliac artery and hypogastric artery

Fig.12.6  Access was obtained from the right groin, and a wire was positioned retrograde across the external iliac thrombus. From the left groin, the right hypogastric artery was selected

Thrombotic occlusions are thought to represent progression of atherosclerotic disease and occur at sites along the arterial tree and most notably the superficial femoral artery at the adductor canal. In comparison, autologous grafts fail at sites of intimal hyperplasia, or fibrotic valves. Due to preexisting collaterals, native arterial thrombosis seldom presents with the profound ischemia seen with embolic ischemia. The presence of palpable pulses on the contralateral limb and a history of cardiac arrhythmia assist in differentiating acute embolus as opposed to thrombotic occlusions.1

12  Acute Thrombosis

119

Fig.12.7  Two self-expanding stents were deployed in an “opposing fashion” in order to maintain patency of both the external iliac and hypogastric arteries. The right hypogastric artery was treated because of concerns for pelvic ischemia and possible residual left hypogastric thrombus

[Q1: E] All of the factors listed can help in differentiating embolic versus thrombotic occlusions. Often, a definitive diagnosis cannot be made preoperatively. However, identifying an embolic source for acute limb ischemia is helpful both for the acute and long-term management of the patient. Clinical classification and diagnosis of acute occlusion of the lower extremity is based on the symptoms of the patient. The severity of symptoms is associated with the extent of the occlusion and the presence of pre-existing vessels. Patients with thrombotic occlusions from underlying disease of the SFA at the adductor canal may only experience worsening claudication while embolic events are usually associated with rapid onset and severe ischemia because of the lack of preexisting collateral flow. Limb ischemia has been classified into three categories by an SVS/ISCVS ad hoc committee, based on severity of ischemia.2 Category I limbs are viable, not immediately threatened and have no motor or sensory loss. There are clearly audible arterial Doppler signals in the foot. Category II includes threatened limbs where salvage may be possible with timely intervention. Importantly, this category is divided into two subgroups, a and b which distinguish the time interval necessary for treatment. Group IIa require prompt treatment whereas Group IIb need immediate therapy to prevent amputation. In Group

120

Z.M. Arthurs and V.S. Kashyap

Fig.12.8  Completion imaging documents rapid flow through the aortoiliac system without any residual thrombus. Bilateral lower extremity runoff documented good runoff without embolization

II, audible venous Doppler signals are present, but there is no arterial signal in the foot. Group IIa patients have minimal sensory loss and have no motor loss. However, patients with Group IIb ischemia, have muscle weakness, and sensory loss encompasses more than the toes. Category III is characterized by irreversible ischemia with profound and permanent neuromuscular damage where amputation may be the only recourse. [Q2: C] This patient has Category IIb limb ischemia characterized by lack of distal arterial signals in the foot, sensory loss, and motor weakness. [Q3: C] Motor loss separates Category IIa from IIb ischemia and determines the urgency on which to proceed to revascularization. The sequence of clinical events in patients with lower extremity ischemia is often predictable. [Q4: A] Most patients with acute ischemia, especially from of an embolic nature, will have pulselessness followed by pain and pallor. Paresthesia indicates sensory nerve ischemia and occurs usually from 1 to 3h after the onset of acute ischemia. Paralysis indicates motor nerve damage that is often irreversible. In the setting of acute ischemia without collateral flow, paralysis occurs approximately 6h after the onset of ischemia.3 Any motor dysfunction should be seen as a worrisome sign and should prompt urgent intervention. Poikilothermia indicates that the foot or limb has approximated ambient temperature. In these irreversible cases (Category III), amputation may be the only option and often has to be done quickly to avoid systemic complications. Both the diagnosis and localization of acute arterial occlusion is based upon the findings on physical exam and imaging studies. A “waterhammer” pulse signifies outflow obstruction, as observed with a common femoral embolus. By contrast, calcified vessels

12  Acute Thrombosis

121

are common with thrombosis from underlying atherosclerotic disease. Multiple options are available for localization of the occlusion. Noninvasive testing with segmental pressures, pulse volume recording and measurement of the ankle brachial indices can provide a baseline study for comparison after treatment. Both vertical and horizontal pressure gradients of 30mmHg or more in the lower extremity can accurately identify the site of occlusion. Duplex ultrasonography can also be utilized to examine the femoral and popliteal vessels and localize area of occlusion. Other causes such as thrombosed popliteal aneurysm can be easily diagnosed in this manner. MRA and CTA are emerging as noninvasive techniques for arteriographic imaging and localization of thrombosis. However, angiography remains the gold standard for localization of arterial occlusion. As importantly, angiography allows percutaneous access to the site of thrombosis and an array of treatment options for restoring blood flow to the limb. Treatment for limb ischemia has evolved over the past two decades with advances in both pharmacologic therapy and endovascular options. [Q5: B, C, D] In patients with acute limb ischemia secondary to iliac occlusion, operative thrombectomy of the occluded iliac system may be feasible. In the setting of profound ischemia and a diseased iliac artery precluding successful thrombectomy, extra-anatomic bypass can be performed to provide expeditious blood flow into the ischemic limb. In these cases, either femoral-femoral bypass or axillo-femoral bypass can be contemplated depending on the inflow source. Aortofemoral bypass is a very durable option for patients with chronic occlusion and chronic ischemia of the limb. However, in patients with acute ischemia ill-prepared for major surgery, proceeding with direct reconstruction with the aorta as the inflow is sometimes hazardous. Multiple endovascular devices are available in the setting of acute thrombosis. Percutaneous mechanical thrombectomy with thrombolysis either via the power-pulse technique or via a standard infusion often quickly resolves the acute ischemia. Continued thrombolytic infusion is required for complete resolution of thrombus. Often, a “culprit” lesion will be unmasked by dissolving all of the acute thrombus, allowing percutaneous treatment of the offending lesion. Systemic thrombolytic therapy has been used to treat peripheral arterial occlusions, but results have been disappointing owing to a significant incidence of bleeding complications. Currently, systemic therapy is usually used for venous thromboembolic states. Regional intravascular infusion of the lytic agent avoids some of the systemic complications and is largely used for peripheral arterial thromboses and graft occlusions. Because a systemic lytic state may occur with prolonged regional intravascular thrombolytic therapy, patient selection is critical. Absolute contraindications include active internal bleeding, recent surgery or trauma to the area to be perfused, recent cerebrovascular accident, or documented left heart thrombus.3 Relative contraindications include recent surgery, gastrointestinal bleeding or trauma, severe hypertension, mitral valve disease, endocarditis, hemostatic defects, or pregnancy. [Q6: A, D, E] Several multicenter trials have examined groups of patients treated with surgical therapy or thrombolysis. The Rochester trial randomized patients to surgery or thrombolysis and demonstrated a lower mortality in the thrombolysis group.4 Following successful thrombolysis, unmasked “culprit lesions” were treated with angioplasty or surgery of a lesser magnitude, thereby reducing the severity of the intervention and overall morbidity. The finding of a lesion that precipitated the thrombosis is critical to avoiding re-thrombosis. The STILE trial (Surgery versus Thrombolysis for Ischemia of the Lower

122

Z.M. Arthurs and V.S. Kashyap

Extremity) compared optimal surgical therapy to intraarterial catheter-directed thrombolysis for native arterial or bypass graft occlusions.5,6 Stratification by duration of ischemic symptoms revealed that patients with ischemia of less than 14 days duration had lower amputation rates with thrombolysis and shorter hospital stays, while patients with ischemia for longer than 14 days who were treated surgically had less ongoing or recurrent ischemia and trends toward lower morbidity. At 6 months, amputation-free survival was improved in patients with acute ischemia treated with thrombolysis, but patients with chronic ischemia had lower amputation rates when treated surgically. Fifty-five percent of patients treated with thrombolysis had a reduction in magnitude of their surgical procedure. Of note, no difference was seen between the use of rt-PA and urokinase.5 A multicenter, randomized, prospective trial comparing thrombolysis to surgery for acute lower extremity ischemia of less than 14 days duration has been performed. The Thrombolysis or Peripheral Arterial Surgery trial (TOPAS) randomized 757 patients to surgery or thrombolytic therapy.7 The most effective dose for recombinant urokinase was determined to be 4,000U/min with complete thrombolysis in 71% (mean duration of therapy 24 ± 0.8h) of patients. After successful thrombolytic therapy, either surgical or endovascular intervention was performed on the lesion responsible for the occlusion if found. When compared to the surgical arm, the 1-year limb salvage rates and mortality were not statistically different. However, although no statistical differences between the two groups were seen with respect to amputation-free survival, thrombolysis was associated with a reduction in the number and magnitude of open surgical interventions over a 1-year follow up period. Perhaps, unlike thrombolysis in coronary or venous systems, dissolution of the larger peripheral arterial thrombi requires direct infusion of thrombolytic agent into the clot. The thrombosed artery or bypass graft must be accessed with a wire, followed by placement of an infusion system into the thrombus. There are multiple dosing regimens for urokinase (UK), t-PA and other thrombolytic agents. A plethora of strategies for thrombolysis have been used and are described in a consensus document.8 In this comprehensive review, 33 recommendations were made by a panel of experienced hematologists, radiologists, and vascular surgeons from North America and Europe. Of note, over 40 dosage schemes were reviewed and described for thrombolytic infusion. This included strategies of continuous versus stepwise infusion, bolusing or lacing the clot, and intraoperative thrombolysis. The most popular strategies included using UK 4,000u/min for 4h, and then decreasing to 2,000 u/min for a maximum of 48 h, t-PA at a dose of 1 mg/h and lacing the clot to increase thrombolytic efficiency. Our preferred current technique is to use low-dose t-PA (0.5–1.0 mg/h) after initial percutaneous mechanical thrombectomy. Low-dose heparin (300–400 u/h) is infused via the arterial sheath side arm to prevent pericatheter thrombosis, but full anticoagulation is avoided. Following successful thrombolysis, any unmasked lesion can be addressed with balloon angioplasty and stenting or with an open surgical procedure. Even when a surgical procedure is necessary, it can usually be performed electively, in a well-prepared patient and is often of a lesser magnitude than what would have been required without thrombolysis. Thrombolytic therapy is an effective option for selected patients with acute thrombotic occlusion.

12  Acute Thrombosis

123

[Q7: D] Both pulmonary failure and myocardial infarction are more common with surgical revascularization and relatively infrequent complications in patients treated with thrombolytic therapy. Bleeding complications are the most frequent complications associated with thrombolytic therapy, and these are typically related to access bleeding that requires transfusion. The access-related bleeding rates ranged from 7% to 12.5% in the Rochester, STILE, and TOPAS trials compared to intracranial hemorrhage rates ranging from 0.5% to 2.5%. STILE found low fibrinogen levels to be associated with bleeding complications, while TOPAS found therapeutic heparin to increase the risk of complications. It is important to note that these trials limited therapy to 24–48h for fear of bleeding complications. Bleeding, unexplained drop in hemoglobin, neurological changes, or fibrinogen levels falling below 100 mg/dL usually require cessation of thrombolytic therapy. The patient in this case represents an aggressive approach to extensive clot burden throughout the aortoiliac segment. In this example, therapy was extended to 72h, longer than our usual arterial thrombolytic case, in order to fully dissolve all of the thrombus.

References 1. Blaisdell FW, Steele M, Allen RE. Management of acute lower extremity arterial ischemia due to embolism and thrombosis. Surgery. 1978;84:822-834. 2. Rutherford RB, Baker JD, Ernst C, et al. Recommended standards for reports dealing with lower extremity ischemia: Revised version. J Vasc Surg. 1997;26:517-538. 3. Kashyap VS, Quinones-Baldrich WJ. Principles of thrombolytic therapy. In: Rutherford RB, ed. Vascular Surgery. 5th ed. Philadelphia, PA: W.B. Saunders; 2000:457-475. 4. Ouriel K, Shortell CK, DeWeese JA, etal. A comparison of thrombolytic therapy with operative revascularization in the initial treatment of acute peripheral arterial ischemia. J Vasc Surg. 1994;19:1021-1030. 5. The STILE Investigators. Results of a prospective randomized trial evaluating surgery versus thrombolysis for ischemia of the lower extremity, The STILE Trial. Ann Surg. 1994;220: 251-268. 6. Weaver F, Camerato A, Papanicolau G, etal. Surgical revascularization versus thrombolysis for non-embolic lower extremity native artery occlusions: results of a prospective randomized trial. The STILE Investigators. J Vasc Surg. 1996;24:513-523. 7. Ouriel K, Veith FJ, Sasahara AA. A comparison of recombinant urokinase with vascular ­surgery as initial treatment for acute arterial occlusion of the legs. N Engl J Med. 1998;338: 1105-1111. 8. Working Party on Thrombolysis in the Management of Limb Ischemia. Thrombolysis in the management of lower limb peripheral arterial occlusion – a consensus document. J Vasc Interv Radiol.  2003 (Sept);14(9 Pt 2):S337-S349.

Part II Acute Ischemia

Arterial Embolism

13

Andre Nevelsteen†

A 65-year-old man presented with acute severe pain in his right leg. Medical history revealed non-insulin-dependent diabetes mellitus for 3 years and a myocardial infarction (MI) some 5 years ago. The pain in the right leg developed suddenly over 6 hours without associated trauma and became worse over time. On admission, the right leg looked pale distally from the level of the knee. There was loss of light touch sensation on examination of the foot. The patient had difficulties in wiggling the toes. Plantarflexion and dorsiflexion of the toes were still possible. Palpation of the calf showed soft but tender muscles. Clinical examination of the abdomen showed no abnormalities. There was no pulsating mass. Irregular but bounding pulsations were felt in the right femoral artery. Popliteal artery and tibial artery pulsations were absent. Normal pulsations were felt in the left popliteal and posterior tibial artery.

Question 1 What is the aetiology of arterial embolism? A.  The aetiology of arterial embolism is most frequently unknown. B.  The most frequent cause of arterial embolism is cardiac valve destruction by rheumatic heart disease or endocarditis. C.  The most frequent cause of arterial embolism is atrial fibrillation in association with atherosclerotic heart disease. D.  Deep venous thrombosis might represent a rare cause of arterial embolism. E.  Arterial embolism is most frequently seen in the presence of increased blood viscosity. With the diagnosis of acute arterial ischaemia in mind, a full dose of intravenous heparin was administered immediately.

A. Nevelsteen Department of Vascular Surgery, University Hospital Gasthuisberg, Leuven, Belgium G. Geroulakos and B. Sumpio (eds.), Vascular Surgery, DOI: 10.1007/978-1-84996-356-5_13, © Springer-Verlag London Limited 2011

127

128

A. Nevelsteen

Question 2 What is the place of heparin in the treatment of arterial embolism? A.  Heparin can dissolve an arterial embolus, avoiding the need for subsequent operation. B. Heparin will avoid subsequent arterial thrombosis, which can complicate treatment of arterial embolism. C.  Heparin will avoid subsequent arterial thrombosis, which can complicate treatment of arterial embolism. In addition, heparin will prevent recurrent emboli. D. The use of heparin is contraindicated since it may lead to fragmentation of an arterial embolism and induce microembolisation in the peripheral arteries. A chest film X-ray showed no abnormalities. Electrocardiogram (ECG) revealed atrial fibrillation and signs of an old MI. Laboratory studies were normal. Duplex examination showed a thrombotic occlusion of the right femoral bifurcation and the superficial femoral artery. A weak flow sign was present in the popliteal artery. The tibial arteries were not visualised.

Question 3 The preferred treatment of arterial embolism is: A.  Local excision of the vessel and reconstruction with interposition graft. B.  Continued heparinisation and wait and see. C.  Simple Fogarty catheter embolectomy with peroperative angiographic control. D.  Simple Fogarty catheter embolectomy, but percutaneous aspiration thromboembolectomy might be a good alternative in selected cases. After placement of a central venous catheter, the patient was taken to the operating theatre and the right femoral bifurcation was exposed under local anaesthesia. A transverse arteriotomy confirmed complete thrombotic occlusion of the femoral bifurcation. There was good inflow. Thrombi were removed from the femoral bifurcation, and pulsatile backflow was obtained from the profunda femoris artery. Multiple thrombi were removed from the superficial femoral artery and the popliteal artery after several passages of Fogarty embolectomy catheters numbers 3 and 4. Intraoperative angiography showed good patency of the superficial, popliteal and peroneal arteries. The anterior tibial artery was completely occluded. The posterior tibial was patent in its first portion but occluded distally. A small catheter was inserted into the popliteal artery, and 350,000 units of urokinase were infused as a dripping infusion over 30 min. Repeated angiography showed further clearance of the posterior tibial artery to the level of the ankle joint. The anterior tibial artery was still occluded. It was decided to accept the situation. The arteries were flushed with a diluted heparinised saline solution, and the transverse arteriotomy was closed with the aid of a Dacron patch. Sodium bicarbonate was administered intravenously before reperfusion.

13  Arterial Embolism

129

Question 4 Reperfusion syndrome after arterial embolectomy: A.  Will never be seen after peripheral but only after aortic embolism. B.  Cannot be prevented medically. C.  Will be prevented by early ambulation. D.  Is induced by metabolic acidosis and myoglobinuria. Postoperatively, the foot was well vascularised and the patient was able to wiggle his toes almost normally. Pulsations were felt in the posterior tibial artery. Intravenous heparin was continued. Brisk diuresis was maintained with mannitol and alkalisation of the urine. Repeated laboratory studies showed no evidence of acidosis or hyperkalaemia.

Question 5 Fasciotomy: A.  Has become obsolete and swelling of the limb should be treated by elevation and bed rest. B.  Is best routinely performed in any patient, treated for arterial embolism of the lower limbs. C.  The indication to fasciotomy needs to be based on objective parameters such as the ­presence of reperfusion syndrome and postoperative compartmental pressure measurements. D.  In daily practice, the indication for fasciotomy is most frequently based on individual preference and clinical feeling. Six hours postoperatively, the patient developed significant limb swelling with augmentation of pain, venous hypertension and sensory impairment of the foot. A perifibular fasciotomy to decompress all four compartments was performed under general anaesthesia. Afterwards, the swelling subsided and the fasciotomy wound was closed in a delayed primary fashion after 1 week.

Question 6 With the pre- and peroperative diagnosis in mind: A.  The patient should be placed under antiplatelet therapy postoperatively in order to prevent another episode of embolism. B.  Heparin and oral anticoagulants remain the treatment of choice during the postoperative period. C.  Subsequent investigation with regard to the source of the embolus is not necessary, because this will not change the medical treatment. D.  Postoperative investigation with regard to the source of embolism can be limited to cardiac examinations such as echocardiography and Holter monitoring.

130

A. Nevelsteen

Abdominal ultrasound performed postoperatively showed atheromatosis of the abdominal aorta but no aneurysmal dilatation. Transthoracic and transoesophageal echocardiography revealed no ventricular aneurysm or intracardiac thrombi. Holter monitoring for 24 h confirmed atrial fibrillation. Pathological examination of the retrieved emboli was compatible with ordinary thrombotic material. Cultures were negative. The problem of atrial fibrillation was handled medically. Oral anticoagulation was initiated, and the patient was discharged after 10 days. Six months later, there were no repeat episodes of acute ischaemia.

13.1  Commentary Acute ischaemia due to arterial embolism represents a limb-threatening event. Although the carotid or intracranial vessels may be involved in a minority of the cases, the upper or lower extremities are involved in 70–80% in most series of arterial embolisation.1 The lower extremity is involved five times as frequently as the upper extremity, and the sites of embolic occlusion are most often related to major arterial bifurcations. The common femoral bifurcation is the most frequent site of embolic occlusion, usually noted in 30–50% of all cases.2 In total, the femoral and popliteal arteries are involved more than twice as often as the aorta. The heart is by far the predominant source of arterial emboli, seen in 80–90% of cases.3 Atrial fibrillation is present in approximately 70% of patients. Previously, it was most frequently the reflection of rheumatic heart disease. Since the incidence of rheumatic heart disease has declined steadily over the last 50 years, atrial fibrillation is now associated most frequently with atherosclerotic heart disease. MI is the second common cause of peripheral embolisation. Left ventricular mural thrombus occurs in 30% of acute transmural infarcts. Clinically evident embolism is seen in only 5% of these patients.4 One should be aware, however, that silent MI may be present in up to 10% of patients with peripheral emboli, and that embolisation may be the presenting symptom of an acute infarction. Apart from the acute period, MI may also cause emboli after longer intervals. This is usually due to areas of hypokinesis or ventricular aneurysm formation. Although most emboli occur within 6 weeks of MI, much longer intervals may be noted. Other cardiac diseases are associated less frequently with peripheral emboli. Thromboemboli can, however, arise from prosthetic cardiac valves or from vegetations on the mitral or aortic valve leaflets. Endocarditis should certainly be ruled out. Finally, intracardiac tumours, such as atrial myxoma, may also give rise to clinically evident embolic events. Non-cardiac sources of peripheral emboli are noted less frequently. Major emboli may arise from aneurysms of the aorta or less frequently from the femoropopliteal vessels.5 With upper-extremity emboli, one should be aware of unsuspected thoracic outlet syndrome and aneurysmal deformation of the subclavian artery. Paradoxical emboli might be seen with deep venous thrombosis in association with a patent foramen ovale. Primary or secondary lung tumours might invade the pulmonary veins, causing tumour emboli. Finally, apart from rare causes such as foreign body embolisation, it should be recognised that the source of embolisation will remain inapparent in some 10% of patients.2 [Q1: C, D]

13  Arterial Embolism

131

The diagnosis of acute ischaemia caused by arterial embolism is usually straight-forward. The most typical signs are characterised by the “five Ps”: pulselessness, pain, pallor, paraesthesia and paralysis. The level of occlusion is determined by the presence or absence of palpable pulses. Once the diagnosis of acute arterial ischaemia has been made, 5,000 units of heparin are administered intravenously. This is not meant as effective treatment but it prevents the propagation and fragmentation of the thrombus. Concomitant venous thrombosis, which can occur with prolonged severe arterial ischaemia, might also be avoided. Heparin administration allows time for diagnosis, evaluation and, if necessary, treatment of cardiac disturbances. [Q2: B] Fogarty catheter embolectomy remains the treatment of choice in most patients with peripheral embolisation.6 The procedure is usually carried out under local anaesthesia and is effective in cases of major emboli. All retrieved emboli should be sent for pathological and microbiological examination. The operative result should be checked by intraoperative fluoroscopy or angioscopy. Remaining thrombi in the distal vessels can be approached directly or by intraoperative thrombolysis.7 Thrombolytic therapy or percutaneous aspiration thromboembolectomy (Fig.13.1) may be used as alternatives to Fogarty catheter embolectomy in selected cases with no motor dysfunction or profound sensory loss.8, 9 [Q3: C, D] All patients undergoing revascularisation of an acutely ischaemic limb are at risk of ischaemia reperfusion syndrome. This was first emphasised by Haimovici,10 described under its most grave form as the myonephropathic-metabolic syndrome. This reperfusion syndrome is the consequence of muscular hypoxia and the associated metabolic changes. A prolonged period of ischaemia results in accumulation of potassium, lactic acid, myoglobin and other cellular enzymes, leading to a significant fall in blood pH due to anaerobic metabolism, paralysis of the sodium potassium cellular pump and rhabdomyolysis.11 Acute washout of these products may lead to hyperkalaemia and metabolic acidosis, resulting in myocardial depression or dysrhythmias. Myoglobin and other products of skeletal muscle breakdown can precipitate within the kidney and result in acute renal failure. Myoglobinuria is the first sign. [Q4: D] These problems should be anticipated with bicarbonate and/or calcium intravenously just before reperfusion. Induction of forced diuresis with mannitol and alkalisation of the urine might avoid acute renal failure. In addition, mannitol also acts as a scavenger of oxygen-derived free radicals, which are an important intermediary in ischaemia reperfusion injury.12, 13 It is clear, therefore, that the patient should be monitored carefully postoperatively with regard to electrolyte changes, development of metabolic acidosis and urinary output. Another problem following revascularisation of an acute ischaemic limb might be significant limb swelling. This may result in secondary muscle or nerve injury, venous compression, further oedema and compartment syndrome, leading to arterial compression and secondary ischaemia. To avoid this, the surgeon might prefer to perform a fasciotomy in conjunction with the embolectomy procedure.14 Alternatively, the extremity can be assessed immediately and at regular intervals postoperatively for evolving compartment syndrome. As described in different textbooks, there are several ways of performing an adequate fasciotomy. The most important point here is that all four compartments should be decompressed. Although concomitant fasciotomy can be preferable in some cases of prolonged acute ischaemia, the more conservative approach might avoid unnecessary fasciotomy and

132

A. Nevelsteen

Fig.13.1  (a) Embolic occlusion of the left popliteal artery; treatment consisted of percutaneous aspiration thromboembolectomy. (b) Normal patency of the popliteal, anterior tibial and peroneal arteries

unaesthetic scars. Since a Fogarty catheter embolectomy can easily be carried out under local anaesthesia, this wait-and-see approach eliminates the need for systematic general anaesthesia, particularly for patients in a poor general condition.15 Despite the fact that the value of postoperative compartmental pressure measurements has been documented by several teams,16, 17 the decision regarding subsequent fasciotomy is most frequently based upon individual preferences and prior clinical experience. [Q5: D] Every effort should be made in the postoperative period to minimise the incidence of recurrent emboli. The patient should be treated with heparin or oral anticoagulants until the source of the embolus has been taken care of. [Q6: B] If extensive investigation fails to show any correctable source, then long-term anti-coagulation is indicated, except in the case of major contraindications.

13  Arterial Embolism

133

References   1. Panetta T, Thompson JE, Talkinton CM, Garrett WV, Smith BL. Arterial embolectomy: a 34-year experience with 400 cases. Surg Clin North Am. 1986;66:339.   2. Thompson JE, Sigler L, Raut PS, Austin DJ, Patman RD. Arterial embolectomy: a 20-year experience. Surgery. 1970;67:212-220.   3. Mills JL, Porter JM. Basic data related to clinical decision making in acute limb ischemia. Ann Vasc Surg. 1991;5:96.   4. Keating EC, Gross SA, Schlamowitz RA. Mural thrombi in myocardial infarctions. Am J Med. 1983;74:989.   5. Reber PU, Patel AG, Stauffer E, Muller MF, Do DD, Kniemeyer HW. Mural aortic thrombi: an important cause of peripheral embolization. Vasc Surg. 1999;30:1084-1089.   6. Abbott WM, Maloney RD, McCabe CC, Lee CE, Wirthlin LS. Arterial embolism: a 44 year perspective. Am J Surg. 1982;143:460-464.   7. Beard JD, Nyamekye I, Earnshaw JJ, Scott DJ, Thompson JF. Intraoperative streptokinase: a useful adjunct to balloon-catheter embolectomy. Br J Surg. 1993;80:21-24.   8. Heymans S, Vanderschueren S, Verhaeghe R, etal. Outcome and one year follow-up of intraarterial staphylokinase in 191 patients with peripheral arterial occlusion. Thromb Haemost. 2000;83:666-671.   9. Sniderman KW, Kalman PG, Quigley MJ. Percutaneous aspiration embolectomy. J Cardiovasc Surg (Torino). 1993;34:255. 10. Haimovici H. Muscular, renal and metabolic complications of acute arterial occlusions: myonephropathic-metabolic syndrome. Surgery. 1979;85:461. 11. Fischer RD, Fogarty TJ, Morrow AG. Clinical and biochemical observations of the effect of transient femoral artery occlusion in man. Surgery. 1970;68:323. 12. Rubin BB, Walker PM. Pathophysiology of acute skeletal muscle injury: adenine nucleotide metabolism in ischemic reperfused muscle. Semin Vasc Surg. 1992;5:11. 13. Pattwell D, McArdle A, Griffiths RD, Jackson MJ. Measurement of free radical production by invivo microdialysis during ischemia/reperfusion injury to skeletal muscle. Free Radic Biol Med. 2001;30:979-985. 14. Padberg FT, Hobson RWII. Fasciotomy in acute limb ischemia. Semin Vasc Surg. 1992;5:52. 15. Rush DS, Frame SB, Bell RM, Berg EE, Kerstein MD, Haynes JL. Does open fasciotomy contribute to morbidity and mortality after acute lower extremity ischemia and revascularization? J Vasc Surg. 1989;10:343-350. 16. Whitesides TE, Heckman MM. Acute compartment syndrome: update on diagnosis and treatment. J Am Acad Orthop Surg. 1996;4:209-218. 17. Janzing HMJ. The acute compartment syndrome, a complication of fractures and soft tissue injuries of the extremities. A clinical study about diagnosis and treatment of the compartment syndrome. Doctoral thesis. Leuven University; 1999.

Blast Injury to the Lower Limb

14

Paul H.B. Blair, Adrian K. Neil, and Christopher T. Andrews

A 40-year-old male was admitted to the emergency room approximately 1.5 h after sustaining a blast injury to both lower limbs. He had been resuscitated at his local accident and emergency department prior to transfer. On arrival, his pulse was 120 bpm and his blood pressure 80/40 mm Hg. Examination revealed that the patient had sustained significant blast injuries to both lower limbs with no obvious torso injuries. The left leg had sustained neurovascular damage above and below the knee with concomitant bone and soft tissue injury; there was no tissue perfusion below the knee. On the right side there was a large wound in the thigh extending anteriorly to the knee joint with profuse bleeding; bony fragments could be seen in the wound and the right foot was pale with no palpable pulses and slight reduction in sensation.

Question 1 The priorities for the care of this patient include: A. Secure an airway, commence oxygen therapy and obtain adequate intravenous (IV) access. B.  Complete a full survey of the patient before transferring for further management. C.  Wait for blood result before deciding on transfer out of the emergency room. D. Transfer the patient to theatre for definitive management during primary resuscitation. E.  Discuss treatment options with relatives.

Question 2 Which of the following are “hard” signs of vascular injury? A.  Limb pain. B.  Absence of pulses. C.  Pallor or cyanosis. D.  Cool to the touch. E.  Bruit or thrill. P.H.B. Blair () Vascular Surgery Unit, Royal Victoria Hospital, Belfast, UK G. Geroulakos and B. Sumpio (eds.), Vascular Surgery, DOI: 10.1007/978-1-84996-356-5_14, © Springer-Verlag London Limited 2011

135

136

Adrian K. Neil et al.

Question 3 Which of the following statements relating to angiography are true? A.  Angiography should be performed in all patients to target surgery. B. Angiography may be a useful tool in trauma patients with no hard signs of vascular injury. C.  Angiography is reserved for stable patients. D.  Angiography should only be performed in a radiology department. E. The patient’s pre-morbid condition should not influence the decision to perform angiography.

Question 4 For how long will the lower limb tolerate ischemia? A.  20–30 min B.  90–120 min C.  6–8 h D.  16–20 h E.  24–36 h The patient was resuscitated as per advanced trauma life support (ATLS) protocol. Supplementary oxygen was administered in addition to obtaining additional IV access. Pressure dressings were applied to the open wounds and further assessment revealed an injury to the patient’s right hand; no other significant injuries were present. The patient was transferred to the operating theatre.

Question 5 What are the primary aims of surgery in such a case? A.  To control life-threatening haemorrhage. B.  To prevent end-organ ischaemia. C.  To restore vascular continuity. D.  To preserve limb function. E.  To detect occult injuries.

Question 6 What factors will influence the decision to perform an amputation? A.  Patient’s age B.  Mechanism of injury

14  Blast Injury to the Lower Limb

137

C.  Time to treatment D.  Degree of contamination E.  All of the above

Question 7 Which of the following statements about complex vein repair are true? A.  Complex vein repair should never be undertaken in the trauma patient. B.  Complex vein repair should only be performed in the absence of major arterial injury. C.  Complex vein repair should be used to improve venous return in unstable patients. D.  Complex vein repair may prevent long-term limb dysfunction. E.  Intraluminal venous shunting is an acceptable intraoperative temporising measure. In the operating theatre, under general anaesthesia, the patient was placed in the supine position. The lower abdomen and both legs were prepared and draped widely and IV broad spectrum antibiotics were administered. Closer examination revealed that the left leg had sustained extensive injuries. The foot and distal calf were cold, pale and mottled. There was a compound injury to the left femur and tibia with complete disruption of the superficial femoral artery, superficial femoral vein and extensive injury to the sciatic nerve. It was decided that primary amputation of the left limb was required. On examination of the right leg there was complete disruption of the distal superficial/popliteal artery, a ragged laceration of the popliteal vein and significant bruising to branches of the sciatic nerve. There was a shrapnel injury to the right hand involving the thumb and middle finger. Immediate surgical steps were as follows: (a) a proximal thigh tourniquet was placed on the left leg to arrest haemorrhage prior to formal amputation. The laceration to the right lower leg was then extended distally to facilitate exposure of the neurovascular structures. Control of the superficial femoral and below-knee popliteal artery was obtained and a careful distal embolectomy performed. A Javid shunt was then placed between the right superficial femoral artery and right below-knee popliteal vessel (Fig.14.1). Significant bleeding from a large defect in the popliteal vein occurred following shunt insertion; this was repaired using a lateral suture. The long saphenous vein was harvested from the left leg, prior to performing above-knee amputation. While the left above-knee amputation was being performed, the orthopaedic surgeons carefully assessed the right lower limb and placed a temporary fixation device traversing the right knee joint (Fig.14.2). Having obtained bony stability, with an external fixator device, the temporary intraluminal shunt was removed and a definitive bypass performed using reversed left long saphenous vein graft. Formal fasciotomy was performed of the right lower leg using a standard lateral and medial approach; distal pulses were confirmed in the right foot. Further debridement of necrotic muscle was performed and the wound on the medial aspect was partially closed; the anterolateral wounds were debrided and irrigated, as were the fasciotomy sites, with sterile dressings being applied to both.

138

Adrian K. Neil et al.

Fig.14.1  Extended wound, medial aspect of right leg with a temporary intraluminal shunt between superficial femoral and below-knee popliteal arteries

Fig.14.2  A multidisciplinary approach. Bony stabilization of right leg (after temporary intraluminal shunt placement) by the orthopaedic surgeons, simultaneous with left above-knee amputation by the vascular surgeons

Question 8 In the absence of obvious haemorrhage, when is it appropriate to reinspect the wounds in the postoperative period? A.  1–2 h B.  4–6 h C.  12–16 h D.  24–48 h E.  5+ days Postoperatively the patient was transferred to the intensive care unit where the right limb was elevated to reduce swelling. The right foot was left exposed to allow access for pedal pulses. Broad spectrum IV antibiotics were continued in addition to standard prophylaxis for deep vein thrombosis, and urine was checked for myoglobinuria. The patient was returned to the operating theatre within 48 h for wound inspection and change of dressing. Eventually skin coverage of the right limb was obtained using a combination of split skin grafting and healing by delayed primary intention. Over the next few months the patient

14  Blast Injury to the Lower Limb

139

required complex orthopaedic surgery including the use of an Ilizarov frame device (Fig.14.3). He was fitted with an above-knee prosthesis for his left leg and is now fully independent (Fig.14.4).

Fig.14.3  Recovery. Healed traumatic and fasciotomy wounds after skin grafting; Ilizarov frame still in place

Fig.14.4  Rehabilitation. An excellent result for limb salvage (right leg) and learning to function with a prosthesis (left)

140

Adrian K. Neil et al.

14.1  Commentary Lower limb injuries, due to penetrating trauma, can be devastating and occasionally may distract the clinician from less obvious but potentially life-threatening injuries to the head, neck and torso. It is important that some form of resuscitation protocol is followed such as the ATLS system to detect less obvious injuries. Time is of the essence when managing vascular injuries. While delays rarely occur in patients with obvious haemorrhage, it is the prompt instigation of life-saving measures and ongoing diagnosis in parallel with transfer to the operating theatre for definitive care that reduces morbidity and mortality. [Q1: A, D] The clinical manifestations of vascular injury have traditionally been divided into “hard” and “soft” signs (Table14.1). [Q2: B, E] In general, preoperative arteriography may be used in the following situations: (1) to confirm the site and extent of vascular injury in stable patients whose clinical signs and symptoms are equivocal; and (2) to exclude vascular injury in patients with no hard signs, but who are considered to be at risk because of the proximity of the injury. The majority of patients with penetrating extremity trauma and the presence of a single hard sign should be transferred directly to the operating theatre. Possible exceptions to this rule include stable patients with multiple levels of injury, extensive bone or soft tissue injury, blast or shotgun injuries, potential injuries to the subclavian or axillary arteries and the pre-existence of peripheral vascular disease. Some centres report excellent results with emergency room angiography1 while recent advances in endovascular technique facilitate high-quality imaging in the operating theatre. [Q3: B, C] Inadequate tissue perfusion due to major vessel disruption is aggravated by hypovolemic shock and associated bone and soft tissue injury. The resulting fall in tissue pO2 increases capillary membrane permeability, with increased exudation of fluid into the interstitial space. Compromised muscle fibres swell within the fascial compartments, causing further resistance to blood flow, and swelling becomes traumatic when arterial repair and restoration of flow brings about reperfusion injury. The degree of reperfusion injury depends on the duration of ischaemia, and is mediated by the generation of free radicals, activation of neutrophils, and production of arachidonic acid metabolites. Eventually, the microvascular bed of the extremity may undergo widespread thrombosis.2 It is generally accepted that a warm ischaemia time of more than 6–8 h makes limb survival unlikely. [Q4: C] To achieve optimal results from emergency vascular repair, and to avoid complications

Table14.1  Signs of vascular injury. Updated Hard signs

Soft signs

Absent pulse Bruit or thrill Haematoma (large or expanding) Distal ischaemia

Haematoma (small) History of haemorrhage at scene Peripheral nerve deficit

14  Blast Injury to the Lower Limb

141

such as compartment syndrome or contracture due to prolonged warm ischaemia and reperfusion injury, surgical exploration should be undertaken expeditiously. A patient with complex lower limb injuries should be placed in a supine position on an operating table suitable for on-table angiography, if required, when clinical stability has been reached. Some form of warming device should be employed to maintain adequate body temperature. In lower limb trauma, both limbs should be prepared from umbilicus to toes; donor saphenous vein harvesting may be required from the contralateral limb, particularly if ipsilateral venous injury is suspected. Careful attention should be given to correct hypothermia, blood loss, electrolyte imbalance and coagulopathy. The principal aims of emergency vascular surgery are to control life-threatening haemorrhage and prevent end-organ ischaemia. [Q5: A, B] An assistant should control haemorrhage using a pressure dressing until the patient is prepared and draped appropriately. Haemorrhage control can be difficult if the proximal vessels are not immediately apparent, and the use of a cephalad incision through virgin territory may be a reasonable alternative to obtain rapid proximal control. Care should be taken when making additional incisions, particularly if it seems likely that plastic surgery will be required at a later date. When access to the proximal or distal vessel is difficult, temporary control can be gained by careful cannulation and inflation of an embolectomy catheter. It is important that the surgeon cooperates fully with the anaesthetist during surgery as it may be necessary to pack the wound for a few minutes to facilitate IV fluid resuscitation before proximal vascular control can be obtained. Complex lengthy operations should be avoided in unstable patients and damage limitation surgery should be considered in patients with significant metabolic acidosis, coagulopathy and/or hypothermia. The use of a temporary intraluminal vascular shunt should be considered in the majority of limb vascular injuries and is particularly important in complex cases with associated bone and soft tissue injury. Temporary shunts for arterial and venous injuries have been employed in Belfast since the late 1970s.2 A considerable body of evidence continues to support the use of these intravascular shunts in the management of both penetrating and blunt major vascular trauma.3–6 Before securing the shunt between the proximal and distal arteries, a careful embolectomy should be performed to remove any thrombus in the distal vessel. If a venous injury is encountered, then an additional shunt can be employed to facilitate venous return. In the absence of coagulopathy or ongoing haemorrhage we use IV heparin routinely. Recent evidence has shown clearly that delayed renewal of venous flow in combined arterial and venous injury compounds ischaemia-reperfusion injury and causes remote lung injury.7 The advantages of shunting artery and vein are the early restoration of blood flow and venous return, respectively, thus avoiding the complications of prolonged ischaemia and ischaemia-reperfusion injury while ensuring that an optimal vascular repair can be performed. In patients with concomitant fractures, accurate internal or external fixation of the fracture can be performed with the shunt secured carefully with sloops before definitive vascular repair is performed. This avoids the dilemma of unnecessary haste for both the orthopaedic and vascular surgeons, ensures that a vein graft will be of optimal length, and eliminates the risk of graft disruption during fracture manipulation. Autologous vein is our preferred bypass conduit in the majority of cases because of its durability and suitability in

142

Adrian K. Neil et al.

a potentially contaminated wound. Satisfactory results, however, have been reported using synthetic grafts and in critically ill, unstable patients this may be a preferable option.8 The acute management of high energy limb trauma can be challenging and significant morbidity and mortality can occur following failed attempts at limb salvage. A number of scoring systems have been devised in an attempt to assist the clinician’s decision to either amputate or perform a limb-salvage procedure.9–13 In each of the systems, a score is assigned based on a range of differing criteria including patient age, “mechanism of injury”, time to treatment, degree of shock, warm ischaemia time and the presence of local injuries to the following structures: major artery, major vein, bone, muscle, nerve, skin, and degree of contamination. [Q6: E] All of these scoring systems demonstrate a much higher degree of specificity than sensitivity and are more useful in highlighting the patients who should be considered for a limb-salvage procedure, than identifying those who should proceed straight to primary amputation. Indeed a number of studies have challenged their use at all.14,15 It is the authors’ opinion that scoring systems can help the surgeon perform a detailed assessment of a complex limb injury. However, the decision to perform a primary amputation must be judged individually in each case. Extensive nerve injuries have a particularly poor prognosis and it is important that such injuries, where possible, are documented before taking the patient to the operating theatre. The patient’s life should never be put at risk in a futile attempt to save a severely compromised limb. Where possible, additional specialties such as orthopaedics and plastic surgery should be involved in the decision to perform a primary limb amputation, particularly in a case of upper limb trauma. Venous injuries can be difficult to manage. Prior to World War II, the traditional treatment for lower extremity venous injuries was ligation. This custom was challenged by Debakey and Simeone16 in 1946 with an analysis of WWII battle injuries. Since then a number of clinical and laboratory investigations have confirmed that ligation of major veins in conjunction with repair of a traumatically injured arterial system leads to significantly poorer clinical outcomes, such as decreased function or even limb loss.17,18 Where possible vein repair should be attempted, particularly in the presence of significant lower limb arterial injury, in an attempt to reduce venous hypertension and associated morbidity. While there are few data regarding the long-term outcome of venous repairs, it is the authors’ impression that maintaining venous patency, in the initial few days after injury, can significantly help reduce acute post-injury swelling. If the superficial femoral vein requires ligation, it is important to maintain patency of the ipsilateral long saphenous and profunda femoris veins. Complex vein repair should never be attempted in unstable patients who have sustained major blood loss and have significant problems with hypothermia and coagulopathy. In more stable patients, however, temporary intraluminal venous shunting can facilitate the construction of larger calibre panel grafts obtained from the contralateral long saphenous vein. [Q7: D, E] Postoperative management of patients with complex limb injuries is critically important. The majority of these patients have been transferred immediately to the operating theatre and it is important that a thorough search for occult injuries is performed on admission to the intensive care unit. These patients are at risk of developing multiple organ dysfunction syndrome as a result of their large transfusion requirements and likely reperfusion injury sustained.19,20 It is important that the vascular surgeon communicates clearly

14  Blast Injury to the Lower Limb

143

with the staff in the intensive care unit regarding the presence or absence of distal pulses, to ensure that vascular repair remains patent. Young trauma patients with normal blood pressure and temperature should have a palpable distal pulse. If there is any doubt regarding the integrity of the vascular repair, the dressings should be removed and a careful assessment performed by a vascular surgeon using handheld Doppler and/or portable ultrasound device. Wounds should be reinspected 24–48 h after initial surgery and at that stage definitive plastic surgery may be required to obtain soft tissue and skin cover. [Q8: D] Some centres advocate a selective policy with regard to fasciotomy based on compartmental pressures, while many continue to advocate a more liberal policy based on clinical grounds. Prolonged ischaemia time, combined arteriovenous injuries, complex injuries including bone and soft tissue destruction and crush injuries remain absolute indications for fasciotomy. The avoidance of compartment syndrome and restoration of limb function far outweigh the low morbidity associated with liberal use of fasciotomy. These patients are at significant risk of wound and other nosocomial infections and prolonged antibiotic use may be required. The management of patients with complex injuries can be difficult; however, timely surgery and the involvement of a multidisciplinary team can produce rewarding results. One possible criticism of the above care could be failure to use the great toe, from the amputated left lower limb, to replace the patient’s right thumb.

References   1. Itani KM, Burch JM, Spjut-Patrinely V, Richardson R, Martin RR, Mattox KL. Emergency center arteriography. J Trauma. 1992;32(3):302-306. discussion 306-37.   2. Barros D’Sa AA. How do we manage acute limb ischaemia due to trauma? In: Greenhalgh RM, Jamieson CW, Nicolaides AN, eds. Limb Salvage and Amputation for Vascular Disease. London: WB Saunders; 1998.   3. D’Sa AA. A decade of missile-induced vascular trauma. Ann R Coll Surg Engl. 1982;64(1): 37-44.   4. Elliot J, Templeton J, Barros D’Sa AA. Combined bony and vascular trauma: a new approach to treatment. J Bone Joint Surg Am. 1984;66B:281.   5. Barros D’Sa AA. The rationale for arterial and venous shunting in the management of limb vascular injuries. Eur J Vasc Surg. 1989;3(6):471-474.   6. Barros D’Sa AA, Moorehead RJ. Combined arterial and venous intraluminal shunting in major trauma of the lower limb. Eur J Vasc Surg. 1989;3(6):577-581.   7. Harkin DW, D’Sa AA, Yassin MM, etal. Reperfusion injury is greater with delayed restoration of venous outflow in concurrent arterial and venous limb injury. Br J Surg. 2000;87(6): 734-741.   8. Lovric Z, Lehner V, Kosic-Lovric L, Wertheimer B. Reconstruction of major arteries of lower extremities after war injuries. Long-term follow up. J Cardiovasc Surg (Torino). 1996;37(3): 223-227.   9. Howe HR Jr, Poole GV Jr, Hansen KJ, etal. Salvage of lower extremities following combined orthopedic and vascular trauma. A predictive salvage index. Am Surg. 1987;53(4):205-208. 10. Johansen K, Daines M, Howey T, Helfet D, Hansen ST Jr. Objective criteria accurately predict amputation following lower extremity trauma. J Trauma. 1990;30(5):568-572. discussion 572-573.

144

Adrian K. Neil et al.

11. Helfet DL, Howey T, Sanders R, Johansen K. Limb salvage versus amputation. Preliminary results of the Mangled Extremity Severity Score. Clin Orthop Relat Res. 1990;256:80-86. 12. Russell WL, Sailors DM, Whittle TB, Fisher DF Jr, Burns RP. Limb salvage versus traumatic amputation. A decision based on a seven-part predictive index. Ann Surg. 1991;213(5): 473-480. discussion 480-481. 13. McNamara MG, Heckman JD, Corley FG. Severe open fractures of the lower extremity: a retrospective evaluation of the Mangled Extremity Severity Score (MESS). J Orthop Trauma. 1994;8(2):81-87. 14. Bonanni F, Rhodes M, Lucke JF. The futility of predictive scoring of mangled lower extremities. J Trauma. 1993;34(1):99-104. 15. Durham RM, Mistry BM, Mazuski JE, Shapiro M, Jacobs D. Outcome and utility of scoring systems in the management of the mangled extremity. Am J Surg. 1996;172(5):569-573. ­discussion 573-574. 16. Debakey ME, Simeone FA. Battle injuries of arteries in World War II: analysis of 2471 cases. Ann Surg. 1946;123:534-579. 17. Nanobashvili J, Kopadze T, Tvaladze M, Buachidze T, Nazvlishvili G. War injuries of major extremity arteries. World J Surg. 2003;27(2):134-139. 18. Kuralay E, Demirkilic U, Ozal E, etal. A quantitative approach to lower extremity vein repair. J Vasc Surg. 2002;36(6):1213-1218. 19. Defraigne JO, Pincemail J. Local and systemic consequences of severe ischemia and reperfusion of the skeletal muscle. Physiopathology and prevention. Acta Chir Belg. 1998;98(4):176-186. 20. Foex BA. Systemic responses to trauma. Br Med Bull. 1999;55(4):726-743.

Endovascular Management of Aortic Transection in a Multiinjured Patient

15

Shiva Dindyal and Constantinos Kyriakides

A 19-year-old female was admitted to casualty following a road traffic collision. A witness of the incident reported that she was driving her car at approximately 70km/h in wet conditions and the car skidded off the road when she turned a sharp bend. She collided with a tree and there were no other passengers involved. She was found in her severely damaged car, drowsy and restrained by her seat belt and the dashboard. The car windscreen had a “bulls-eye” on the driver’s side and she had a laceration to her forehead, which was profoundly bleeding. She complained of difficulty in breathing and pain in her chest, abdomen and obviously deformed right leg. The paramedics attended the scene with the fire-service who helped extricate her form the wreckage then carefully immobilized her cervical spine. She was immediately transported by helicopter to the nearest emergency department. There she was treated by the duty surgical trauma team.

Question 1 Which of the following interventions should be performed by the paramedics as their initial management? A.  Reduction, splinting and immobilization of her right femur fracture B.  Intravenous cannulation and bolus fluid administration C.  High flow oxygen administration D.  Administration of analgesia Primary examination in casualty revealed a patent airway as she was talking but she was short of breath. Her trachea was deviated to the right side, the left chest was hyper-resonant and devoid of breath sounds. Hemodynamically her heart rate was raised (109 beats/min) and blood pressure (120/75mmHg) was within normal limits. Her abdomen was tender in the left hypochondrium and right femur had an open mid-shaft fracture. Routine trauma blood investigations were requested. Neurologically she was drowsy and becoming increasingly confused.

S. Dindyal () Department of General Surgery, The Royal London Hospital, London, UK G. Geroulakos and B. Sumpio (eds.), Vascular Surgery, DOI: 10.1007/978-1-84996-356-5_15, © Springer-Verlag London Limited 2011

145

146

S. Dindyal and C. Kyriakides

Question 2 Which is the most appropriate initial investigation required? A.  Computerized tomography of her head and neck B.  Plain radiographs of the pelvis and right femur C.  Computerized tomography of her abdomen and pelvis D.  Plain portable chest radiograph Her heart rate further increased (120 beats/min) and blood pressure reduced (110/65mmHg). She was visibly more confused and breathing was more labored, whilst her abdomen had also become distended. Hemodynamically she was a transient responder to a bolus intravenous fluid replacement.

Question 3 Which is the most appropriate immediate intervention required? A.  Chest drain insertion B.  Emergency laparotomy and damage control surgery C.  Reduction, splinting and immobilization of her right femur fracture D.  Diagnostic peritoneal lavage Chest imaging revealed a widened mediastinum and left tension pneumothorax. Immediate left chest needle decompression revealed a “whoosh of air” and the trachea centralized (Fig.15.1). Consequently a left chest drain was inserted. The patient was becoming more confused and combative with a reducing Glasgow Coma Scale (GCS 7), so was intubated and sedated. Initial blood results revealed a low hemoglobin, however her hemodynamics

Fig.15.1  Chest radiograph showing a widened mediastinum and left tension pneumothorax

15  Endovascular Management of Aortic Transection in a Multiinjured Patient

147

had returned to values within normal ranges with a continuous fluid infusion and whole blood transfusion. A plain pelvic radiograph and clinical examination were normal.

Question 4 Which investigation or treatment should be performed next? A. Plain radiographs of the right femur then reduction, splinting and immobilization of her right femur fracture B.  Computerized tomography of head, neck, chest, abdomen and pelvis C.  Emergency laparotomy and damage control surgery D.  Diagnostic angiography Imaging revealed that she had suffered from polytraumatic injuries. She had bilateral cerebral contusions, left clavicle and cervical vertebra (Fig.15.2) fractures, multiple rib fractures including the left first rib with a left hemo-pneumothorax and bilateral lung contusions. Her thoracic aorta was disrupted and a pseudoaneurysm of the proximal descending vessel had formed (Figs.15.3 and 15.4). Her abdominal imaging revealed a liver laceration, large splenic hematoma and free abdominal fluid suggestive of bleeding. Her pelvis was normal but she had an open, displaced fracture of her right femoral shaft. Clinically she was becoming increasing more difficult to ventilate, and was deteriorating hemodynamically. Her left chest drain continued to swing and bubble however blood was also still draining. Her abdomen had become more distended. She was in hypovolemic shock and was no longer responding to intravenous fluid and blood administration. Her right thigh wound was becoming more tense and swollen. An arterial blood gas revealed that she was suffering a metabolic acidosis, with a raised lactate, and her hemoglobin level had further dropped. She was taken immediately to the operating room. She underwent an emergency laparotomy, splenectomy and packing of her liver. Her right femoral shaft fracture was debrided, irrigated, reduced then immobilized with a splint. An intracranial bolt was inserted for pressure measurements. The duty vascular surgeon was called to assess her transected thoracic aorta, he scrutinized the Computerized Tomographic chest imaging.

Question 5 Using Fig.15.5 below, which is the correct list order of the commonest anatomical sites of traumatic aortic disruption starting with the most frequent to the least common in descending order? A.  1, 2, 3, 4 B.  4, 2, 3, 1 C.  3, 1, 2, 4 D.  3, 1, 4, 2 E.  1, 4, 2, 3

148 Fig.15.2  MRI showing a cervical vertebral fracture

S. Dindyal and C. Kyriakides

15  Endovascular Management of Aortic Transection in a Multiinjured Patient Fig.15.3  CT scan reconstruction showing a disrupted thoracic aorta with a pseudoaneurysm of the proximal descending vessel

149

150

S. Dindyal and C. Kyriakides

Fig.15.4  CT scan crosssectional slice showing a disrupted thoracic aorta with a pseudoaneurysm of the proximal descending vessel

3 2 1

4

Fig.15.5  Anatomical sites of traumatic aortic disruption

1 = Ascending Aorta 2 = Innominate Artery 3 = Ligamentum arteriosum 4 = Lower descending aorta

15  Endovascular Management of Aortic Transection in a Multiinjured Patient

151

Question 6 Which of the following is a favorable feature for thoracic endovascular aortic stent graft access? A.  Tortuous iliac arteries B.  Iliac diameter 

Vascular Surgery: Cases, Questions and Commentaries, Third Edition - PDF Free Download (2024)
Top Articles
Latest Posts
Article information

Author: Mrs. Angelic Larkin

Last Updated:

Views: 6295

Rating: 4.7 / 5 (67 voted)

Reviews: 82% of readers found this page helpful

Author information

Name: Mrs. Angelic Larkin

Birthday: 1992-06-28

Address: Apt. 413 8275 Mueller Overpass, South Magnolia, IA 99527-6023

Phone: +6824704719725

Job: District Real-Estate Facilitator

Hobby: Letterboxing, Vacation, Poi, Homebrewing, Mountain biking, Slacklining, Cabaret

Introduction: My name is Mrs. Angelic Larkin, I am a cute, charming, funny, determined, inexpensive, joyous, cheerful person who loves writing and wants to share my knowledge and understanding with you.